Тест. Основы экономической теории — ответы к тесту
Тест по экономической теории, по теме изучение основ экономики. Предназначен для тестирования знаний студентов, по пройденному курсу. Всего в тесте 149 вопросов. Не правильные варианты ответов — удалены.
Вопрос 1. Какая из школ экономической теории была исторически первой: Ответ: Меркантилизм.
Вопрос 2. Если исследуется экономика как целостная система, то это анализ: Ответ: Макроэкономический.
Вопрос 3. Какова экономическая цель, если общество стремится минимизировать издержки и максимизировать отдачу от ограниченных производственных ресурсов? Ответ: Экономическая эффективность.
Вопрос 4. Что изучает экономическая теория: Ответ: Законы экономического развития и производственные отношения.
Вопрос 5. Представители меркантилизма считали: Ответ: Совокупность перечисленного.
Вопрос 6. Что из перечисленного изучает микроэкономика: Ответ: Побудительные мотивы в поведении индивида.
Вопрос 7. Экономическая теория пригодна для изучения: Ответ: Всех экономических систем.
Вопрос 8. В чем главное содержание учения физиократов: Ответ: Источники богатства — сельскохозяйственный труд.
Вопрос 9. Позитивная экономика изучает: Ответ: «Что есть».
Вопрос 10. Что из перечисленного не изучает макроэкономика: Ответ: Заработная плата рабочего завода.
Вопрос 11. Какое из перечисленных положений принадлежит сторонникам кейнсианской школы: Ответ: Необходимо государственное вмешательство в развитие рыночной экономики.
Вопрос 12. Какое из перечисленных положений не имеет отношения к определению предмета экономической теории? Ответ: Неограниченность производственных ресурсов.
Вопрос 13. К экономическим законам не относятся: Ответ: Редкость экономических благ.
Вопрос 14. Экономические законы выражают: Ответ: Необходимые, устойчивые, повторяющиеся причинно-следственные связи между экономическими процессами и явлениями.
Вопрос 15. Экономическая теория используется для выработки конкретной экономической политики, тем самым ею выполняются функции: Ответ: По формированию экономического образа мышления.
Вопрос 16. Для студентов альтернативную ценность обучения в институте отражает: Ответ: Максимальный заработок, который можно получать, бросив учебу.
Вопрос 17. Как можно охарактеризовать объем потребностей человека? Ответ: Безграничные.
Вопрос 18. Что из следующего перечня благ является неэкономическим благом? Ответ: Энергия ветра.
Вопрос 19. К экономическим благам относят блага для создания, которых требуется Ответ: Затраты человеческого труда.
Вопрос 20. Редкость ресурсов — это: Ответ: Концепция, отражающая невозможность полного удовлетворения человеческих потребностей.
Вопрос 21. Проблема относительной ограниченности ресурсов в рыночной экономике объясняется: Ответ: Опережением потребностями имеющихся ресурсных возможностей.
Вопрос 22. Общий уровень цен и безработицы в экономической системе изучается в курсе: Ответ: Макроэкономики.
Вопрос 23. Фундаментальная проблема, с которой сталкиваются все экономические системы это: Ответ: Ограниченность ресурсов.
Вопрос 24. Проблема относительной ограниченности ресурсов в рыночной экономике объясняется: Ответ: Нехваткой (дефицитом) ресурсов.
Вопрос 25. Какой из указанных относится к общественному товару: Ответ: Уличный светофор.
Вопрос 26. Когда экономисты говорят об ограниченности , они подразумевают, что: Ответ: Природных ресурсов недостаточно, чтобы удовлетворить все потребности всех людей.
Вопрос 27. К потребительским товарам кратковременного пользования относятся: Ответ: Картофель в домашнем хозяйстве.
Вопрос 28. Чем потребности человека отличаются от спроса? Ответ: Потребности индивидуума — неограниченны, а спрос — ограничен.
Вопрос 29. Собственность как экономическая категория выражает: Ответ: Отношения между людьми по поводу присвоения материальных благ и услуг.
Вопрос 30. Чем отличается частная собственность от личной: Ответ: Частная собственность приносит доход.
Вопрос 31. Какие отношения выражает экономическая категория «собственность»? Ответ: Отношения по поводу присвоения средств и результатов производства.
Вопрос 32. Что такое «отчуждение»? Ответ: Это невозможность превращения жизненных благ, производственных и финансовых ресурсов в собственную жизнедеятельность человека.
Вопрос 33. Что такое собственность как экономическая категория? Ответ: Предметы потребления
Вопрос 34. Какая пара категорий раскрывает экономическое содержание собственности? Ответ: Присвоение-отчуждение.
Вопрос 35. Как можно охарактеризовать право владения? Ответ: Право исключительного физического контроля над благами.
Вопрос 36. Чем характеризуется право распоряжения? Ответ: Возможность определять фактическую или юридическую судьбу вещи.
Вопрос 37. В Республике Казахстан по Конституции существуют следующие формы собственности: Ответ: Частная и государственная.
Вопрос 38. Какой вид собственности является определяющим в условиях экономической свободы: Ответ: Частная.
Вопрос 39. Когда собственность реализует себя экономически? Ответ: Когда она начинает приносить доход.
Вопрос 40. Приватизация — это: Ответ: Приобретение собственности физическими и юридическими лицами у государства.
Вопрос 41. Приватизация государственной собственности — это: Ответ: Национализация собственности.
Вопрос 42. Разгосударствление представляет собой: Ответ: Снятие с государства функции полного управления экономическими процессами.
Вопрос 43. Разгосударствление собственности — это: Ответ: Снятие с государства права единоличного владения и выполнения хозяйственных функций;
Вопрос 44. Экономическое содержание процессов приватизации и разгосударствления: Ответ: Создание многообразных форм собственности.
Вопрос 45. Что не является целью приватизации в Республике Казахстан? Ответ: Дифференциация доходов населения.
Вопрос 46. Необходимость многообразия форм собственности в рыночной экономике определяется: Ответ: Многообразием экономических интересов, существующих в экономической системе
Вопрос 47. Процесс передачи государственных объектов собственности и имущества в частную или в частно-коллективную собственность: Ответ: Приватизация.
Вопрос 48. К характеристике натурального хозяйства относится: Ответ: Универсальный труд производителя.
Вопрос 49. Натуральное хозяйство характеризуется тем, что: Ответ: Все ответы верны.
Вопрос 51. Товар- это: Ответ: Экономическое благо, предназначенное для обмена.
Вопрос 52. Два свойства товара: Ответ: Потребительная и меновая стоимость.
Вопрос 53. Предпосылкой товарной организации производства является: Ответ: Общественное разделение труда.
Вопрос 54. Что не относится к функциям денег: Ответ: Средство осуществления бартерных сделок.
Вопрос 55. Проблемы: «что, как и для кого производить?» — могут иметь отношение: Ответ: К любой экономической системе.
Вопрос 56. Фундаментальная проблема, с которой сталкиваются все экономические системы это: Ответ: Ограниченность ресурсов.
Вопрос 57. Когда экономические проблемы решаются частично рынком, частично правительством, то экономика: Ответ: Смешанная.
Вопрос 58. Экономическая система решает следующие вопросы: Ответ: Что, как, для кого производить.
Вопрос 59. К специфическим чертам переходной экономики относится: Ответ: Неустойчивость.
Вопрос 60. Что не характерно для командной экономики: Ответ: Индикативное планирование.
Вопрос 61. Экономическая система, в которой традиции, обычаи, опыт определяют практическое использование производственных ресурсов: Ответ: Традиционная.
Вопрос 62. Экономическая система, основанная на частной собственности, хозяйственное развитие которой регулируется исключительно ценовым механизмом: Ответ: Рыночная.
Вопрос 63. Основной признак традиционной экономической системы: Ответ: Частные мелкие товаропроизводители и государственные предприятия.
Вопрос 64. Основной признак административно-командной системы: Ответ: Преобладания частного сектора.
Вопрос 65. Назовите главную из перечисленных экономических функций государства в cовременной смешанной экономике: Ответ: Создание правовой основы.
Вопрос 66. Кругооборот доходов и расходов в смешанной экономике характеризует взаимодействие: Ответ: Домашних хозяйств, фирм и государства.
Вопрос 67. Один из принципов, обязательных условий рыночной экономики: Ответ: Экономическая свобода.
Вопрос 68. По территориальному признаку выделяют рынки: Ответ: Местные, национальные и мировые.
Вопрос 69. Чем характеризуется рыночная экономика: Ответ: Все перечисленное верно.
Вопрос 70. Рыночную экономику не характеризует: Ответ: Централизованное планирование производства товаров и услуг.
Вопрос 71. Какой экономический субъект в наибольшей степени определяет уровень производства в рыночной экономике: Ответ: Предприниматель.
Вопрос 72. Тип рынка, на котором оперирует небольшое количество крупных фирм: Ответ: Олигополия.
Вопрос 73. Рынок, где множество фирм продают совершенно одинаковые товары и ни одна фирма не обладает достаточно большой долей рынка, способной влиять на рыночную цену товара: Ответ: Чистая или совершенная конкуренция.
Вопрос 74. Чтобы получить максимум прибыли, монополист должен выбрать такой объем выпуска, при котором: Ответ: Предельный доход равен предельным издержкам.
Вопрос 75. Условие максимизации прибыли только для фирмы, функционирующей на рынке совершенной конкуренции Ответ: Правило MR= MC=P.
Вопрос 76. Чем характеризуется рынок олигополии: Ответ: Господством несколько крупных компаний; Несколько затруднен вход на рынок; Производит как однородные, так и разнородные товары; Контроль над ценой может быть как значительный при «тайном голосовании», так и незначительный если фирмы действуют разрозненно.
Вопрос 77. Единственный в отрасли продавец продукта, не имеющего заменителей: Ответ: Монополия.
Вопрос 78. Если рынок по своей структуре является монополией, это означает, что: Ответ: На рынке действует один продавец, который назначает цену на свою продукцию.
Вопрос 79. В отличие от конкурентной фирмы простая монополия стремится: Ответ: Максимизировать прибыль.
Вопрос 80. Рынок совершенной конкуренции: Ответ: Отличается множеством покупателей и продавцов.
Вопрос 81. Монополистическая конкуренция связана с производством: Ответ: Дифференцированного продукта
Вопрос 82. Монополия связана с лицензированием деятельности поскольку производится: Ответ: Уникальный продукт.
Вопрос 83. Понятие «совершенная конкуренция» предполагает, что: Ответ: Значительное число фирм в отрасли выпускает стандартные товары; Имеется много покупателей, приобретающих этот товар по текущей цене; Все продавцы и покупатели имеют полную информацию о рынке; Объем производства незначительный
Вопрос 84. Укажите область, где, по Вашему мнению, может быть эффективная монополия: Ответ: Водопроводные сети.
Вопрос 85. Что относится к условиям совершенной конкуренции: Ответ: Свободное ценообразование.
Вопрос 86. Если в какой — то отрасли несколько предприятий контролирует подавляющую часть рынка данной отрасли, то сложившаяся конкуренция называется: Ответ: Олигополия.
Вопрос 87. В условиях совершенной конкуренции фирма обычно: Ответ: Не оказывает влияния на формирование рыночной цены.
Вопрос 88. Антимонопольная политика должна обеспечить: Ответ: Поддержание конкурентной борьбы.
Вопрос 89. Антимонопольная политика государства прямым образом связана с: Ответ: Защитой прав потребителя.
Вопрос 90. Принцип рыночной экономики: Ответ: Свободное ценообразование.
Вопрос 91. Монополия это рыночная структура, где оперирует: Ответ: Только один продавец (фирма) на множество покупателей.
Вопрос 92. Когда экономические проблемы решаются частично рынком, частично правительством, то экономика: Ответ: Смешанная.
Вопрос 93. В условиях монополии вход в отрасль: Ответ: Блокирован.
Вопрос 94. Совершенная конкуренция предполагает производство: Ответ: Стандартного продукта.
Вопрос 95. Монополия связана с лицензированием деятельности поскольку производится: Ответ: Уникальный продукт.
Вопрос 96. Если два товара являются взаимозаменяемыми, то рост цен на первый товар вызовет: Ответ: Повышение спроса на второй товар.
Вопрос 97. Что произойдет со спросом, если доходы покупателей (при прочих равных условиях) возрастут: Ответ: Кривая спроса переместится вправо.
Вопрос 98. Закон спроса отражает: Ответ: Обратную зависимость между спросом и ценой.
Вопрос 99. В чем разница между спросом и объемом спроса? Ответ: Спрос характеризуется изменением неценовых факторов, а объем спроса — ценовыми факторами.
Вопрос 100. Какой термин отражает способность и желание людей платить за что-либо: Ответ: Спрос.
Вопрос 101. Закон спроса предполагает, что: Ответ: Когда цена товара падает, объем планируемых покупок растет.
Вопрос 102. Какую зависимость величины спроса на товар от рыночной цены на него выражает закон спроса: Ответ: Обратную зависимость.
Вопрос 103. Ожидание снижения цен на хозяйственное мыло в ближайшем будущем, в настоящее время вызовет сдвиг: Ответ: Кривой спроса влево и вниз.
Вопрос 104. Если цена на фотопленку выросла, то можно ожидать, что результатом этого станет… Ответ: Уменьшение спроса на фотоаппарат.
Вопрос 105. Рыночный спрос не испытывает влияния: Ответ: Цен на ресурсы.
Вопрос 106. Изменение какого фактора не вызывает сдвиг кривой спроса? Ответ: Цены товара.
Вопрос 107. Взаимозаненяющими товарами называются товары, которые: Ответ: Могут заменять друг друга.
Вопрос 108. Когда спрос на видеомагнитофоны возрастет, то спрос на видеокассеты: Ответ: Увеличится.
Вопрос 109. Если спрос на жевательную резинку эластичен по цене, то уменьшение цены на нее приведет к: Ответ: Снижению расходов на жевательную резинку.
Вопрос 110. Функция предложения отражает: Ответ: Взаимосвязь предложения и цены.
Вопрос 111. Понижение цен на бензин вызовет сдвиг: Ответ: Вправо кривой предложения автомобилей.
Вопрос 112. Факторы рыночного предложения товаров и услуг: Ответ: Цены на ресурс.
Вопрос 113. Совершенствование технологии сдвигает: Ответ: Кривую предложения вправо и вниз.
Вопрос 114. Какой из факторов ближе к предложению товаров, чем к спросу: Ответ: Технология и эффективность производства.
Вопрос 115. Все условия, кроме названных ниже, неизменны. Рост объема предложения — результат: Ответ: Роста рыночной цены.
Вопрос 116. Спрос на благо эластичен, если ценовая эластичность спроса: Ответ: Больше 1.
Вопрос 117. Спрос на благо неэластичен, если ценовая эластичность спроса: Ответ: Меньше 1.
Вопрос 118. Спрос на ресурсы считается эластичным, если: Ответ: Все ответы верны.
Вопрос 119. Если, несмотря на изменения цены товара, объем спроса на данный товар не изменяется, то коэффициент ценовой эластичности: Ответ: Равен 0.
Вопрос 120. В результате снижения цены на товар на 5% объем покупок данного товара вырос на 3%, тогда спрос является: Ответ: Неэластичным.
Вопрос 121. Кривая абсолютно неэластичного спроса представляет собой: Ответ: Вертикальную линию.
Вопрос 122. Эластичность предложения определяется отношением: Ответ: Изменение предложения в % к изменению цены в %.
Вопрос 123. Если спрос на товар Х неэластичный, то снижение его цены на 10% приведет к: Ответ: Увеличению объема спроса менее чем на 10 %.
Вопрос 124. Предельная полезность-это Ответ: Полезность, которую потребитель получает от потребления дополнительной единицы блага.
Вопрос 125. Линия, графически отображающая множество наборов продуктов, приобретение которых требует одинаковых затрат: Ответ: Линия бюджетного ограничения.
Вопрос 126. Предположим, что потребитель имеет доход в 8 долл. Цена товара равна 1 долл, а цена товара В-2 долл. Какая из следующих комбинаций товаров находится на бюджетной линии? Ответ: 4А и 4В.
Вопрос 127. Что понимают под » полезностью» товара: Ответ: Потребительную стоимость товара.
Вопрос 128. Укажите неправильное утверждение: Ответ: Все точки на кривой безразличия означают одинаковый уровень денежного дохода.
Вопрос 129. Что означает ситуация потребительского равновесия? Ответ: Максимизируется общая полезность при имеющимся бюджетном ограничении.
Вопрос 130. Потребительское равновесие на карте безразличия — это: Ответ: Та точка, в которой наклон бюджетной линии равен наклону касательной к ней кривой безразличия.
Вопрос 131. Какой из следующих перечней значений общей полезности иллюстрирует Закон убывающей предельной полезности? Ответ: 200, 250, 270, 280.
Вопрос 132. Положение и наклон кривой безразличия для отдельного потребителя объясняется: Ответ: Только его предпочтениями.
Вопрос 133. Что означает кривая безразличия: Ответ: Линию одинаковой общей полезности.
Вопрос 134. Общая полезность растет. Когда предельная полезность: Ответ: Уменьшается.
Вопрос 135. Чтобы оказаться в положении равновесия потребитель должен: Ответ: Максимизировать общую полезность.
Вопрос 136. Какое из следующих утверждений является неверным? Ответ: Все точки бюджетной линии означают одинаковый уровень полезности.
Вопрос 137. Воспроизводство — это: Ответ: Постоянное повторение процесса производства.
Вопрос 138. Процесс воспроизводства предполагает: Ответ: Воспроизводство производственных отношений.
Вопрос 139. Простое воспроизводство предполагает: Ответ: Производство в неизменных масштабах.
Вопрос 140. Расширенное воспроизводство предполагает: Ответ: Увеличение размеров производства.
Вопрос 141. Основной капитал фирмы представляет собой: Ответ: Капитал, который многократно используется в производственных циклах и переносит стоимость по частям.
Вопрос 142. Для предприятия амортизационный фонд необходим в целях: Ответ: Накопления средств на возмещение или воспроизводство основного капитала;
Вопрос 143. Оборотный капитал фирмы представляет собой: Ответ: Капитал который используется полностью и переносит стоимость в течении одного производственного цикла.
Вопрос 144. Основные производственные фонды — это: Ответ: Созданная на производстве готовая продукция.
Вопрос 145. Амортизация-это: Ответ: Ежегодные отчисления, равные стоимости потребленного за этот период основного капитала;
Вопрос 146. Скорость оборота капитала — это: Ответ: Число оборотов ресурсов, совершаемых в течение года.
Вопрос 147. Оборотные производственные фонды — это: Ответ: Материальные средства, полностью используемые в течении одного производственного цикла и включаемые в стоимость готовой продукции.
Вопрос 148. Когда на предприятии имеет место моральный износ оборудования? Ответ: Когда созданы аналогичные имеющимся, но более дешевые средства труда или же выпущено более производительное оборудование.
Вопрос 149. Начисление износа на оборудование — это: Ответ: Амортизация
вы прирожденный бизнесмен, если наберете 11/15
Тест по теме основы экономики не могут пройти даже опытные бухгалтеры. #wzss» data-test-id=»1334″ data-post-id=»5682″ data-answer-count=»4059″> Человек, средства производства и научно-технический прогресс
Вопрос 2 из 15
Что является предметом труда?
Продукт процесса производства
Средство, с помощью которого производится продукт
То, на что направлен труд и из чего в результате получается продукт труда
Вопрос 3 из 15
Что можно отнести к средствам труда?
То, из чего получается продукт труда
То, на что направлен человеческий труд
То, с помощью чего человек воздействует на предмет труда
Вопрос 4 из 15
Что можно отнести к фактором производства?
Естественные ресурсы
Станки, машины, сырье
Способы производства товаров и услуг
Все перечисленное
Вопрос 5 из 15
Для чего нужна экономическая модель?
Для определения количественных параметров и качественного уровня развития экономики, к которому следует стремиться
Для объяснения того, как функционирует мировая экономика
Для раскрытия экономических принципов и законов
Вопрос 6 из 15
Что изучает микроэкономика?
Рациональные экономические решения
Ценообразование
И то, и другое
Вопрос 7 из 15
Какая школа экономической теории была исторически первой?
Маржинализм
Марксизм
Меркантилизм
Вопрос 8 из 15
Что означает ситуация, когда экономика движется по выпуклой кривой производственных возможностей вправо и вниз?
Альтернативные издержки увеличиваются
Альтернативные издержки уменьшаются
Альтернативные издержки остаются неизменными
Вопрос 9 из 15
Чем характеризуется разделение труда?
Обособлением предметов труда от средств труда
Отделением работника от средств производства
Обособлением различных видов трудовой деятельности
Вопрос 10 из 15
Что понимается под термином «собственность»?
Отношение людей друг к другу по поводу присвоения вещей
Отношение человека к вещи
Сама вещь
Вопрос 11 из 15
Что из этого является субъектом собственности?
Земля
Производственные здания и сооружения
Государство
Вопрос 12 из 15
На основе каких отношений собственник земли присваивает ренту, если он сдает ее в аренду фермеру?
Пользования
Владения
Распоряжения
Вопрос 13 из 15
Что такое товар?
Вещь, обмениваемая на другую вещь или деньги
Вещь, являющаяся продуктом человеческого труда
Вещь, обладающая потребительской стоимостью
Вопрос 14 из 15
На чем базируется трудовая теория стоимости Карла Маркса?
На равновесной цене на рынке труда
На двойственном характере труда
На предельном продукте труда
Вопрос 15 из 15
Возникновение кредитных денег связано с функцией денег:
Как средства накопления
Как меры стоимости
Как средства платежа
Комментарии
Основы экономики | 122 воспроизведения
Поиск среди миллионов викторин
ВИКТОРИНА
Специальность
57%
точность
122
играет
Конни
Завтра
6 лет
Специальность
Конни Морроу
122
играет
20 вопросов
20
вопросы
Показать ответы
См. предварительный просмотр
1. Множественный выбор
30 секунд
1 балл отказаться, когда вы делаете выбор.
Дефицит
Факторы производства
Инновации
Альтернативная стоимость
2. Множественный выбор
30 секунд
1 балл
Есть 4 _______________. Это земля, труд, капитал и предпринимательство.
Факторы производства
Рыночная экономика
Прибыль
Экономика
3. Множественный выбор
30 секунд
1 балл
Группа покупателей и продавцов, которые собираются вместе для торговли
Торговля
Рынок
Макроэкономика
Экономическая модель
4. Множественный выбор
30 секунд
1 балл
Что из следующего является примером инвестиций в человеческий капитал?
Компания строит новый завод
Государство прокладывает новое шоссе
Человек идет в колледж
Страна строит новые школы
5. Множественный выбор
30 секунд
90 004 1 балл
Экономика, в которой Правительство решает, как будут распределяться ВСЕ экономические ресурсы, например, Северная Корея
Рыночная экономика
Смешанная экономика
Централизованно-плановая экономика
Предприниматель
6. Множественный выбор
30 секунд
1 балл
Искусственные ресурсы, используемые в производственном процессе, т. е. машины на заводе.
Труд
Капитал
Домохозяйство
Доход
7. Множественный выбор
30 секунд
1 балл
Доход от продаж (также известный как выручка) за вычетом затрат на содержание бизнес.
Компромисс
Экономическая модель
Фактор производства
Прибыль
8. Множественный выбор
30 секунд 900 03
1 балл
Организация земли, труда и капитала (факторы производства)
Предпринимательство
Смешанная экономика
Прибыль
Плановая экономика
9. Множественный выбор
30 секунд
1 балл
_______________это идея о том, что из-за дефицита производство большего количества одного товара или услуги означает производство меньшего количества другого товара или услуги.
Дефицит
Структура рынка
Компромисс
Экономика
10. Множественный выбор
30 секунд
1 точка
Ситуация, при которой неограниченные потребности превышают ограниченные ресурсы, доступные для удовлетворения этих желаний.
Рынок
Компромисс
Дефицит
Капитал
11. Множественный выбор
30 секунд 90 003
1 pt
Плата за землю, труд, капитал и предпринимательскую способность составляет соответственно:
рента, прибыль, заработная плата и проценты
прибыль, заработная плата, проценты и рента
рента, заработная плата, проценты и прибыль
заработная плата, рента, прибыль и проценты
12. Множественный выбор
30 секунд
1 балл
Рыночная экономика имеет преимущество перед традиционной экономики в том, что рыночная экономика
контролирует цены и заработную плату
фиксирует доходы потребителей
ограничивает прибыль крупных корпораций
приспосабливается к потребительскому спросу с течением времени
13. Множественный выбор
30 секунд
1 балл
В какой экономической системе государство владеет средствами производства?
традиционный
смешанный
рынок
командный
14. Множественный выбор
30 секунд
900 04 1 pt
Как в условиях рыночной экономики решаются основные экономические вопросы что, как, а за кого выдавать ответы?
с использованием национальных социальных обычаев и традиций
с использованием комбинации традиционной и командной экономики
отдельными лицами и фирмами на национальном рынке
федеральными и/или местными органами власти страны
15 , Несколько -выбор
30 секунд
1 балл
Фабрика является частью какого из следующих факторов производства?
капитал
труд
земля
предпринимательство
16. Множественный выбор
30 секунд
900 04 1 pt
Белинда занимается изготовлением стальных декоративных статуй для газонов. Дуговая сварка, которую она использовала для соединения кусков стали, считается:
человеческий капитал
предпринимательство
труд
капитал
17. Множественный выбор
30 секунд
1 балл
Что из перечисленного НЕ считается дефицитным ресурсом?
сырая нефть
время
выбросы загрязняющих веществ
сельскохозяйственные угодья
18. Множественный выбор
30 секунд 9000 3
1 pt
Товар, который используется вместо другого товара для удовлетворения такая же потребность называется a:
дополнение
основной товар
замена
потребительский товар
19. Множественный выбор
30 секунд
9 0004 1 балл
Какая из следующих характеристик лучше всего определяет командную экономику?
неравенство в распределении доходов
государственная собственность на средства производства
частная собственность на средства производства
децентрализация принятия экономических решений
20. Множественный выбор
30 секунд
1 pt авиакомпания, телеканалы, больницы, университеты и оборона подрядчики. Однако в большинстве других отраслей участвуют частные фирмы, которые конкурируют за бизнес. Что из следующего лучше всего описывает тип экономики Англии?
традиционный
команда
рынок
смешанный
Рассказать все вопросы с бесплатной учетной записью
Уже есть учетная запись?
25+ вопросов с несколькими вариантами ответов по экономике
Викторина по экономике:
Вопросы. В экономике обычно считается, что основная цель банка состоит в том, чтобы (a) Нанять все больше и больше людей (b) Максимизировать общую прибыль (c) Максимально увеличить общий объем производства (d) Предоставлять финансовые услуги населению по всей стране
Ans. (d)
Вопрос. Рынок продавцов обозначает ситуацию, когда (a) Товары доступны по конкурентоспособным ценам (b) Спрос превышает предложение (c) Предложение превышает спрос (d) Спрос и предложение равны
Ответ. (b)
Вопрос. Для достижения высоких темпов роста национального производства экономика должна (а) Снизить темпы роста населения (b) Занять иностранный капитал (c) Увеличить норму сбережений (d) Увеличить норму инвестиций и снизить норму выпуска капитала
Отв. (d)
Вопрос. Фундаментальной причиной краха Бреттонвудской системы было: (a) Проблема ликвидности (b) Проблема корректировки (c) Проблема уверенности (d) Все эти
Ответ. (d)
Вопрос. Развитие означает экономический рост плюс (a) Инфляция (b) Дефляция (c) Стабильность цен (d) Социальные изменения
Анс. (d)
Связанный: тип теста на безработицу
Ques. Направление торговли указывает: (a) Характер товаров, которые экспортируются и импортируются (b) Стоимость товаров, которые экспортируются и импортируются (c) Страны, в которые экспортируются товары, и страны, из которых товары импортируются (d) Условия обмена экспорта на импорт
Ответ. (c)
Вопрос. Относительная бедность в основном связана с (a) Бедные (b) Богатые (c) Оба (d) Нет
Ответ. (b)
Вопрос. Какая из следующих стран имеет максимальную сумму внешнего долга? (a) США (b) Мексика (c) Бразилия (d) Япония
Ответ. (а)
Вопрос. Какая из следующих теорий принадлежит Карлу Марксу? (а) Функциональная теория (b) Эволюционная теория (c) Теория классовой борьбы (d) Ничего из перечисленного
Ответ. (c)
Связанные: вопросы викторины по инфляции
Ques. Смешанная экономика предусматривает: (a) Сосуществование капиталистов и рабочих (b) Интегрированное экономическое развитие (c) Одновременное развитие сельского хозяйства и промышленности (d) Частный и государственный секторы
Ans. (d)
Вопрос. Какой из следующих терминов используется в экономике? (a) Кейнсианский (b) Адсорбция (c) Матрица сродства (d) Поток генов
Анс. (а)
Вопрос. Важными целями развития кооперативного сектора являются: (a) Предотвращение концентрации экономической власти (b) Более широкое рассредоточение собственности на производственные ресурсы (c) Активное вовлечение людей в программы развития (d) все из них
Анс. (d)
Вопрос. Какой из следующих терминов не используется в экономике? (a) Спрос и предложение (b) Адвалорный налог (c) Безубыточность (d) ВИЧ-положительные
Ответ. (d)
Связанные: страны и валюты
Ques. Кого называют отцом экономики? (a) Нельсон Мандела (b) Васко де Гама (c) Адам Смит (d) Нильс Бор
Анс. (c)
Вопрос. Когда слишком много денег преследует слишком мало товаров, ситуация __ (a) Дефляция (b) Инфляция (c) Рецессия (d) Стагфляция
Ответ. (b)
Вопрос. Глобализация способствует (a) индустрии культуры (b) окружающей среде (c) экономике (d) налогам
Ans. (c)
Вопрос. Маркс принадлежал— (а) Германии (б) Голландии (в) Франции (г) Англии
Анс. (a)
Связанный: вопросы экономического развития и ответы
Ques. Экономическое развитие идет двумя путями. Какой путь? (a) Урбанизация и индустриализация (b) Модернизация и индустриализация (c) Урбанизация и модернизация (d) Секуляризм и либерализм
Анс. (а)
Вопрос. Термины «Микроэкономика» и «Макроэкономика» были введены — (a) Альфредом Маршаллом (b) Рагнером Нурксе (c) Рагнером Фришем (d) Дж. М. Кейнсом
Анс. (c)
Вопрос. Экономическая доктрина Laissez faire просто означает (a) Оставить все в покое (b) Регулируемая торговля (c) Вмешательство государства в торговлю (d) Ничего из перечисленного
Ответ. (а)
Вопрос. В период инфляции налоговые ставки должны— (a) Увеличиваться (b) Уменьшаться (c) Оставаться постоянными (d) Колебаться
Ответ. (a)
Связанный: управленческая экономика важные вопросы и ответы
Ques. Как экономическое учение меркантилизм характеризовался (a) Контроль за экономической деятельностью (b) свободная торговля (c) Ценовой механизм спроса и предложения (d) открытый рынок
Ans. (а)
Вопрос. «Закрытая экономика» – это экономика, в которой ____ (a) имеет место только экспорт (b) денежная масса полностью контролируется (c) имеет место финансирование дефицита (d) не происходит ни экспорта, ни импорта
Ответ. (d)
Вопрос. Первая фаза капитализма обычно известна как (a) Промышленный капитализм (b) Финансовый капитализм (c) Торговый капитализм (d) Капитализм ТНК
Ans. (c)
Вопрос. Отсутствие доступа к финансовым услугам технически известно как (a) Финансовая нестабильность (b) Финансовая стабильность (c) Финансовая доступность (d) Финансовая недоступность
Ответ. (d)
Связанный: Викторина по международной торговле
Ques. Деньги (а) Постоянная покупательная способность (b) Приемлем только в том случае, если он имеет внутреннюю стоимость (c) Самый ликвидный из всех активов (d) Ничего из перечисленного
Ответ. (c)
Вопрос. Что из нижеперечисленного вообще свободно от торговых барьеров и где экспорт и импорт составляют большую часть ВВП? (a) Современная экономика (b) Открытая экономика (c) Закрытая экономика (d) Макроэкономика
Ответ. (b)
Вопрос. Кто отец капитализма? (а) Адам Смит (b) Владимир Ленин (c) Жан-Жак Руссо (d) Мартин Лютер Кинг мл.
Анс. (а)
Вопрос. ВВП страны обычно рассчитывается (a) Международным валютным фондом (b) Системой национальных счетов (c) Министром финансов (d) Национальным статистическим агентством
Ответ. (d)
Связанный: Что такое чек получателя счета?
Вопросы. Экономическая деятельность, тем не менее, сгруппирована в разные категории, в высшей степени (а) независимые (б) автономные (в) несвязанные (г) взаимозависимые
Вопрос. Спрос в экономике означает (a) Совокупный спрос (b) Рыночный спрос (c) Индивидуальный спрос (d) Спрос, обеспеченный покупательной способностью
Ответ. (d)
Вопрос. Каждому члену МВФ назначается квота, выраженная в: (a) валюте страны-члена (b) долларах (c) специальных правах заимствования (d) ничего из вышеперечисленного
Ans. (c)
Связано: Викторина «Мировая столица»
Вопросы. Рыночная стоимость всех конечных товаров и услуг, произведенных и/или произведенных в географических границах страны за год, известна как __ (a) Валовой внутренний продукт (b) Валовые национальные сбережения (c) Валовой дефицит бюджета (d) Валовое накопление внутреннего капитала
Отв. (b)
Вопрос. Промышленная революция впервые началась в (а) Германии (б) Англии (в) Италии (г) Франции
Анс. (b)
Вопрос. Люди, занятые какой деятельностью, в основном сталкиваются с сезонной и скрытой безработицей? (a) Сельское хозяйство (b) Производство (c) Строительство (d) Услуги
Ответ. (а)
Вопрос. Что из следующего не является влиянием безработицы? (a) Растрата трудовых ресурсов (b) Увеличение экономической перегрузки (c) Препятствие общему росту экономики (d) Увеличение дохода на душу населения
Ответ. (d)
Вопрос. Что происходит, когда экономика попадает в тиски «рецессии»? 1. Рост ВВП замедляется. 2. Люди теряют работу. 3. Резерв иностранной валюты существенно увеличивается, так как начинает поступать много кредитов/грантов. (a) Только 1 (b) Только 2 (c) Только 3 (d) Только 1 и 2
Украшает, форматирует и сделает CSS код более читаемым.
Уменьшитель CSS
Сделает CSS код уменьшенным, сжатым путем удаления новых строк, пробелов, комментариев и отступов.
Украшатель HTML
Украшает, форматирует и сделает HTML код более читаемым.
Уменьшитель HTML
Сделает HTML код уменьшенным, сжатым путем удаления новых строк, пробелов, комментариев и отступов.
Украшатель Javascript
Украшает, форматирует и сделает Javascript код более читаемым.
Уменьшитель Javascript
Сделает Javascript код уменьшенным, сжатым путем удаления новых строк, пробелов, комментариев и отступов.
Обфускатор Javascript
Сделает Javascript код более сложным для понимания или чтения для защиты.
Украшатель JSON
Украшает, форматирует и сделает JSON код более читаемым.
Уменьшитель JSON
Сделает JSON код уменьшенным, сжатым путем удаления новых строк, пробелов, комментариев и отступов.
Украшатель XML
Украшает, форматирует и сделает XML код более читаемым.
Уменьшитель XML
Сделает XML код уменьшенным, сжатым путем удаления новых строк, пробелов, комментариев и отступов.
Украшатель OPML
Украшает, форматирует и сделает OPML код более читаемым.
Уменьшитель OPML
Сделает OPML код уменьшенным, сжатым путем удаления новых строк, пробелов, комментариев и отступов.
Украшатель SQL
Украшает, форматирует и сделает SQL код более читаемым.
Уменьшитель SQL
Сделает SQL код уменьшенным, сжатым путем удаления новых строк, пробелов, комментариев и отступов.
Конвертеры
Конвертер CSV в JSON
Конвертирует CSV данные в JSON и украшает.
Конвертер CSV в TSV
Конвертирует CSV данные в TSV и украшает.
Конвертер CSV в Excel
Конвертирует CSV данные в Excel и украшает.
Конвертер CSV в HTML
Конвертирует CSV данные в HTML, просматривая его ниже.
Конвертер CSV в SQL
Конвертирует CSV в SQL формат и украшает.
Конвертер CSV в Многострочные данные
Конвертирует CSV в многострочные данные и делает его более читаемым.
Конвертер CSV в Текст
Конвертирует CSV в обычный текст и делает его более читаемым.
Конвертер CSV в XML/JSON
Конвертирует CSV в XML и JSON онлайн.
Конвертер CSV в XML
Конвертирует CSV в XML и украшает.
Конвертер CSV в YAML
Конвертирует CSV в YAML и украшает.
Извлечь столбец CSV
Извлекает один столбец из CSV.
Конвертер Excel в CSV
Конвертирует Excel в CSV и украшает.
Конвертер Excel в TSV
Конвертирует Excel в TSV и украшает.
Конвертер Excel в HTML
Конвертирует Excel в HTML и украшает.
Excel в формульный вид
Конвертирует Excel в формульный вид и украшает.
Конвертер Excel в SQL
Конвертирует Excel в SQL и украшает.
Конвертер Excel в JSON
Конвертирует Excel в JSON и украшает.
Конвертер Excel в XML
Конвертирует Excel в XML и украшает.
Конвертер Excel в YAML
Конвертирует Excel в YAML и украшает.
Конвертер Excel в Текст
Конвертирует Excel в Текст и украшает.
Извлечь столбец Excel
Извлекает один столбец из Excel.
Конвертер TSV в JSON
Конвертирует данные TSV в JSON и украшает.
Конвертер TSV в CSV
Конвертирует данные TSV в CSV и украшает.
Конвертер TSV в Excel
Конвертирует данные TSV в Excel и украшает.
Конвертер TSV в HTML
Конвертирует данные TSV в HTML, с просмотром ниже.
Конвертер TSV в SQL
Конвертирует TSV в SQL формат и украшает.
Конвертер TSV в Многострочные данные
Конвертирует TSV в многострочные данные и делает более читаемым.
Конвертер TSV в Текст
Конвертирует TSV в обычный текст и делает более читаемым.
Конвертер TSV в XML/JSON
Конвертирует TSV в XML/JSON и украшает.
Конвертер TSV в XML
Конвертирует TSV в XML и украшает.
Конвертер TSV в YAML
Конвертирует TSV в YAML и украшает.
Извлечь столбец TSV
Извлекает один столбец из TSV.
Конвертер HTML в CSV
Конвертирует HTML в CSV и украшает.
Конвертер HTML в EXCEL
Конвертирует HTML в EXCEL и украшает.
Конвертер HTML в TSV
Конвертирует HTML в TSV и украшает.
Конвертер HTML в Многострочные данные
Конвертирует HTML в Многострочные данные и украшает.
Конвертер HTML в JSON
Конвертирует HTML в JSON и украшает.
Конвертер HTML в XML
Конвертирует HTML в XML и украшает.
Конвертер HTML в YAML
Конвертирует HTML в TAML и украшает.
Конвертер HTML в SQL
Конвертирует HTML в SQL и украшает.
Конвертер HTML в PHP
Конвертирует HTML в PHP и украшает.
Конвертер HTML в Javascript
Конвертирует HTML в Javascript и украшает.
Конвертер HTML в Asp
Конвертирует HTML в Asp и украшает.
Конвертер HTML в JSP
Конвертирует HTML в JSP и украшает.
Конвертер HTML в Perl
Конвертирует HTML в Perl и украшает.
Конвертер HTML в Jade
Конвертирует HTML в Jade и украшает.
Конвертер HTML в Текст
Конвертирует HTML в обычный текст.
Конвертер Jade в HTML
Конвертирует Jade в HTML и украшает.
Конвертер Markdown в HTML
Конвертирует Markdown в HTML код.
Конвертер JSON в XML
Конвертирует JSON в XML и украшает.
Конвертер JSON в CSV
Конвертирует JSON в CSV и украшает.
Конвертер JSON в Excel
Конвертирует JSON в Excel и украшает.
Конвертер JSON в TSV
Конвертирует JSON to TSV и украшает.
Конвертер JSON в YAML
Конвертирует JSON в YAML и украшает.
Конвертер JSON в HTML
Конвертирует JSON в HTML и украшает.
Конвертер JSON в SQL
Конвертирует JSON в SQL и украшает.
Конвертер JSON в C# класс
Конвертирует JSON в C# класс и украшает.
Конвертер JSON в Текст
Конвертирует JSON в Текст и украшает.
Конвертер SQL в HTML
Конвертирует SQL в HTML и украшает.
Конвертер SQL в CSV
Конвертирует SQL в CSV и украшает.
Конвертер SQL в Excel
Конвертирует SQL в Excel и украшает.
Конвертер SQL в TSV
Конвертирует SQL в TSV и украшает.
Конвертер SQL в XML
Конвертирует SQL в XML и украшает.
Конвертер SQL в JSON
Конвертирует SQL в JSON и украшает.
Конвертер SQL в YAML
Конвертирует SQL в YAML и украшает.
Конвертер SQL в Text
Конвертирует SQL в Text и украшает.
Конвертер XML в JSON
Конвертирует XML в JSON и украшает.
Конвертер XML в CSV
Конвертирует XML в CSV и украшает.
Конвертер XML в Excel
Конвертирует XML в Excel и украшает.
Конвертер XML в TSV
Конвертирует XML в TSV и украшает.
Конвертер XML в YAML
Конвертирует XML в YAML и украшает.
Конвертер XML в HTML
Конвертирует XML в HTML и украшает.
Конвертер XML в SQL
Конвертирует XML в SQL и украшает.
Конвертер XML в Текст
Конвертирует XML в Текст и украшает.
Конвертер YAML в XML/JSON/CSV
Конвертирует YAML в JSON/CSV/XML и украшает.
Конвертер YAML в Excel
Конвертирует YAML в Excel и украшает.
Конвертер YAML в HTML
Конвертирует YAML в HTML и украшает.
Конвертер XML в PDF
Конвертировать XML в PDF и Скачать.
Конвертер CSV в PDF
Конвертировать CSV в PDF и Скачать.
Конвертер TSV в PDF
Конвертировать TSV в PDF и Скачать.
Конвертер EXCEL в PDF
Конвертировать EXCEL в PDF и Скачать.
Конвертер JSON в PDF
Конвертировать JSON в PDF и Скачать.
Конвертер YAML в PDF
Конвертировать YAML в PDF и Скачать.
Конвертер SQL в PDF
Конвертировать SQL в PDF и Скачать.
Конвертер Текст в PDF
Конвертировать Текст в PDF и Скачать.
Конвертер PDF в JPG
Конвертировать PDF в JPG и Скачать.
Конвертер PDF в PNG
Конвертирует PDF в PNG и украшает.
Конвертер Текст в HTML
Конвертирует Текст в HTML и украшает.
Конвертер RSS в JSON
Конвертирует RSS в JSON и украшает.
Конвертер OPML в JSON
Конвертирует OPML в JSON и украшает.
Инструменты проверки валидности кода
Валидатор CSS
Проверьте ваш исходник CSS.
Валидатор Javascript
Проверьте ваш исходник Javascript.
Тестер Javascript
Проверьте ваш Javascript код.
Тестер HTML
Проверьте ваш HTML код.
Валидатор JSON
Проверьте ваш JSON код и украсьте.
Валидатор XML
Проверьте ваш XML код и украсьте.
Валидатор YAML
Проверьте ваш YAML код и украсьте.
Валидатор UUID
Проверьте ваш UUID код.
Тестер XPath
Онлайн Xpath тестер.
Тестер и генератор регулярных выражений
Проверка регулярного выражения и создание кода.
Препроцессоры CSS
Компилятор LESS
Создает отформатированные стили CSS из меньшего источника.
Компилятор Stylus
Создает украшенные стили CSS из Stylus
Конвертер CSS в LESS
Конвертирует CSS в Less и украшает.
Конвертер CSS в SCSS
Конвертирует CSS в SCSS и украшает.
Конвертер CSS в SASS
Конвертирует CSS в SASS и украшает.
Другие утилиты
Генераторы
Генератор случайных паролей
Генератор Favicon
Безопасный каталог htaccess
Генератор htpasswd
Генератор Lorem Ipsum
Генератор адресов IPv4
Генератор адресов IPv6
Генератор MAC адресов
Генератор календарных дат
Конвертеры величин
Конвертер веса
Конвертер площади
Конвертер плотности и массы
Конвертер байтов/битов
Конвертер электроэнергии
Конвертер энергии
Конвертер силы
Конвертер Топлива
Конвертер длины
Конвертер объема и емкости
Конвертер температуры
Конвертер скорости и ускорения
Конвертер угла
Конвертер массы
Конвертер мощности
Конвертер давления и напряжения
Конвертер времени
Астрономический конвертер
Конвертер частоты
Утилиты
Информация о браузере
Конвертер Base64 в Изображение
Конвертер Изображение в Base64
Конвертер Файла в Base64
Генератор символов
Конвертер текста в HTML объекты
Парсер URL
Автообновление страницы
Экранирование и разэкранирование
Экранирование и разэкранирование JSON
Экранирование и разэкранирование C#
Экранирование и разэкранирование Javascript
Экранирование и разэкранирование Java
Экранирование и разэкранирование CSV
Экранирование и разэкранирование SQL
Экранирование и разэкранирование HTML
Экранирование и разэкранирование XML
Шифрование
Генератор HMAC
Хэш калькулятор
Стеганография изображений
Стеганографический декодер
Генератор паролей MySQL/MariaDB
Генератор паролей Postgres
Строчные утилиты
Конвертер базового номера
Кодер/Декодер Base64
Средство просмотра различий
Кодировщик Url
Декодер Url
Кодировщик Html
Декодер Html
Добавить слэш
Убрать слеш
Конвертер числа в слово
Утилиты строк
Трансформер текста
Конвертер регистра
Калькулятор даты
Конвертер Даты/Времени в временную метку Unix
Конвертер временную метку Unix в время Дата/Время
Конвертер Секунд в человеческое время
Конвертер Секунд в Часы:Минуты:Секунды
Конверторы изображений
Конвертер JPG в PNG
Конвертер PNG в JPG
Конвертер GIF в PNG
Конвертер PNG в GIF
Конвертер BMP в PNG
Конвертер BMP в JPG
Генератор изображений с закругленными углами
Инструменты домена и IP
Получить IP и имя хоста
Просмотр имени хоста
Whois сервис
Просмотр DNS
Просмотр MX
Просмотр сервера имён
Проверка IP сайта
IP утилиты
Мой IP адрес
Редакторы кода
Просмотр исходного кода
Онлайн Редактор Кода
Пример кода
Конвертеры цвета
Конвертер RGB в HEX
Конвертер RGB в CMYK
Конвертер RGB в HSV
Конвертер HEX в HSV
Конвертер HEX в CMYK
Конвертер HSV в CMYK
Convert PDF to Excel Free Online
Convert PDF to Excel Free Online — No email required
PremiumDesktop
English
Espanol
Français
Italiano
Ellinika
Nederlands
Bahasa Indonesia
Deutsch
Türkçe
日本語
简体中文
العربية
Bahasa Malaysia
Português
Polski
Pусский
Upload
Choose from Dropbox
Choose from Google Drive
Open from OneDrive
Rank this page:
Upload the document, the conversion will start automatically.
No need to leave an email or other personal information.
All files are deleted from our servers after 6h max (without a trace).
Safe and Secure
All data uploaded to our servers will be used for conversion purposes only and deleted promptly after conversion. Your files are accessed only by automated software agents.
Full Document Conversion
The web app converts entire PDFs to Microsoft Excel. If your PDF file has multiple pages, rest assured that the full document will be converted to Excel free.
Exact Spreadsheet Recognition
All tables trapped in PDF will be extracted accurately. The rows and columns structure will be the same as in the original file, but fully editable and ready for reuse.
Excel’s .xlsx Format
The new spreadsheet will be in .xlsx format ready to use in Microsoft Excel, LibreCalc, OpenCalc or any other office suite compatible with MS Office.
No Email
There is no need to leave an email address or other identifiable data. Upload your file and the document will be converted right on the page.
Extract Scanned Tables
Besides regular PDF files, the service works with scanned PDFs as well. Using the latest OCR technology, it accurately recognizes numbers and data.
Quick and Easy
Conversion speed depends on the file size and servers’ overload.
Drive, Dropbox and OneDrive
Import PDFs from your Google Drive, Dropbox or OneDrive account. Log in, choose a file and start converting.
Free
This service is free to use, without any file conversion limits. Try it out right now.
How to convert PDF to Excel Free Online
Upload or drag and drop any PDF (regular or scanned) to PDFtoExcel.com. Alternatively, you can import the PDF for conversion directly from Google Drive, Dropbox or OneDrive.
The conversion starts automatically as soon as the file has been uploaded.
Wait for PDFtoExcel.com to finish conversion.
Download your .XLSX spreadsheet by clicking on the Free Download button.
This website uses cookies to ensure you get the best possible experience. By continuing you agree to our Terms of Service and Privacy Policy
Close
Excel в CSV — конвертируйте XLS в CSV бесплатно онлайн
Конвертируйте XLS в CSV онлайн и бесплатно
Шаг 1. Выберите файлы для конвертации
Перетаскивание файлов Макс. размер файла 50MB (хотите больше?)
Как мои файлы защищены?
Шаг 2. Конвертируйте ваши файлы в
Конвертируйте в
Или выберите другой формат
Шаг 3. Начните конвертировать
(и примите наши Условия)
Электронная почта, когда закончите?
Вы пытаетесь загрузить файл, размер которого превышает наш свободный лимит в 50 МБ.
Вам нужно будет создать платную учетную запись Zamzar, чтобы иметь возможность скачать преобразованный файл. Хотите продолжить загрузку файла для конвертации?
* Ссылки должны иметь префикс http или https , например. http://48ers.com/magnacarta.pdf
Частные лица и компании доверяют Zamzar с 2006 года. Мы обеспечиваем безопасность ваших файлов и данных и предлагаем выбор и контроль над удалением файлов.
Свободно конвертированные файлы надежно хранятся не более 24 часов
Файлы платных пользователей хранятся до тех пор, пока они не решат их удалить
Все пользователи могут удалять файлы до истечения срока их действия
Попробовала и сразу влюбилась! Это было так легко использовать! После пары преобразований я купил ребятам чашку кофе. Еще пара и решил, что это слишком хорошо, чтобы злоупотреблять! Я присоеденился! Моя жизнь намного проще!
Тилли
У меня был огромный проблемный файл для преобразования, который не мог пройти обычный процесс автоматического преобразования. Команда Zamzar быстро отреагировала на мою просьбу о помощи и предприняла дополнительные шаги, необходимые для того, чтобы сделать это вручную.
ПДинСФ
Я использовал этот продукт в течение многих лет. И обслуживание клиентов отличное. Только что возникла проблема, когда мне предъявили обвинение, и я не согласился с обвинением, и они позаботились об этом, хотя в этом не было необходимости.
JH
Я был так благодарен Замзару за поддержку с начала пандемии до наших дней. Их обслуживание является первоклассным, и их готовность помочь всегда на высоте.
Мэри
Очень полезный и профессиональный сайт. Сервис прост в использовании, а администраторы услужливы и вежливы.
Дэвид Шелтон
Я впервые им пользуюсь. У меня были некоторые сложности. Я не очень хорош в этом. Но я написал в компанию, и мне очень помогли. Я доволен обслуживанием клиентов и приложением.
Ана Суарес
Я использую Zamar всякий раз, когда мне нужно преобразовать аудио- и видеофайлы из нескольких отправителей в единый формат файла для редактирования аудио и видео. Я могу сделать несколько больших файлов за короткий промежуток времени.
Кристофер Би
Большое спасибо всем вам за помощь в правильном преобразовании СТАРЫХ файлов. 20 лет, довольно долгий срок, просмотр файлов навевает мне много воспоминаний. Это лучший подарок, который я получил в прошлом году. Спасибо всем еще раз.
Цзюнн-Ру Лай
Я чувствую, что Замзар — активный член команды, особенно в проектах, над которыми я работаю, где я являюсь рабочей лошадкой, и это экономит так много времени и нервов. Я избалован Zamzar, потому что они установили очень высокую планку для преобразования файлов и обслуживания клиентов.
Дебора Герман
Фантастический сервис! Компьютер моей мамы умер, и у нее есть более 1000 файлов Word Perfect, которые она по какой-то причине хочет сохранить. Поскольку Word Perfect практически мертв, я решил конвертировать все ее файлы. Преобразователь Замзара был идеальным.
Арон Бойетт
Нам доверяют сотрудники этих брендов
Сотрудники некоторых из самых известных мировых брендов полагаются на Zamzar для безопасного и эффективного преобразования своих файлов, гарантируя, что у них есть форматы, необходимые для работы. Сотрудники этих организаций, от глобальных корпораций и медиа-компаний до уважаемых учебных заведений и газетных изданий, доверяют Zamzar предоставление точных и надежных услуг по конвертации, в которых они нуждаются.
Ваши файлы в надежных руках
От вашего личного рабочего стола до ваших бизнес-файлов, мы обеспечим вас
Мы предлагаем ряд инструментов, которые помогут вам конвертировать ваши файлы наиболее удобным для вас способом. Помимо нашей онлайн-службы преобразования файлов, мы также предлагаем настольное приложение для преобразования файлов прямо с вашего рабочего стола и API для автоматического преобразования файлов для разработчиков. Какой инструмент вы используете, зависит от вас!
Хотите конвертировать файлы прямо с рабочего стола?
Получить приложение
Полностью интегрирован в ваш рабочий стол
Преобразование более 150 различных форматов файлов
Конвертируйте документы, видео, аудио файлы в один клик
Нужна функциональность преобразования в вашем приложении?
Изучите API
Один простой API для преобразования файлов
100 форматов на ваш выбор
Документы, видео, аудио, изображения и многое другое. ..
Почему выбирают Замзар?
С Zamzar конвертация файлов проста, надежна и удобна, поэтому вы можете быстро конвертировать документы, изображения, видео и многое другое в нужные вам форматы. Благодаря более быстрой загрузке преобразованных файлов и дружелюбной и полезной поддержке, когда вам это нужно, у вас будет все необходимое для работы с вашими файлами.
Quick
Мы всегда стараемся улучшить скорость преобразования файлов. Вот почему мы недавно добавили преобразование в реальном времени, что означает, что вам больше не нужно вводить адрес электронной почты, чтобы получить преобразованный файл.
Программное обеспечение не требуется
Zamzar поддерживает более 1100 типов преобразования файлов, что означает, что вам больше не потребуется загружать какое-либо программное обеспечение для преобразования файла.
Доступен 24*7
Вы можете конвертировать файлы в любое время, так как сайт доступен каждый день в году.
Пользовательский опыт
Мы всегда стремимся сделать наш сервис лучше, поэтому мы обновили нашу домашнюю страницу и добавили больше типов конверсий.
Инструменты для преобразования ваших файлов
В Zamzar вы найдете все необходимые инструменты для преобразования и сжатия в одном месте. С поддержкой более 1100 типов преобразования файлов, независимо от того, нужно ли вам конвертировать видео, аудио, документы или изображения, вы легко найдете то, что вам нужно, и вскоре ваши файлы будут в форматах и размерах, которые вам подходят.
Формат документа XLS
XLS-конвертер
Формат файла XLS был разработан Microsoft для своей программы электронных таблиц Excel. XLS — это двоичный формат электронной таблицы, который может включать данные ячеек и формулы, а также диаграммы и макросы. Excel — одна из самых популярных программ для записи, организации, расчета и анализа данных. Он часто используется для составления бюджета и отчетности.
Microsoft представила новый формат XLSX для Excel в 2007 году, чтобы заменить XLS. Однако вы по-прежнему можете открывать файлы XLS в более новых версиях Excel, а также сохранять файлы XLSX в этом более старом формате. Excel является частью пакета Microsoft Office, который доступен как разовая покупка программного обеспечения или подписка на Office 365. Другие программы для работы с электронными таблицами, которые обычно могут открывать файлы XLS, включают Google Sheets, Apple Numbers и LibreOffice Calc.
Связанные инструменты
Конвертеры документов
XLS-конвертер
Формат документа CSV
CSV-конвертер
CSV — это тип файла для сохранения данных электронной таблицы, который распознается почти всеми программами для работы с электронными таблицами. CSV означает «значения, разделенные запятыми», и если вы откроете CSV-файл вне программы для работы с электронными таблицами, вы увидите обычный текст с данными, разделенными, как правило, запятыми, точкой с запятой или кавычками. Если вы откроете тот же CSV-файл в программе для работы с электронными таблицами, такой как Microsoft Excel, Apple Numbers, Google Sheets или LibreOffice Calc, каждый отдельный фрагмент данных должен быть перенесен в отдельную ячейку.
Поскольку файлы CSV распознаются почти всеми программами для работы с электронными таблицами, они используются как простой способ экспорта данных или их передачи между различными программами. Однако CSV — это обычный текстовый формат, поэтому файлы CSV не могут включать расширенные функции электронных таблиц, такие как диаграммы, формулы, фильтры или форматирование. Также в настоящее время не существует универсального открытого стандарта для всех файлов CSV.
Связанные инструменты
Конвертеры документов
CSV-конвертер
Как преобразовать XLS в файл CSV?
1. Выберите файл XLS, который вы хотите преобразовать.
2. Выберите CSV в качестве формата, в который вы хотите преобразовать файл XLS.
3. Нажмите «Преобразовать», чтобы преобразовать файл XLS.
Преобразование из XLS
Используя Zamzar, можно конвертировать файлы XLS во множество других форматов:
XLS в BMP
XLS в CSV
XLS в GIF
XLS в HTML
XLS в HTML4
XLS в HTML5
XLS в JPG
XLS в MDB
XLS в НОМЕРА
XLS в NUMBERS09XLS в ODS
XLS в PDF
XLS в PNG
XLS в RTF
XLS в TIFF
XLS в TXT
XLS в XLSX
XLS в XML
Преобразовать в XLS
Используя Zamzar, можно конвертировать множество других форматов в файлы XLS:
CSV в XLS
MPP в XLS
НОМЕРА в XLS
НОМЕРА. ZIP в XLS
ODS в XLS
PDF в XLS
WKS в XLS
XLR в XLS
От XLSX до XLS
Часто задаваемые вопросы
Если у вас есть какие-либо вопросы о преобразовании или сжатии файлов с помощью Zamzar, мы будем рады помочь! Ниже мы ответили на несколько часто задаваемых вопросов, чтобы вы могли начать работу, а дополнительную информацию о преобразовании и сжатии файлов с помощью Zamzar вы можете найти в нашем Справочном центре.
Книгу легко преобразовать в файл CSV с помощью онлайн-инструментов преобразования, таких как Zamzar, или открыв файл XLS с помощью Excel и сохранив его как «CSV». Чтобы открыть Excel, вам потребуется программное обеспечение на вашем компьютере или онлайн-доступ к Microsoft Office 365. Если у вас есть учетная запись Google, вы также можете импортировать файл XLS в Google Таблицы, а затем преобразовать его. Если вам нужно преобразовать несколько файлов, надежный инструмент преобразования, такой как Zamzar, сэкономит ваше время по сравнению с преобразованием вручную с помощью Excel или Google Sheets.
Используя онлайн-инструмент Zamzar, можно легко преобразовать все рабочие листы XLS в документе в формат CSV [SD1] за один раз без потери данных. Проблема с использованием функции «Сохранить как» в Excel заключается в том, что иногда вы можете потерять данные, вы можете преобразовать только тот лист, который вы открыли, и он также может не передавать специальные символы или другие данные.
На веб-сайте Zamzar есть трехэтапный инструмент преобразования: вы просто загружаете один или несколько файлов XLS, нажимаете, чтобы преобразовать их, и загружаете файлы CSV без потери данных.[SD2]
Хотя вы можете использовать функцию «Сохранить как» в Excel для создания файла CSV, у вас может не быть доступа к Microsoft Office для открытия исходных файлов Excel. С Zamzar вам не нужно открывать файл Excel, чтобы преобразовать его в CSV. Вы можете конвертировать несколько файлов XLS в CSV, не открывая их — вам даже не нужен доступ к Microsoft Office. Всего три шага: загрузите электронную таблицу XLS в бесплатный инструмент преобразования Zamzar, нажмите, чтобы преобразовать ее, а затем загрузите новый файл CSV.
Онлайн-инструмент Zamzar позволяет быстро конвертировать несколько листов Excel в CSV без потери данных. Это избавляет вас от ручного открытия и преобразования каждого документа и рабочего листа в CSV с помощью Microsoft Office или Google Sheets. На веб-сайте Zamzar вы можете загрузить несколько файлов Excel, щелкнуть, чтобы преобразовать их, а затем загрузить файлы CSV в режиме реального времени.
Zamzar — это действительно быстрый способ преобразования Excel в CSV. Это онлайн-инструмент трехэтапного преобразования. Просто загрузите XLS-файл, выберите CSV в качестве формата для преобразования и нажмите «Преобразовать сейчас» — вы получите новый CSV-файл за считанные секунды без потери каких-либо данных или загрузки какого-либо программного обеспечения. Кроме того, вы можете использовать функцию «Сохранить как» в Excel или преобразовать файл с помощью Google Таблиц, но это может занять много времени, если у вас есть более одного файла для преобразования.
Преобразование EXCEL в CSV онлайн бесплатно
редактор
Зритель
Преобразование
Слияние
Разблокировать
Защищать
Сплиттер
Сравнение
Аннотация
Парсер
Метаданные
Водяной знак
Поиск
Заменять
Повернуть
Обеспечить регресс
Диаграмма
Ипотека
Сборка
Перевод
Компресс
Прозрачный
ИМТ
ВебКонвертер
Питаться от
aspose. com
&
aspose.cloud
Перетащите или загрузите свои файлы
Введите адрес
*Загружая свои файлы или используя наш сервис, вы соглашаетесь с нашими
Условия использования
&
политика конфиденциальности
Вычислить определитель матрицы системы методом гаусса онлайн. Вычисление определителя
Здесь вы сможете бесплатно решить систему линейных уравнений методом Гаусса онлайн больших размеров в комплексных числах с очень подробным решением. Наш калькулятор умеет решать онлайн как обычную определенную, так и неопределенную систему линейных уравнений методом Гаусса, которая имеет бесконечное множество решений. В этом случае в ответе вы получите зависимость одних переменных через другие, свободные. Также можно проверить систему уравнений на совместность онлайн, используя решение методом Гаусса.
При решении системы линейных уравнений онлайн методом Гаусса выполняются следующие шаги.
Записываем расширенную матрицу.
Фактически решение разделяют на прямой и обратный ход метода Гаусса. Прямым ходом метода Гаусса называется приведение матрицы к ступенчатому виду. Обратным ходом метода Гаусса называется приведение матрицы к специальному ступенчатому виду. Но на практике удобнее сразу занулять то, что находится и сверху и снизу рассматриваемого элемента. Наш калькулятор использует именно этот подход.
Важно отметить, что при решении методом Гаусса, наличие в матрице хотя бы одной нулевой строки с НЕнулевой правой частью (столбец свободных членов) говорит о несовместности системы. Решение линейной системы в таком случае не существует.
Чтобы лучше всего понять принцип работы алгоритма Гаусса онлайн введите любой пример, выберите «очень подробное решение» и посмотрите его решение онлайн.
В ходе решения задач по высшей математике очень часто возникает необходимость вычислить определитель матрицы . Определитель матрицы фигурирует в линейной алгебре, аналитической геометрии, математическом анализе и других разделах высшей математики. Таким образом, без навыка решения определителей просто не обойтись. Также для самопроверки Вы можете бесплатно скачать калькулятор определителей , он сам по себе не научит решать определители, но очень удобен, поскольку всегда выгодно заранее знать правильный ответ!
Я не буду давать строгое математическое определение определителя, и, вообще, буду стараться минимизировать математическую терминологию, большинству читателей легче от этого не станет. Задача данной статьи – научить Вас решать определители второго, третьего и четвертого порядка. Весь материал изложен в простой и доступной форме, и даже полный (пустой) чайник в высшей математике после внимательного изучения материала сможет правильно решать определители.
На практике чаще всего можно встретить определитель второго порядка, например: , и определитель третьего порядка, например: .
Определитель четвертого порядка тоже не антиквариат, и к нему мы подойдём в конце урока.
Надеюсь, всем понятно следующее: Числа внутри определителя живут сами по себе, и ни о каком вычитании речи не идет! Менять местами числа нельзя!
(Как частность, можно осуществлять парные перестановки строк или столбцов определителя со сменой его знака, но часто в этом нет никакой необходимости – см. следующий урок Свойства определителя и понижение его порядка)
Таким образом, если дан какой-либо определитель, то ничего внутри него не трогаем!
Обозначения : Если дана матрица , то ее определитель обозначают . Также очень часто определитель обозначают латинской буквой или греческой .
1) Что значит решить (найти, раскрыть) определитель? Вычислить определитель – это значит НАЙТИ ЧИСЛО. Знаки вопроса в вышерассмотренных примерах – это совершенно обыкновенные числа.
2) Теперь осталось разобраться в том, КАК найти это число? Для этого нужно применить определенные правила, формулы и алгоритмы, о чём сейчас и пойдет речь.
Начнем с определителя «два» на «два» :
ЭТО НУЖНО ЗАПОМНИТЬ, по крайне мере на время изучения высшей математики в ВУЗе.
Сразу рассмотрим пример:
Готово. Самое главное, НЕ ЗАПУТАТЬСЯ В ЗНАКАХ.
Определитель матрицы «три на три» можно раскрыть 8 способами, 2 из них простые и 6 — нормальные.
Начнем с двух простых способов
Аналогично определителю «два на два», определитель «три на три» можно раскрыть с помощью формулы:
Формула длинная и допустить ошибку по невнимательности проще простого. Как избежать досадных промахов? Для этого придуман второй способ вычисления определителя, который фактически совпадает с первым. Называется он способом Саррюса или способом «параллельных полосок». Суть состоит в том, что справа от определителя приписывают первый и второй столбец и аккуратно карандашом проводят линии:
Множители, находящиеся на «красных» диагоналях входят в формулу со знаком «плюс». Множители, находящиеся на «синих» диагоналях входят в формулу со знаком минус:
Пример:
Сравните два решения. Нетрудно заметить, что это ОДНО И ТО ЖЕ, просто во втором случае немного переставлены множители формулы, и, самое главное, вероятность допустить ошибку значительно меньше.
Теперь рассмотрим шесть нормальных способов для вычисления определителя
Почему нормальных? Потому что в подавляющем большинстве случаев определители требуется раскрывать именно так.
Как Вы заметили, у определителя «три на три» три столбца и три строки. Решить определитель можно, раскрыв его по любой строке или по любому столбцу . Таким образом, получается 6 способов, при этом во всех случаях используется однотипный алгоритм.
Определитель матрицы равен сумме произведений элементов строки (столбца) на соответствующие алгебраические дополнения. Страшно? Все намного проще, будем использовать ненаучный, но понятный подход, доступный даже для человека, далекого от математики.
В следующем примере будем раскрывать определитель по первой строке . Для этого нам понадобится матрица знаков: . Легко заметить, что знаки расположены в шахматном порядке.
Внимание! Матрица знаков – это мое собственное изобретение. Данное понятие не научное, его не нужно использовать в чистовом оформлении заданий, оно лишь помогает Вам понять алгоритм вычисления определителя.
Сначала я приведу полное решение. Снова берем наш подопытный определитель и проводим вычисления:
И главный вопрос: КАК из определителя «три на три» получить вот это вот: ?
Итак, определитель «три на три» сводится к решению трёх маленьких определителей, или как их еще называют, МИНОРОВ . Термин рекомендую запомнить, тем более, он запоминающийся: минор – маленький.
Коль скоро выбран способ разложения определителя по первой строке , очевидно, что всё вращается вокруг неё:
Элементы обычно рассматривают слева направо (или сверху вниз, если был бы выбран столбец)
Поехали, сначала разбираемся с первым элементом строки, то есть с единицей:
1) Из матрицы знаков выписываем соответствующий знак:
2) Затем записываем сам элемент:
3) МЫСЛЕННО вычеркиваем строку и столбец, в котором стоит первый элемент:
Оставшиеся четыре числа и образуют определитель «два на два», который называется МИНОРОМ данного элемента (единицы).
Переходим ко второму элементу строки.
4) Из матрицы знаков выписываем соответствующий знак:
5) Затем записываем второй элемент:
6) МЫСЛЕННО вычеркиваем строку и столбец, в котором стоит второй элемент:
Ну и третий элемент первой строки. Никакой оригинальности:
7) Из матрицы знаков выписываем соответствующий знак:
8) Записываем третий элемент:
9) МЫСЛЕННО вычеркиваем строку и столбец, в котором стоит третий элемент:
Оставшиеся четыре числа записываем в маленький определитель.
Остальные действия не представляют трудностей, поскольку определители «два на два» мы считать уже умеем. НЕ ПУТАЕМСЯ В ЗНАКАХ!
Аналогично определитель можно разложить по любой строке или по любому столбцу. Естественно, во всех шести случаях ответ получается одинаковым.
Определитель «четыре на четыре» можно вычислить, используя этот же алгоритм. При этом матрица знаков у нас увеличится:
В следующем примере я раскрыл определитель по четвертому столбцу :
А как это получилось, попробуйте разобраться самостоятельно. Дополнительная информация будет позже. Если кто захочет прорешать определитель до конца, правильный ответ: 18. Для тренировки лучше раскрыть определитель по какому-нибудь другому столбцу или другой строке.
Потренироваться, раскрыть, провести расчёты – это очень хорошо и полезно. Но сколько времени вы потратите на большой определитель? Нельзя ли как-нибудь быстрее и надёжнее? Предлагаю ознакомиться с эффективными методами вычисления определителей на втором уроке – Свойства определителя. Понижение порядка определителя .
2.2 Метод Гаусса для решения систем линейных уравнений…………………… 6
2.3 Метод Гаусса для вычисления определителя……………………………………. 8
3. Функциональные модели и блок-схемы решения задачи…………………….. 9
4. Программная реализация решения задачи………………………………………. 11
5. Пример выполнения программы…………………………………………………….. 16
Заключение……. ………………………………………………………………………………… 18
Список использованных источников и литературы……………………………… 19
Введение
Многие проблемы, возникающие в экономических исследованиях, планировании и управлении, будучи сформулированными математически, представляют собой задачи, в которых необходимо решить систему алгебраических уравнений.
Исторически первым, наиболее распространенным методом решения систем линейных уравнений является метод Гаусса, или метод последовательного исключения неизвестных. Сущность этого метода состоит в том, что посредством последовательных исключений неизвестных данная система превращается в ступенчатую (в частности, треугольную) систему, равносильную данной.
При практическом решении системы линейных уравнений методом Гаусса удобнее приводить к ступенчатому виду не саму систему уравнений, а расширенную матрицу этой системы, выполняя элементарные преобразования над ее строками. Последовательно получающиеся в ходе преобразования матрицы обычно соединяют знаком эквивалентности. Этот метод (который также называют методом последовательного исключения неизвестных) известен в различных вариантах уже более 2000 лет.
Помимо аналитического решения СЛАУ, метод Гаусса также применяется для нахождения матрицы, обратной к данной, определения ранга матрицы и нахождения определителя.
Целью данной курсовой работы является реализация вычисления определителя методом исключения Гаусса.
1. Постановка задачи
Вычисление определителя матрицы заключается в выполнении над матрицей алгоритма Гаусса для решения систем линейных алгебраических уравнений. В результате выполнения алгоритма получаем диагональную матрицу, её определитель равен произведению элементов, стоящих на диагонали.
.
~.
.
.
Вычислить определитель матрицы методом A исключения Гаусса.
.
Приведем матрицу к диагональному виду методом Гаусса.
~.
Тогда определитель матрицы равен произведению ее элементов, стоящих на диагонали:
.
Знак определяется количеством обменов строк, следовательно определитель матрицы
.
2. Математические и алгоритмические основы решения задачи
2.1 Определитель матрицы
Введем определение определителя квадратной матрицы любого порядка. Это определение будет рекуррентным, то есть чтобы установить, что такое определитель матрицы порядка n, нужно уже знать, что такое определитель матрицы порядка n-1. Отметим также, что определитель существует только у квадратных матриц.
Определитель квадратной матрицы A будем обозначать
или det A.
Определение. Определителем квадратной матрицы
второго порядка называется число
.
Определителем
квадратной матрицы порядка n,
, называется число
— определитель матрицы порядка n-1, полученной из матрицы A вычеркиванием первой строки и столбца с номером k.
2.2 Метод Гаусса для решения систем линейных уравнений
Пусть дана квадратная матрица A размером NxN. Требуется вычислить её определитель.
Воспользуемся идеями метода Гаусса решения систем линейных уравнений.
Дана система:
a11 x1 + a12 x2 + … + a1n xn = b1
a21 x1 + a22 x2 + … + a2n xn = b2
an1 x1 + an2 x2 + … + ann xn = bn
Выполним следующий алгоритм.
На первом шаге найдём в первом столбце наибольший по модулю элемент, поставим уравнение с этим элементом на первую строчку (обменяв две соответствующие строки матрицы A и два соответствующих элемента вектора B), а затем будем отнимать это уравнение от всех остальных, чтобы в первом столбце все элементы (кроме первого) обратились в ноль. Например, при прибавлении ко второй строке будем домножать первую строку на -a21/a11, при добавлении к третьей — на -a31/a11, и т.д.
На втором шаге найдём во втором столбце, начиная со второго элемента, наибольший по модулю элемент, поставим уравнение с этим элементом на вторую строчку, и будем отнимать это уравнение от всех остальных (в том числе и от первого), чтобы во втором столбце все элементы (кроме второго) обратились в ноль. Понятно, что эта операция никак не изменит первый столбец — ведь от каждой строки мы будем отнимать вторую строку, домноженную на некоторый коэффициент, а во второй строке в первом столбце стоит ноль.
Т.е. на i-ом шаге найдём в i-ом столбце, начиная с i-го элемента, наибольший по модулю элемент, поставим уравнение с этим элементом на i-ю строчку, и будем отнимать это уравнение от всех остальных. Понятно, что это никак не повлияет на все предыдущие столбцы (с первого по (i-1)-ый).
В конце концов, мы приведём систему к так называемому диагональному виду:
Т.е. мы нашли решение системы.
Замечание 1. На каждой итерации найдётся хотя бы один ненулевой элемент, иначе система бы имела нулевой определитель, что противоречит условию.
Замечание 2. Требование, что на каждом шаге мы выбираем наибольший по модулю элемент, очень важно в смысле численной устойчивости метода. Если выбирать произвольный ненулевой элемент, то это может привести к гигантской погрешности, когда получившееся решение будет отличаться в разы от правильного.
2.3 Метод Гаусса для вычисления определителя
Будем выполнять те же самые действия, что и при решении системы линейных уравнений, исключив только деление текущей строки на a[i][i] (точнее, само деление можно выполнять, но подразумевая, что число выносится за знак определителя). Тогда все операции, которые мы будем производить с матрицей, не будут изменять величину определителя матрицы, за исключением, быть может, знака (мы только обмениваем местами две строки, что меняет знак на противоположный, или прибавляем одну строку к другой, что не меняет величину определителя).
Но матрица, к которой мы приходим после выполнения алгоритма Гаусса, является диагональной, и определитель её равен произведению элементов, стоящих на диагонали. Знак, как уже говорилось, будет определяться количеством обменов строк (если их нечётное, то знак определителя следует изменить на противоположный). Таким образом, мы можем с помощью алгоритма Гаусса вычислять определитель матрицы за O(N3).
Осталось только заметить, что если в какой-то момент мы не найдём в текущем столбце ненулевого элемента, то алгоритм следует остановить и вернуть 0.
3. Функциональные модели и блок-схемы решения задачи
Блок-схема решения задачи представлена на рисунке 1.
Рисунок 1 – Блок-схема решения задачи для функции DETERMINATE
4 Программная реализация решения задачи
;ФУНКЦИЯ, ВЫЧИСЛЯЮЩАЯ ОПРЕДЕЛИТЕЛЬ
(DEFUN DETERMINANT (MATRIX SIZE)
;ОБЪЯВЛЕНИЕ ПЕРЕМЕННЫХ
;ОПРЕДЕЛИТЕЛЬ
(DECLARE (SPECIAL DET))
;ВСПОМОГАТЕЛЬНЫЕ МАССИВЫ И ПЕРЕМЕННЫЕ
(DECLARE (SPECIAL PAR))
(DECLARE (SPECIAL R))
(DECLARE (SPECIAL T_))
(DECLARE (SPECIAL I))
(DECLARE (SPECIAL II))
;*********************
(SETQ R (MAKE-ARRAY SIZE:ELEMENT-TYPE «FLOAT:INITIAL-ELEMENT 0))
((>= J (- SIZE 1)))
;ИСКЛЮЧАЕМ ДЕЛЕНИЕ НА 0
(IF (= (AREF MATRIX J J) 0)
(SETQ II (+ J 1))
;ИЩЕМ СТРОКУ В КОТОРОЙ J-Й ЭЛЕМЕНТ НЕ 0
((OR (/= (AREF MATRIX II J) 0) (= II (- SIZE 1))))
(SETQ II (+ II 1))
;ЕСЛИ НЕТ ТАКОЙ СТРОКИ ОПРЕДЕЛИТЕЛЬ РАВЕН 0
(IF (AND (= (AREF MATRIX II J) 0) (= II (- SIZE 1))) (SETQ T_ 0))
Вычислим определитель методом Гаусса.
Суть метода состоит в следующем:
определитель приводится к треугольному
виду с помощью элементарных преобразований,
и тогда он равен произведению элементов,
стоящих на главной диагонали.
Идея метода состоит в следующем: пусть
дан определитель третьего порядка
элементдолжен быть равен ,
для этого первую строку разделим на.
Получим определитель вида (2)
Обнулим элементы, стоящие в первом
столбце, кроме первого. Для этого из
второй строки вычтем первую, умноженную
на ,
далее из третьей строки вычтем первую,
умноженную на.
Получим определитель вида .
Обозначим его элементы буквой с, тогда
(3)
Теперь надо обнулить элемент
.
Элемент должен быть равен ,
для этого вторую строку разделим на .
Получим определитель вида .
.
Обозначим его элементы буквой t,
тогда
(4)
Вот мы привели определитель к треугольному
виду, теперь он равен .
Разберем теперь это на конкретном
примере.
Пример 4: Вычислить определительметодом Гаусса.
Решение: Поменяем местами первую и
третью строки (при замене двух столбцов
(строк) определитель меняет знак на
противоположный).
Получили
Из второй строки вычтем первую, умноженную
на 2, далее из третьей строки вычтем
первую, умноженную на 3. Получили
Получили —
§2.Матрицы Виды матриц
Определение 7: Если в матрицеmстрок иnстолбцов, то она
называетсяразмерностью mnи пишут .
Определение 8: Если ,
то матрица называется квадратной.
Определение 9: Матрица, состоящая
лишь из одной строки (столбца) называется
матрицей-строкой (столбцом).
Определение 10: Матрица, состоящая
из нулей, называется нулевой матрицей.
Определение 11: Диагональной матрицей
называется квадратная матрица, у которой
все элементы, не принадлежащие главной
диагонали равны нулю.
Определение 12: Единичной матрицей
называется диагональная матрица, у
которой все элементы, стоящие на главной
диагонали равны единице.
Определение 13: Треугольной называется
квадратная матрица, у которой элементы,
расположенные по одну сторону от главной
диагонали равны нулю.
Действиянад матрицами.
Определение 14: Две матрицы считаются
равными, если они имеют одинаковое число
строк и столбцов и равные соответствующие
элементы.
Пример 5:
Матрицы А и В равны, т.е.
Определение 15: Суммой (разностью)
матриц А и В называется такая матрица
С, у которой каждый элемент равен .
Пример 6: Найти матрицу ,
если
Решение:
Cвойства сложения
А+В=В+А(переместительное)
2 0 А+О=А, где О-нулевая матрица
3 0 А+(В+С)=(А+В)+С (дистрибутивное)
4 0 А+(-А)=О, где – А противоположная
матрица
(т.е. элементы имеют противоположные
знаки)
Определение 16: Произведением матрицы
А на число называется матрица, полученная из
данной умножением всех ее элементов на
число.
Пример 7:
Умножение матиц
Это действие распространяется на так
называемые согласованные матрицы.
Определение 17: Матрица А называетсясогласованной с матрицей В, если число
столбцов у матрицы А равно числу строк
у матрицы В.
Пример 8: и — согласованные
и — несогласованные
и несогласованные
Определение 18: Произведением двух
матриц А и В называется такая матрица
С, каждый элемент которой равен сумме
произведений элементовiстроки матрицы А на соответствующие
элементыj-го столбца
матрицы В.
Если матрица А имеет размерность ,
а матрица В ,
то .
Пример 9: Умножить матрицы
Echelon Transform
Матрица имеет форму эшелона строк (ссылка)
когда он удовлетворяет следующим условиям.
Первый ненулевой элемент в каждой строке, называемый ведущая запись , равна 1.
Каждая ведущая запись находится в столбце справа от
ведущая запись в предыдущей строке.
Строки со всеми нулевыми элементами, если таковые имеются, находятся ниже строк, имеющих
ненулевой элемент.
Матрица представлена в виде сокращенной ступенчатой формы строки (rref)
когда он удовлетворяет следующим условиям.
Матрица имеет эшелонированную форму (т. е. удовлетворяет условию
три условия, перечисленные выше).
Первая запись в каждой строке является единственной ненулевой записью в
его столбец.
Матрица в форме эшелона называется -ступенчатой матрицей .
Матрица A и матрица B являются примерами
ступенчатых матриц.
1
2
3
4
0
0
1
3
0
0
0
1
0
0
0
0
1
2
0
0
0
0
1
0
0
0
0
1
0
0
0
0
А
Б
Матрица A имеет форму эшелона строк, а матрица B находится в форме редуцированного эшелонированного ряда.
Как преобразовать матрицу в ее ступенчатую форму
Любую матрицу можно преобразовать в ступенчатую форму, используя серию
элементарные операции со строками. Вот как.
Поворот матрицы
Найдите опорную точку , первую ненулевую запись в
первый столбец матрицы.
Поменять местами строки, переместив основную строку в первую строку.
Умножить каждый элемент в сводной строке на величину, обратную
точка опоры, поэтому точка опоры равна 1.
Добавить несколько строк сводной таблицы в каждую из нижних строк,
поэтому каждый элемент в сводном столбце нижнего
строки равны 0,
Чтобы получить матрицу в виде эшелона строк, повторите поворот
Повторите процедуру, начиная с шага 1 выше, игнорируя предыдущий
сводные ряды.
Продолжайте, пока не закончатся опорные точки.
обработанный.
Чтобы получить матрицу , уменьшенную строковую форму эшелона, обработайте ненулевой
записи над каждым сводом.
Определите последнюю строку, имеющую опорную точку, равную 1, и пусть
это будет сводная строка.
Добавить несколько строк сводной таблицы в каждую из верхних строк,
пока каждый элемент над опорной точкой не будет равен 0,
.
Перемещаясь вверх по матрице, повторите этот процесс для каждой строки.
Преобразование матрицы в ее эшелонированные формы: пример
Чтобы проиллюстрировать процесс преобразования,
давайте трансформируем Матрицу A в рядной эшелон
форме и к форме редуцированного эшелонированного ряда.
0
1
2
1
2
1
2
7
8
⇒
1
2
1
0
1
2
2
7
8
⇒
1
2
1
0
1
2
0
3
6
А
А 1
А 2
⇒
1
2
1
0
1
2
0
0
0
⇒
1
0
-3
0
1
2
0
0
0
А №
А rref
Для преобразования матрицы A в ее ступенчатые формы мы реализовали
следующую серию элементарных операций над строками.
Мы нашли первую ненулевую запись в первом столбце
матрицы в строке 2; поэтому мы поменяли местами ряды 1 и 2, в результате чего
в матрице A 1 .
Работа с матрицей А 1 ,
мы умножили каждый элемент строки 1 на -2 и добавили результат к
Ряд 3. Получилось А 2 .
Работа с матрицей А 2 ,
мы умножили каждый элемент строки 2 на -3 и добавили результат к
Ряд 3. Получилось A ref . Заметить, что Номер ref находится в форме эшелонированной строки, потому что он
удовлетворяет следующим требованиям: (a) первая ненулевая запись каждого
строка равна 1, (b) первая ненулевая запись находится справа от первой
ненулевой элемент в предыдущей строке и (c) строки, полностью состоящие из
нули находятся внизу матрицы.
И, наконец, работа с матрицей A ref ,
мы умножили вторую строку на -2 и добавили ее к
первый ряд. Это произвело A rref .
Обратите внимание, что A rref в уменьшено эшелонированную форму строки, так как она удовлетворяет требованиям к
форма эшелона строки плюс каждая ведущая ненулевая запись является единственной ненулевой
запись в своей колонке.
Примечание: Эшелонная матрица строк, полученная в результате ряда
элементарные операции со строками не обязательно уникальны. Другой набор строк
операции могут привести к другой ступенчатой матрице строк. Однако,
уменьшенная -строчная эшелонированная матрица уникальна; каждая матрица имеет только один
уменьшенная ступенчатая матрица строк.
Проверьте свое понимание
Задача 1
Рассмотрим матрицу X , как показано ниже.
X =
0
1
1
2
0
5
Какая из следующих матриц является редуцированной эшелонированной формой строк
матрица Х ?
0
1
1
0
0
0
1
0
0
1
0
0
1
0
0
1
0
1
1
0
0
0
0
1
А
Б
С
Д
(А) Матрица А (В) Матрица В (С) Матрица С (D) Матрица D (E) Ничего из вышеперечисленного
Решение
Правильный ответ (В). Элементарные операции со строками, используемые для изменения
Показана матрица X в ее уменьшенной эшелонированной форме строк.
ниже.
0
1
1
2
0
5
⇒
1
2
0
1
0
5
⇒
1
2
0
1
0
0
⇒
1
0
0
1
0
0
Х
Х 1
Х 2
Х rref
Чтобы преобразовать X в его сокращенную эшелонированную форму, мы берем
следующие шаги:
Поменяйте местами ряды 1 и 2, получив X 1 .
В Х 1 ,
умножьте 2-й ряд на -5 и прибавьте его к 3-му ряду, получив Х 2 .
В Х 2 ,
умножьте 2-й ряд на -2 и прибавьте его к 1-му ряду, получив X rref .
Примечание: Матрица A не имеет сокращенной ступенчатой формы, потому что
ведущая запись в строке 2 находится слева от ведущей записи в
Ряд 3; он должен быть справа. Матрица С не находится в форме редуцированного эшелона строк, потому что столбец 2 имеет больше
чем одна ненулевая запись. И, наконец, матрица D не
в сокращенной эшелонированной форме строки, потому что следует строка 2 со всеми нулями
по строке с ненулевым элементом; все нулевые строки должны следовать за ненулевыми строками.
Последний урок
Следующий урок
RREF Калькулятор с шагами | Калькулятор формы эшелона с уменьшенным рядом
Калькулятор RREF с шагами | Калькулятор формы эшелона уменьшенного ряда
Калькулятор RREF с шагами
Размер матрицы: x
Показать шаги:
Знание того, как найти Сокращенную форму эшелона строк (RREF) матрицы, очень помогает в решении систем линейных уравнений. По этой причине мы даем вам в руки эти Калькулятор RREF с шагами , который позволяет быстро и легко привести матрицу к форме эшелонирования строк.
Чтобы использовать этот калькулятор, вы должны выполнить следующие простые шаги:
Введите размеры матрицы, которую вы хотите уменьшить.
Введите матрицу в предназначенные для этого поля.
Нажмите кнопку «Рассчитать RREF», при этом автоматически появится окно с подробным пошаговым решением.
Ниже вы найдете краткое изложение наиболее важных теоретических концепций, связанных с как сделать сокращенную форму эшелонирования строк .
Содержание
1 Калькулятор RREF с шагами
2 Что такое эшелонированная форма (RREF) матрицы?
3 Как найти сокращенную форму эшелона строк
4 Примеры и решения формы сокращенного эшелона строк
4.1 Пример 01: найти сокращенную форму эшелона строк этой матрицы 2×2
4.2 Пример 02: найти сокращенную форму эшелона строк этой матрицы 3×3
Что такое сокращенная ступенчатая форма (RREF) матрицы?
Сокращенная эшелонированная форма строк матрицы A — это другая матрица H, которая удовлетворяет следующим свойствам:
Она имеет строки, полностью состоящие из нулей (пустые строки), они сгруппированы в нижней части матрицы.
Опорная точка (первый ненулевой элемент) каждой непустой строки равна 1.
Опорная точка каждой ненулевой строки находится справа от предыдущей строки.
Все элементы, которые появляются в том же столбце, что и сводная строка строки, равны нулю.
Вот пример сокращенной эшелонированной формы строки матрицы:
Каждая матрица имеет одну эшелонированную форму строки, независимо от того, как вы выполняете операции над строками.
Важность матриц в сокращенной ступенчатой форме строк вытекает из следующей теоремы:
Каждая матрица может быть преобразована в уменьшенную ступенчатую форму строк с помощью последовательности операций с элементарными строками.
Сокращенная форма эшелона строк полезна, поскольку она обеспечивает стандартную форму для записи матриц и систем линейных уравнений, что упрощает их решение. Например, если матрица находится в форме эшелона с уменьшенными строками, вы можете легко найти решения соответствующей системы линейных уравнений, считывая значения переменных из матрицы.
Как найти сокращенную форму эшелона строк
Для выполнения этого процесса необходимо выполнить ряд элементарных преобразований строк, а именно:
Тип I: Поменять местами две строки.
Тип II: умножить все элементы строки на ненулевой скаляр.
Тип III: добавьте к одной строке другую, умноженную на скаляр.
Мы можем начать с получения опорной точки 1 в первой строке. Если в первом столбце есть какой-то элемент, отличный от нуля, мы получим этот пивот в позиции (1,1). Если все элементы в первом столбце равны нулю, переходим ко второму (позиция (1,2)) и так далее. В зависимости от случая есть несколько способов получить эту опорную точку 1 (конечно, мы можем получить ее в любой строке, а затем обменять, чтобы перейти к первой).
В каждой из оставшихся строк элемент, расположенный ниже опорной точки, становится равным 0 путем прибавления первого, умноженного на удобный скаляр (преобразование типа III).
[Al(h3O)6]Cl3 катион NaCl Ca[Hg(CN)4] анион Ca(NO3)2 Строение комплексных соединений объясняется с позиций координационной теории швейцарского учёного А. Вернера. В комплексном соединении различают 2 сферы: внешнюю и внутреннюю. Пример анионного комплекса Пример катионного комплекса [Cu(Nh4)4][PtCl4] Пример катионанионного комплекса Пример нейтрального комплекса (соль Пейроне) Кислоты Основания Соли Неэлектролиты H[AuCl4] [Ag(Nh4)2]OH [Ni(Nh4)6](NO3)2 [Pt(Nh4)2Cl2] h3[SiF6] [Cu(Nh4)4](OH)2 Na3[AlF6] [Ni(CO)4] Наиболее часто встречаются комплексы с координационным числом центрального атома 6 и 4
10.
Получение• CuSO4 + 4Nh4 • Fe(CN )2 + 4KCN • PtCl2 + 2Nh4 [Cu(Nh4)4]SO4 K4[Fe(CN)6] [Pt(Nh4)2Cl2] Если центральный ион входит в состав комплексного аниона, то анион называется: Fe – феррат Cu – купрат Ag – аргентат Au – аурат Hg – меркурат Zn – цинкат Al – алюминат и т.д. [Cu(Nh4)4]SO4 Сульфат тетраамминмеди(II) K3[Fe(CN)6] Гексацианоферрат(III) калия [Co(Nh4)4(h3O)CN]3(PO4)2 Фосфат цианоакватетраамминкобальта (III) K3[Ag(S2O3)2] Дитиосульфатоаргентат(I) калия [Ru(h3O)(Nh4)4SO3] Cульфитотетраамминакварутений Nh5[Cr(SCN)4(Nh4)2] Диамминтетрацианатохромат(III) аммония Химические свойства: 1. Диссоциация • Na2[Zn(OH)4] →2Na+ + [Zn(OH)4]2[Zn(OH)4]2- — комплексный анион • [Ag(Nh4)2]Cl → [Ag(Nh4)2]+ + Cl[Ag(Nh4)2]+ — комплексный катион 2. Реакция по внешней сфере . • [Cu(Nh4)4]SO4+BaCl2 = BaSO4 + [Cu(Nh4)4]Cl2 . FeCl3 K 4 Fe CN 6 KFe Fe CN 6 3 3KCl 3. Подвергаются термическому разложению [Cu(Nh4)4]SO4 CuSO4 + 4Nh4 4. Разрушаются в результате образования более устойчивого комплекса [Ag(Nh4)2]Cl + 2KCN =K[Ag(CN)2] + + KCl + 2Nh4 5.Разрушаются под действием кислот [Ag(Nh4)2]Cl+2HNO3= AgCl↓+2Nh5 NO3 Качественная реакция на Fe2+ (кр. кровяная соль) FeSO4+K3[Fe(CN)6]→K2SO4+KFe3+[Fe+2(CN)6] Турнбулева синь Качественная реакция на Fe3+ (жёлт. кровяная соль) FeCl3 K 4 Fe CN 6 KFe Fe CN 6 3 3KCl Берлинская лазурь 3+ +2 Переносчиком кислорода у моллюсков и членистоногих является не гемоглобин, а комплексное соединение -гемоцианин Кровь этих животных имеет голубой цвет. Построить формулы веществ по названию: 1. Тетрайодомеркурат(II) калия 2. Гексагидроксохромат (III) натрия 3. Нитрат гексаамминникеля(II) 4. Тетрахлораурат (III) водорода Определить степень окисления комплексообразователя и комплексного иона, назвать вещество: 1. Na4[Fe(CN)6] 2. [Ag(Nh4)2]Cl
English
Русский
Правила
7 фактов о структуре Льюиса Cu[(nh4)4] 2+, характеристики —
By Сарнали Мукерджи
В этой статье будут описаны структура Льюиса Cu[(nh4)4] 2+ и 7 фактов об ионе Cu[(Nh4)4]2+, который в химии называют тетрааминовым ионом меди. Факты будут раскрыты через набросок структуры Льюиса этого сложного соединения.
Факты, которые будут описаны, иллюстрирующие внутренние характеристики соединения:
Cu[(Nh4)4]2+ Название
Cu[(Nh4)4]2+ Степень окисления
Cu[(Nh4)4]2+ Магнитный момент
Cu[(Nh4)4]2+ Форма
Cu[(Nh4)4]2+ Цвет
Cu[(Nh4)4]2+ Координационное число
Cu[(Nh4)4]2+ структура Льюиса
Cu[(Nh4)4]2+ Реакция с HCl
Cu[(Nh4)4]2+ Название
Комплексному соединению всегда необходимо иметь конкретное название, которое определяется применением правил, установленных IUPAC (Международным союзом теоретической и прикладной химии) с помощью его химической формулы.
Название, данное ИЮПАК Cu[(Nh4)4]2+, представляет собой тетраамин меди. Тетрааммин определяется выделением четырех аминогрупп, которые присоединены к одному иону меди.
Cu[(Nh4)4]2+ Степень окисления
Степень окисления является одной из важных характеристик, которая определяет избыток или недостаток электронов в соединениях, то есть приобретение или потерю электрона в центральном атоме.
Степень окисления Cu в тетраамине меди составляет +2, поскольку общий комплексный ион имеет дефицит двух электронов, а соединение (Nh4)4 (аммиак) имеет нулевую степень окисления.
Cu[(Nh4)4]2+ Магнитный момент
Магнитный момент относится к характеристике, которая указывает на магнитный дипольный момент, то есть тенденцию объектов или элементов выравниваться с определенной величиной магнитного поля.
Это соединение накладывает парамагнитный эффект. Магнитный момент Cu[(Nh4)4]2+ составляет 1. 73 м.н., это свойство надежно подтверждает факт о количестве n-спаренных электронов в этом соединении. Существует определенная формула, помогающая вычислить количество неспаренных электронов.
Cu[(nh4)4] 2+ Структура Льюиса из Википедия
Формула: количество неспаренных электронов = √n(n+2), где N относится к величине магнитного момента. Следовательно, это показывает, что один неспаренный электрон присутствует в соединении [√п (п + 2) = √1.73(1.73+2) = 1].
Cu[(Nh4)4]2+ Форма
Форма соединений выявляется путем расчета количества электронов, присутствующих в соединениях. Электронная конфигурация соединений достоверно отражает представление о форме соединения. Электронная конфигурация Куппера в Cu[(Nh4)4]2+: 1s2 2s2 2p6 3s2 3p6 3d0 4s0. Cu демонстрирует гибридизацию dsp2 с одним спаренным электроном. Соединение показывает квадратную планировочную форму в химии вместо тетраэдрической геометрии.
Cu[(Nh4)4]2+ Цвет
Цвет соединений зависит от обмена электронами на орбиталях. Перемещение электронов с одной орбитали на другую может влиять на степень окисления и отражает определенные элементы соединений.
Cu[(Nh4)4]2+ цвет от Википедия
В норме медь не имеет неспаренных электронов с заполненной D10-орбиталью, поэтому выглядит бесцветной. В Cu[(Nh4)4]2+ медь получает неспаренный электрон после добавления аммиака, что придает всему иону темно-синий цвет.
Cu[(Nh4)4]2+ Координационное число
Координационное число относится к числу атомов, которые координируют или создают связи с другим атомом, участвующим в формировании общей сложной структуры.
Как соединение с четырьмя аммиаками, присоединяйтесь к одному ион Cu2+, что означает факт что соединение имеет четыре координационных числа. Этот комплексный ион имеет четыре сильных лиганда, поэтому он рассматривается как стабильное координационное соединение в ряду.
Cu[(Nh4)4]2+ структура Льюиса
Число валентных электронов, присутствующих в Cu, равно 1, но в Cu2+ оно становится равным 9. Четыре соединения Nh4 содержат четыре атома азота, каждое из которых имеет одну неподеленную пару после связывания с тремя молекулами водорода. Следовательно, общее число электронов, присутствующих в четырех соединениях Nh4, равно 8.
Для получения стабильности происходит частичное разделение электронов между соединениями Cu2+ и Nh4. Согласно с Метод рисования Льюиса., структура этого соединения Cu занимает центральное положение в геометрии.
Полный эскиз Структура Льюиса показывает, что это соединение создает ковалентную связь, поскольку Cu является металлом, а Nh4 выглядит как неметалл, а как ионное соединение. Эта структура актуальна для представления электронного устройства и отсутствия электронов в комплексном ионе.
Cu[(Nh4)4]2+ Реакция с HCl
Соляная кислота (HCl) является сильной коррозионной кислотой, которая легко вступает в реакцию с соединениями. Реакция между этим комплексным ионом и HCl является своего рода реакцией, которая вызывает эффект изменения цвета как химическое свойство кислоты при взаимодействии с ионами.
HCl сначала дает водный раствор тетраамина меди (II) в присутствии кислорода. Затем он дает избыток атома хлора I-й в середине реакции. Сбалансированное химическое уравнение приводится ниже, чтобы описать его процесс и причину проявления эффекта изменения цвета.
Cu[(Nh4)4(h3O)2]2+ + 4Cl = [CuCl]2- + 4Nh4 + 2h3O
Атом хлора добавляется к меди, так как сильное присутствие HCl разрушает ион. Другая часть комплексного иона, представляющая собой Nh4 (аммиак), извлекается из соединения. Это реакция замещения лиганда так как здесь происходит замещение сильного лиганда аммиаком в присутствии сильной кислоты.
Добавление ионов хлорида к меди превращает темно-синий цвет водного раствора тетраамина меди (II) вводят Зеленый цвет. Это происходит за счет перехода электронов. Однако после вытеснения аммиака такое изменение окраски происходит Комплекс тетрахлормеди (II) и молекула воды выступает в качестве основного продукта этой реакции.
Заключение
В этой статье широко обсуждался тетраамин меди (II), один из комплексных ионов и координационных соединений в химии. Исключительный факты об этом соединении и его реакции с соляной кислотой были объяснены в этой статье.
Cu(Nh4)4SO4·h3O (Cu[SO4][Nh4]4[h3O]) Кристаллическая структура — SpringerMaterials
Получить доступ
СИФ
Скачать справку (pdf)
У вас нет доступа к этому содержимому
Опции доступа
Дополнительные опции доступа
Свяжитесь с нами, если вам нужна помощь в доступе к этому контенту
Узнайте об институциональных подписках
Просмотр трехмерной интерактивной структуры
Цитировать эту страницу
Цитата
Кристаллографические данные
Параметры ячейки
Стандартизированные данные
Космическая группа
У вас нет доступа к этому контенту
Опции доступа
Дополнительные опции доступа
Свяжитесь с нами, если вам нужна помощь в доступе к этому контенту
Узнайте об институциональных подписках
и
б
с
α
β
γ
а/б
б/к
к/с
В
Координаты атома
Стандартизированный
У вас нет доступа к этому содержимому
Параметры доступа
Дополнительные возможности доступа
Свяжитесь с нами, если вам нужна помощь в доступе к этому контенту
Узнайте об институциональных подписках
Опубликовано
У вас нет доступа к этому контенту
Опции доступа
Дополнительные опции доступа
Свяжитесь с нами, если вам нужна помощь в доступе к этому контенту
Узнайте об институциональных подписках
Детали эксперимента
У вас нет доступа к этому контенту
Опции доступа
Дополнительные опции доступа
Свяжитесь с нами, если вам нужна помощь в доступе к этому контенту
Узнайте об институциональных подписках
Ссылка
У вас нет доступа к этому содержимому
Опции доступа
Дополнительные опции доступа
Свяжитесь с нами, если вам нужна помощь в доступе к этому контенту
Узнайте об институциональных подписках
3D интерактивная структура
У вас нет доступа к этому контенту
Опции доступа
Дополнительные опции доступа
Свяжитесь с нами, если вам нужна помощь в доступе к этому контенту
неорганическая химия — Каков диапазон значений n в [Cu(Nh4)4(h3O)n]SO4?
спросил
Изменено
3 года, 11 месяцев назад
Просмотрено
1к раз
$\begingroup$
В моем учебнике ион меди представлен как «сульфат тетрааминомеди(II)-1-вода», поэтому я предположил, что этот комплексный ион имеет только 1 кристаллизационную воду.
Затем в статье в Википедии ион представлен как $\ce{[Cu(Nh4)4(h3O)_n]SO4}$. Используется та же формула, но количество кристаллизационной воды варьируется и обозначается $n$. Таким образом, $n$ должен иметь диапазон значений.
Мой вопрос: что определяет значение $n$? При каком условии $n$ будет больше или меньше?
неорганическая химия
вода
координационные соединения
ионы
$\endgroup$
1
$\begingroup$
Сульфат тетраамминмеди(II) или сульфат купраммония может быть синтезирован в виде его моногидрата (ссылка 1):
Раствор $\pu{50 г}$ мелкодисперсного $\ce{CuSO4 . 5 ч3O}$ в $\pu{75 мл}$ конц. аммиака и $\pu{50 мл}$ воды фильтруют и осаждают медленным добавлением $\pu{75 мл}$ спирта. Выдержав несколько часов на холоде, кристаллы фильтруют на воронке Бюхнера, промывают смесью спирта и конц. аммиака (1:1), затем спиртом и эфиром и высушивают отсасыванием. 9{16}_{2h}$, с четырьмя молекулами в элементарной ячейке. Определение структуры проводилось с помощью проекций Паттерсона и Фурье на $(001)$ и $(100)$. Структура состоит из слоев, параллельных $(010)$, образованных квадратами $\ce{Cu(Nh4)4}$ и тетраэдрами $\ce{SO4}$, скрепленными молекулами $\ce{h3O}$. Атомы меди, лежащие в плоскостях симметрии, имеют четырех ближайших $\ce{Nh4}$ соседей ($\ce{Cu-N} = \pu{2,04-2,06 Å}$) и еще двух $\ce{h3O}$ соседи ($\ce{Cu-O} = \pu{2,59-3,37 Å}$), которые дополняют искаженную октаэдрическую координационную группу. Каждый тетраэдр $\ce{SO4}$ ($\ce{S-O} = \pu{1,40-1,43-1,46 Å}$) имеет два атома кислорода, связанных водородными связями с соседними молекулами $\ce{h3O}$ ($\ ce{OH…O} = \pu{2,67 Å}$).
Все эти данные относятся к комментарию @andselisk, который утверждает, что сульфат тетраамминмеди (II) представляет собой моногидрат , но, вероятно, поэтому статья Википедии предлагает его как $\ce{[Cu(Nh4)4(h3O)n] SO4}$, что это соединение может быть гигроскопичным, и поэтому его содержание воды может отклоняться (или меняться).
Дешевые авиабилеты Калининград Усть-кут от 13 683 рублей, цена на билеты на самолет Калининград — Усть-кут
Самые популярные авиабилеты из Калининграда в Усть-Кут
Прямой перелёт из Калининграда в Усть-Кут очень удобный и быстрый способ долететь в место назначения. Стоимость билета на самолет зависит от времени суток. Цена билетов также зависит от дня недели. Стоит учесть, что вылеты в предпраздничные дни, как правило дороже. Как правило, самые дорогие билеты продаются меньше чем за 14 дней до вылета на вечер пятницы — туда, и вечером воскресенья — обратно.
* Конечная стоимость билета по маршруту Калининград Усть-кут действительна на момент поиска и может меняться в зависимости от количества оставшихся мест на рейс по данному тарифу.
Авиабилеты из Калининграда
Хочется купить билет, но ещё не определились, куда? Найдите идею в списке всех рейсов из Калининграда в другие города.
Авиабилеты в популярные города России
Усть-кут — не единственный город в России, держи ещё ТОП-10, куда можно улететь из Калининграда!
Москва
от 543 ₽
Санкт-Петербург
от 900 ₽
Екатеринбург
от 1 666 ₽
Краснодар
от 1 062 ₽
Казань
от 1 067 ₽
Ростов-на-Дону
от 849 ₽
Уфа
от 1 512 ₽
Самара
от 1 062 ₽
Сочи
от 889 ₽
Симферополь
от 1 050 ₽
Mathway | Популярные задачи
1
Найти точное значение
sin(30)
2
Найти точное значение
sin(45)
3
Найти точное значение
sin(30 град. )
4
Найти точное значение
sin(60 град. )
5
Найти точное значение
tan(30 град. )
6
Найти точное значение
arcsin(-1)
7
Найти точное значение
sin(pi/6)
8
Найти точное значение
cos(pi/4)
9
Найти точное значение
sin(45 град. )
10
Найти точное значение
sin(pi/3)
11
Найти точное значение
arctan(-1)
12
Найти точное значение
cos(45 град. )
13
Найти точное значение
cos(30 град. )
14
Найти точное значение
tan(60)
15
Найти точное значение
csc(45 град. )
16
Найти точное значение
tan(60 град. )
17
Найти точное значение
sec(30 град. )
18
Найти точное значение
cos(60 град. )
19
Найти точное значение
cos(150)
20
Найти точное значение
sin(60)
21
Найти точное значение
cos(pi/2)
22
Найти точное значение
tan(45 град. )
23
Найти точное значение
arctan(- квадратный корень из 3)
24
Найти точное значение
csc(60 град. )
25
Найти точное значение
sec(45 град. )
26
Найти точное значение
csc(30 град. )
27
Найти точное значение
sin(0)
28
Найти точное значение
sin(120)
29
Найти точное значение
cos(90)
30
Преобразовать из радианов в градусы
pi/3
31
Найти точное значение
tan(30)
32
Преобразовать из градусов в радианы
45
33
Найти точное значение
cos(45)
34
Упростить
sin(theta)^2+cos(theta)^2
35
Преобразовать из радианов в градусы
pi/6
36
Найти точное значение
cot(30 град. )
37
Найти точное значение
arccos(-1)
38
Найти точное значение
arctan(0)
39
Найти точное значение
cot(60 град. )
40
Преобразовать из градусов в радианы
30
41
Преобразовать из радианов в градусы
(2pi)/3
42
Найти точное значение
sin((5pi)/3)
43
Найти точное значение
sin((3pi)/4)
44
Найти точное значение
tan(pi/2)
45
Найти точное значение
sin(300)
46
Найти точное значение
cos(30)
47
Найти точное значение
cos(60)
48
Найти точное значение
cos(0)
49
Найти точное значение
cos(135)
50
Найти точное значение
cos((5pi)/3)
51
Найти точное значение
cos(210)
52
Найти точное значение
sec(60 град. )
53
Найти точное значение
sin(300 град. )
54
Преобразовать из градусов в радианы
135
55
Преобразовать из градусов в радианы
150
56
Преобразовать из радианов в градусы
(5pi)/6
57
Преобразовать из радианов в градусы
(5pi)/3
58
Преобразовать из градусов в радианы
89 град.
59
Преобразовать из градусов в радианы
60
60
Найти точное значение
sin(135 град. )
61
Найти точное значение
sin(150)
62
Найти точное значение
sin(240 град. )
63
Найти точное значение
cot(45 град. )
64
Преобразовать из радианов в градусы
(5pi)/4
65
Найти точное значение
sin(225)
66
Найти точное значение
sin(240)
67
Найти точное значение
cos(150 град. )
68
Найти точное значение
tan(45)
69
Вычислить
sin(30 град. )
70
Найти точное значение
sec(0)
71
Найти точное значение
cos((5pi)/6)
72
Найти точное значение
csc(30)
73
Найти точное значение
arcsin(( квадратный корень из 2)/2)
74
Найти точное значение
tan((5pi)/3)
75
Найти точное значение
tan(0)
76
Вычислить
sin(60 град. )
77
Найти точное значение
arctan(-( квадратный корень из 3)/3)
78
Преобразовать из радианов в градусы
(3pi)/4
79
Найти точное значение
sin((7pi)/4)
80
Найти точное значение
arcsin(-1/2)
81
Найти точное значение
sin((4pi)/3)
82
Найти точное значение
csc(45)
83
Упростить
arctan( квадратный корень из 3)
84
Найти точное значение
sin(135)
85
Найти точное значение
sin(105)
86
Найти точное значение
sin(150 град. )
87
Найти точное значение
sin((2pi)/3)
88
Найти точное значение
tan((2pi)/3)
89
Преобразовать из радианов в градусы
pi/4
90
Найти точное значение
sin(pi/2)
91
Найти точное значение
sec(45)
92
Найти точное значение
cos((5pi)/4)
93
Найти точное значение
cos((7pi)/6)
94
Найти точное значение
arcsin(0)
95
Найти точное значение
sin(120 град. )
96
Найти точное значение
tan((7pi)/6)
97
Найти точное значение
cos(270)
98
Найти точное значение
sin((7pi)/6)
99
Найти точное значение
arcsin(-( квадратный корень из 2)/2)
100
Преобразовать из градусов в радианы
88 град.
Джинсы прямого кроя — Кут из ткани
Джинсы прямого кроя — Кут из ткани
перейти к содержанию
Быстрый просмотр
Прямые штаны Reese со средней посадкой по щиколотку (Glory Wash)
89,00 $
4. 181818182/5.0
8 отзывов
Распродажа
Быстрый просмотр
Rachael Mom Jean, Plus (исследовательская стирка)
$53,40
$98.00
4.333333333 / 5.0
6 отзывов
Быстрый просмотр
Укороченные прямые брюки Amy со средней посадкой (черный)
89,00 $
5,0 / 5,0
16 отзывов
Быстрый просмотр
Укороченные прямые брюки Amy со средней посадкой (в честь стирки)
89,00 $
4,0 / 5,0
18 отзывов
Быстрый просмотр
Rachael High Rise Fab Mom Jean (воображаемая стирка)
$99,00
4,454545455/5,0
11 отзывов
Быстрый просмотр
Прямые штаны Stevie со средней посадкой (стойкая стирка)
89,00 $
4. 714285714/5.0
7 отзывов
Распродажа
Быстрый просмотр
Джинсы Rachael High Rise Fab Ab Mom (экологически безопасный — Emulate Wash)
$56.40
$94,00
4,888888889 / 5,0
9 отзывов
Распродажа
Быстрый просмотр
Прямые штаны Reese с покрытием до щиколотки (черные)
58,80 $
$98.00
4.4761
/5.0
23 отзыва
Быстрый просмотр
Укороченные прямые брюки Amy со средней посадкой (имитация стирки)
89,00 $
4,8 / 5,0
5 отзывов
Быстрый просмотр
Rachael High Rise Fab Ab Mom Необработанный подол
$92,00
5,0 / 5,0
3 отзыва
Быстрый просмотр
Reese High Rise Fab Ab Ankle Straight Raw Hem (белый)
$92,00
5,0 / 5,0
2 отзыва
Распродажа
Быстрый просмотр
Reese High Rise Fab Ab Straight Leg (Приходская стирка)
49,00 $
$98. 00
4,4 / 5,0
13 отзывов
Быстрый просмотр
Укороченные прямые брюки Amy со средней посадкой (оливковый)
89,00 $
4.
1904/5.0
42 отзыва
Быстрый просмотр
Rachael High Rise Fab Ab Mom с необработанным краем (Perect Wash)
$92,00
5,0 / 5,0
7 отзывов
Быстрый просмотр
Укороченные прямые штанины со средней посадкой Amy (Bubbly Wash)
89,00 $
3,75 / 5,0
12 отзывов
Быстрый просмотр
Прямые штаны Reese со средней посадкой и необработанным краем до щиколотки (Respect Wash)
$94,00
3,6 / 5,0
5 отзывов
Распродажа
Быстрый просмотр
Chrissie High Rise Fab Ab Straight Leg (Поддерживающая стирка)
46,00 $
$92. 00
4,857142857/5,0
10 отзывов
Распродажа
Быстрый просмотр
Джинсы Rachael с завышенной талией и необработанным краем (Poised Wash)
46,00 $
$92.00
4.333333333 / 5.0
12 отзывов
1 2 3 … 6 Далее »
Женские прямые джинсы Kut from the Kloth
Дом
Кут из ткани
Женщины
Джинсы
Прямая нога
Прямая ножка
Сортировать по:
Джинсы скинни прямого кроя Kut from the Kloth
25 долларов США
111 долларов
Размер: 10
Кут из ткани
канатик
Джинсы прямого кроя Kut from the Kloth с высокой посадкой и задним карманом из смеси спандекса
25 долларов
Размер: 10
Кут из ткани
эбйорк3
Sz4 Kut from the Kloth Синие джинсы средней стирки Стиль KP782MA2
$20
Размер: 4
Кут из ткани
мью2ю
KUT from the Kloth Джинсы Stevie Straight Leg Dark Wash Denim, размер 4
40 долларов
Размер: 4
Кут из ткани
дахлитуп78
КУТ ИЗ ТКАНИ СИНИЙ ДЖИНС
$35
89 долларов
Размер: Размер 2
Кут из ткани
шопинг_просто
Джинсы прямого кроя с эффектом потертости Kut from the Kloth Stevie
$18
$99
Размер: 2
Кут из ткани
кимберли5120
Кут из джинсовой ткани размер 6
$23
Размер: 6
Кут из ткани
длиптак
Джинсы прямого кроя KUT from the Kloth с потертостями, размер 2
$30
Размер: 2
Кут из ткани
Джессикаропер97
Женские темные выстиранные джинсы Kut From The Kloth, размер 2
$44
Размер: 2
Кут из ткани
Лесик777
Черные прямые узкие джинсы Kut from the Kloth 2 26
$33
89 долларов
Размер: 2
Кут из ткани
silas_avery
Кут из ткани Kloth джинсы с необработанным краем 28
$35
Размер: AU 14
Кут из ткани
Стефенс23
Женские темные выстиранные джинсы Kut From The Kloth, размер 2
$38
Размер: 2
Кут из ткани
Лесик777
Прямые джинсы до щиколотки NWT Kut from the Kloth Naomi High Rise Fab Ab Girlfriend 22W
$54
Размер: 22 Вт
Кут из ткани
боходжое
Джинсы прямого кроя из темного денима Kut From The Kloth, размер 6
$30
95 долларов
Размер: 6
Кут из ткани
Аннарейдд
Kut From The Kloth Женские прямые джинсы с высокой посадкой, размер 12, наборные, с необработанным краем
$32
Размер: 12
Кут из ткани
просто 4эль
Укороченные женские джинсы прямого кроя с высокой посадкой Kut From The Kloth Amy размера 12 из денима
$34
Размер: 12
Кут из ткани
просто 4эль
EUC Kut from the Kloth Женские джинсы-бойфренды Екатерины прямого денима, размер 14
30 долларов
89 долларов
Размер: 14
Кут из ткани
doxie_finds
Женский Кут от The Kloth размер 12 полупотёртый. Прямо подходит.
$26
Размер: 12
Кут из ткани
бабеттелехью
Джинсы прямого кроя до щиколотки KUT From The Cloth Reese
$35
89 долларов
Размер: 4
Кут из ткани
bchicandбережливый
Kut from the Kloth — Джинсы прямого кроя
$28
99 долларов
Размер: 6
Кут из ткани
огненная долина
Kut from the Kloth Черные джинсы прямого кроя Stevie, размер 12, большие, со средней посадкой, Regula
$36
Размер: Большой
Кут из ткани
wvsrylegirl
Джинсы-скинни Kut Diana в стиле модерн средней длины
$17
89 долларов
Размер: 2
Кут из ткани
пинксболлер
Черные прямые джинсы Kut from the Kloth
25 долларов США
Размер: 6
Кут из ткани
jamiem569
Женские джинсы прямого кроя из денима средней степени стирки Kut from the Kloth, размер 10
40 долларов
Размер: 10
Кут из ткани
лоластринки
1X ВЫБОР ХОЗЯИНА! Джинсы прямого кроя Kut From The Kloth
45 долларов США
$98
Размер: 14
Кут из ткани
pjkpurple
СЗТ Укороченные винтажные прямые джинсы с высокой посадкой Rosa Kut from the Kloth, размер 8
$50
Размер: 8
Кут из ткани
верующие романы
Женские синие джинсы Kut From The Kloth, размер 4S, прямые штанины
$17
Размер: 4
Кут из ткани
царапины
Kut from the Kloth, укороченный узкий бойфренд Angie, размер 0
$30
Размер: 0
Кут из ткани
удивительные находки
Джинсы-бойфренды Katy from the Kloth, размер 12
$38
Размер: 12
Кут из ткани
chat_gris
Kut from Kloth Женские джинсы Reese прямого кроя до щиколотки средней степени стирки, размер 10
$32
85 долларов
Размер: 10
Кут из ткани
метрусиасниколь
KUT Женские ботильоны Reese с прямыми штанинами для миниатюрных | Размер 4P
$30
90 долларов
Размер: 4P
Кут из ткани
юлесиннаш
Женские прямые темные джинсы Kut From The Kloth, размер 10
$15
Размер: 10
Кут из ткани
метрусиасниколь
Кут из Клот Джинс Стиви Прямая Нога
$35
Размер: 4
Кут из ткани
часы желаний
Эмма Кут Джинсы From the Kloth Размер 29/8
$30
$333
Размер: 8
Кут из ткани
девочкапещерный центр
KUT from the Kloth Прямые джинсы Reese с высокой посадкой до щиколотки, размер 6, синий необработанный край
40 долларов
999 долларов
Размер: 6
Кут из ткани
люблю это00
KUT FROM THE KLOTH REESE ДЖИНСЫ С ПРЯМЫМИ БРЮКАМИ СРЕДНЕЙ СТИРКИ, РАЗМЕР 2
$50
Размер: 2
Кут из ткани
bijou_bug
Прямые джинсы средней стирки Kut From The Kloth, размер 4
$20
80 долларов
Размер: 4
Кут из ткани
dsutherd99
Джинсы Kut from the kloth, размер 8, черные, со средней посадкой, прямые, стрейч
$22
Размер: 8
Кут из ткани
Венера встречается с Плутоном
Джинсы-скинни до щиколотки NWT KUT From The Kloth стрейч с эффектом потертости-10
$39
99 долларов
Размер: 10
Кут из ткани
Грейфорд и Харп
Женские джинсы прямого кроя KUT FROM THE KLOTH Stevie свободного кроя
$35
Размер: 10
Кут из ткани
Челси выбор
Джинсы с потертостями Kut from the Kloth 6p со средней посадкой
$30
89 долларов
Размер: 6P
Кут из ткани
бужибранды
Джинсы Mom с высокой посадкой Kut From The Cloth
$55
Размер: 14 Вт
Кут из ткани
ветмейл
Kut From The Kloth Reese Прямые штанины с необработанным краем до щиколотки, размер 2
Даже если ваша профессиональная деятельность никак не связана с точными науками, хотя бы основные математические действия на английском знать нужно. Они встречаются не только в специальной литературе, но и в фильмах, книгах, повседневной речи. В этой статье мы рассмотрим термины, связанные с арифметическими задачами, дробями, процентами. В конце я привожу озвученные карточки со основными словами на тему математики.
Обратите внимание, здесь рассматриваются только математические термины. Если вы ищете сведения о числительных, рекомендую эту статью: Числительные в английском языке.
Содержание:
Основные математические действия на английском: сложение, вычитание, умножение и деление.
Дроби и проценты на английском языке.
Простые дроби — common fractions.
Десятичные дроби — decimal fractions, decimals.
Проценты в английском языке, трудности с числом глагола.
Возведение в степень в английском.
Математические выражения со скобками.
Карточки с английскими словами на тему «Математика».
Основные математические действия на английском: сложение, вычитание, умножение и деление
Наиболее употребительные математические термины относятся к арифметике. Обратите внимание, в русском языке у нас есть такие слова, как:
Сложение, вычитание, деление, умножение — название действия.
Плюс, минус, разделить, умножить — название действия, которое мы используем в речи, когда читаем выражение, именно оно используется чаще всего.
В английском языке точно так же, поэтому представим арифметические действия в виде таблицы:
Пройдите тест на уровень английского:
Узнать свой уровень
Название действия (сущ.)
Название действия (глагол)
Используется в речи
Addition — сложение
Add — прибавлять
Plus — плюс
Subtraction — вычитание
Subtract — вычитать
Minus — минус
Multiplication — умножение
Multiply by — умножать на
Times — умножить
Division — деление
Divide by — делить на
Divided by — разделить
Equality — равенство
Equals to \ is equal to — равняться чему-то
Equals to \ is equal to \ is — равно
Сама арифметическая задача (например, 2+2) называется problem (по-научному) или sum (разговорный вариант), решение или ответ — answer, а глагол «решать» — to solve (the problem).
Приведу примеры:
2+2=4 — Two plus two equals four.
7-2=5 — Seven minus two equals five.
Часто вместо equals или is equal to говорят просто is.
5×3=15 — Five times three is fifteen.
8÷4=2 — Eight divided by four is two.
Дроби на английском языке
Простые дроби — common fractions
Если у вас с математикой так же «прекрасно», как у меня, напомню самое основное о дробях.
Простые дроби (common fractions) состоят из числителя (numerator) и знаменателя (denominator). Напоминаю, числитель сверху, знаменатель снизу 🙂 Если число состоит из целого и дроби, например 1½, — это называется смешанная дробь или смешанное число (mixed numeral).
Числитель выражается количественным числительным, а знаменатель порядковым. Наиболее употребительные в речи дроби 1/2, 1/3, 1/4 в русском языке имеют не только «умные» называния «одна вторая», «одна третья», одна четвертая, но и простые: половина, треть, четверть. В английском точно так же.
1/2 — a half, one half.
1/3 — a third, one third.
1/4 — a quarter, one fourth.
1/5 — one fifth.
1/6 — one sixth.
2/3 — two thirds.
3/4 — three fourths.
1/8 — one eighth.
1/10 — a tenth.
1/100 — a hundredth.
1¼ — one and a quarter.
1½ — one and a half.
1¾ — one and three quarters.
Обратите внимание, когда числитель больше одного, к окончанию добавляется -s, так как знаменатель используется во множественном числе (как и в русском: две третьих, три четвертых).
Существительное, которое определяется дробью, используется с предлогом of:
3/4 mile — Three fourths of a mile.
1/4 bottle — A quarter of a bottle.
Существительное, определяемое смешанной дробью, используется без предлога, но во множественном числе:
2 ½ miles — Two and a half miles.
1¼ bottles — One and a quarter bottles.
Десятичные дроби — decimal fractions, decimals
В английском в десятичных дробях (decimals) целое от дроби отделяется точкой (point), а не запятой, как у нас.
Ноль перед точкой называется zero или (британский вариант) nought. Ноль после точки может называться oh (как буква «o»), zero, nought. Лично я для простоты всегда говорю zero, потому что это слово проще выговорить и расслышать. Если целое число в дроби равно нулю, его часто опускают в речи, начиная говорить сразу с «point».
Целое число читается как обычное количественное числительное, например 45.1 — forty five point one. Но в дробной части каждая цифра читается отдельно тоже как количественное: 2.45 — two point four five (а не two point forty five).
Примеры:
0.1 — Point one, zero point one.
0.35 — Point three five, zero point three five.
1.25 — One point two five.
35.158 — Thirty five point one five eight.
15.05 — Fifteen point zero five.
Проценты в английском языке, трудности с числом глагола
Сотые доли могут выражаться с помощью процентов, тогда используется стандартный значок % и слово percent, всегда использующееся в единственном числе.
1% — One percent.
10% — Ten percent.
17% — Seventeen percent.
Трудность может вызвать число глагола в выражениях с процентами. Например:
Twenty percent of the students are/is present. — 20% студентов присутствуют.
The remaining twenty percent of the script has/have been rewritten. — Оставшиеся 20% сценария были переписаны.
В таких случаях глагол согласуется в числе с существительным после of:
Twenty percent of the students are present (т. к. students — мн. число).
The remaining twenty percent of the script has been rewritten (т. к. script — ед. число).
Возведение в степень в английском
Для обозначение степени используются выражения to the power of five, to the fifth power, raised to the power of five, raised to the fifth power. Для 2-ой и 3-ей степени используются термины «в квадрате» (squared) и «в кубе» (cubed).
32 — Three squared, three to the second power.
33 — Three cubed, three to the third power.
104 — Ten to the fourth power, ten to the power of four.
3024 — Thirty to the power of twenty four.
Квадратный корень называется square root:
√16 = 4 — The square root of sixteen is four.
√25 = 5 — The square root of twenty five is five.
Математические выражения со скобками
Круглые скобки называются parentheses (ед. число parenthesis) или, проще, round brackets. Если выражение стоит в скобках, и к нему применяется операция, используется слово quantity.
(2+3)×4=24 — Two plus three quantity times four equals to twenty four.
(3+5)2=64 Three plus five quantity squared is sixty four.
Карточки с английскими словами на тему «Математика»
Математические термины из этой статьи можно выучить с помощью карточек на Quizlet и PDF-карточек для распечатки.
math (mathematics)
математика
do the math
считать (матем. действия)
problem (sum)
арифметическая задача
to solve
решать
answer
ответ
digit
цифра
number
число
odd number
нечетное число
even number
четное число
to add
прибавлять
to subtract
вычитать
to multiply by
умножать на
to divide by
делить на
to be equal to
равняться
plus
плюс
minus
минус
times
умножить
divided by
разделить
equals to
равно
common fractions
простые дроби
numerator
числитель
denominator
знаменатель
mixed number
смешанное число (дробь)
half
половина
quarter
четверть
decimals (decimal fractions)
десятичные дроби
point
точка (в дес. дробях)
percent
процент
to the power of five
в пятой степени
two squared
два в квадрате
two cubed
два в кубе
square root
квадратный корень
round brackets
круглые скобки
brackets
квадратные скобки
to round up the numbers
округлять числа
Словарные карточки
Здравствуйте! Меня зовут Сергей Ним, я автор этого сайта, а также книг, курсов, видеоуроков по английскому языку.
Подпишитесь на мой Телеграм-канал, чтобы узнавать о новых видео, материалах по английскому языку.
У меня также есть канал на YouTube, где я регулярно публикую свои видео.
n = \underbrace{x \times x \times \cdots \times x}_{n \text{times}}.
\end{собрать*}
Мы можем назвать это «возведение $x$ в степень $n$», «возведение $x$ в степень $n$» или просто «возведение $x$ в степень $n$». а.
\конец{выравнивание*}
9b$, где $b$ — рациональные числа, приближающиеся к $a$.)
Что такое экспонента?
до вершины
МАТЕМАТИКА
ОБЗОР: ПОЛЕЗНАЯ МАТЕМАТИКА ДЛЯ КАЖДОГО
СЕКЦИЯ
3. ЧТО ТАКОЕ ПОКАЗАТЕЛЬ?
Коэффициенты
и пропорции
Алгебраический
Выражения
Экспоненты
Логарифмы
Глоссарий
и ссылки
Показатель степени относится
количество раз, когда число умножается само на себя. Например,
2 к 3-му (записывается так: 2 3 ) означает:
2 х 2 х 2 = 8 .
2 3 не то же самое, что 2 x 3 = 6.
Помните, что число, возведенное в степень 1, является само собой. Например,
1 =
5 1 = 5 .
Есть несколько особых случаев:
1. а 0 = 1
Когда показатель степени равен нулю,
как и в 6 0 , выражение всегда равно 1.
а 0 = 1
6 0 = 1
14 356 0 = 1
2. а -м = 1 / а м
Когда показатель степени отрицательный
число, результатом всегда является дробь. Дроби состоят из числителя
над знаменателем. В этом случае числитель всегда равен 1.
Чтобы найти знаменатель, предположим, что отрицательный показатель положителен,
и возведите число в эту степень, например:
а -м = 1/а м
6 -3 = 1 / 6 3
Вы можете иметь
переменная в данной степени, например, 3 , что означает a x a x a.
Вы также можете иметь число в переменной степени, например
2 м , что будет означать 2, умноженное само на себя m раз.
Алгебраическая дробь только с виду страшна и зубаста. На деле — это коллаборация старых-добрых обыкновенных дробей и буквенных множителей. Давайте познакомимся с ними поближе и узнаем, что такое сокращение алгебраических дробей.
Определение
Алгебраическая дробь — это дробь, в числителе и/или знаменателе которой стоят алгебраические выражения (буквенные множители). Вот так:
Алгебраическая дробь содержит буквенные множители и степени.
Необыкновенной алгебраическую дробь делают буквы. Если заменить их на цифры, то карета превратится в тыкву — алгебраическая дробь тут же станет обыкновенной.
Если вы засомневались, что должно быть сверху — числитель или знаменатель — переходите по ссылке и освежите знания по теме обыкновенных дробей.
Реши домашку по математике на 5.
Подробные решения помогут разобраться в самой сложной теме.
Сокращение алгебраических дробей
Сократить алгебраическую дробь — значит разделить ее числитель и знаменатель на общий множитель. Общий множитель числителя и знаменателя в алгебраической дроби — многочлен и одночлен.
Сокращение дробей с буквами и степенями проходит в три этапа:
Определите общий множитель.
Сократите коэффициенты.
Поделите все числители и все знаменатели на общий множитель.
Для сокращения степеней в дробях применяем правило деления степеней с одинаковыми основаниями:
Пример сокращения дроби со степенями и буквами:
Следуя формуле сокращения степеней в дробях, сокращаем x3 и x2
Всегда делим на наименьшее значение в степени
Вычитаем: 3 — 1
Получаем сокращенную дробь.
Запоминаем: сокращать можно только одинаковые буквенные множители. Иными словами, сокращать можно только дроби с одинаковыми буквами.
❌ Так нельзя
✅ Так можно
Примеры сокращения алгебраических дробей с одночленами:
Пример сокращения №1.
Как решаем:
Общий множитель для числителя и знаменателя — 8.
Х и x2 делим на x и получаем ответ.
Получаем сокращенную алгебраическую дробь.
Пример сокращения №2.
Как решаем:
Общий множитель для числителя и знаменателя — 7.
b3 и b делим на b.
Вычитаем: 3 — 1 и получаем ответ.
Получаем сокращенную дробь.
Курсы по математике в онлайн-школе Skysmart помогут подтянуть оценки, подготовиться к контрольным, ВПР и экзаменам.
Сокращение алгебраических дробей с многочленами
Чтобы верно сократить алгебраическую дробь с многочленами, придерживайтесь двух главных правил:
сокращайте многочлен в скобках только с таким же многочленом в скобках;
сокращайте многочлен в скобках целиком — нельзя сократить одну его часть, а другую оставить. Не делайте из многочленов одночлены.
❌ Так нельзя
✅ Так можно
Запомните: многочлены в алгебраической дроби находятся в скобках. Между этими скобками вклиниться может только знак умножения. Всем остальным знакам там делать нечего.
Примеры сокращения алгебраических дробей с многочленами:
Последовательно сокращаем: сначала x, затем (x+c), далее сокращаем дробь на 6 (общий множитель).
Сокращаем многочлены a+b (в дроби их 3).
Вынесение общего множителя при сокращении дробей
При сокращении алгебраических дробей иногда не хватает одинаковых многочленов. Для того, чтобы они появились, вынесите общий множитель за скобки.
Чтобы легко и непринужденно выносить множитель за скобки, пошагово выполняйте 4 правила:
Найдите число, на которое делятся числа каждого одночлена.
Найдите повторяющиеся буквенные множители в каждом одночлене.
Вынесите найденные буквенные множители за скобку.
Далее работаем с многочленом, оставшимся в скобках.
Алгебра не терпит неточность. Всегда проверяйте, верно ли вынесен множитель за скобки — сделать это можно по правилу умножения многочлена на одночлен.
Для умножения одночлена на многочлен нужно умножить поочередно все члены многочлена на этот одночлен.
Пример 1.
Как решаем:
Выносим общий множитель 6
Делим 42/6
Сокращаем получившиеся одинаковые многочлены.
Пример 2.
Как решаем: в числителе выносим общий множитель a за скобки, в знаменателе выносим общий множитель c за скобки и сокращаем оставшиеся в скобках многочлены.
Сокращение дробей. Формулы сокращенного умножения
Перед формулами сокращенного умножения не устоит ни одна дробь — даже алгебраическая.
Чтобы легко ориентироваться в формулах сокращенного умножения, сохраняйте и заучивайте таблицу. Формулы подскажут вам, как решать алгебраические дроби.
Квадрат суммы
(a+b)2 = a2 + 2ab + b2
Квадрат разности
(a-b)2 = a2 — 2ab — b2
Разность квадратов
a2 – b2 = (a – b)(a+b)
Куб суммы
(a+b)3 = a3 + 3a2b + 3ab2 + b3
Куб разности
(a-b)3 = a3 — 3a2b + 3ab2 — b3
Сумма кубов
a3 + b3 = (a + b)(a2— ab+b2)
Разность кубов
a3 — b3 = (a — b)(a2+ ab+b2)
Примеры сокращения дробей с помощью формул сокращенного умножения:
Применяем формулу разности квадратов a2 − b2 = (a − b) (a + b) и сокращаем одинаковые многочлены.
Чтобы раскрыть тему сокращения алгебраических дробей и полностью погрузиться в мир числителей и знаменателей, решите следующие примеры для самопроверки.
Примеры сокращения дробей за 7 и 8 классы
Сократите дроби:
Тема сокращения алгебраических дробей достаточно обширна, и требует к себе особого внимания. Чтобы знания задержалась в голове хотя бы до ЕГЭ, сохраните себе памятку по сокращению дробей. Этот алгоритм поможет не растеряться при встрече с алгебраическими дробями лицом к лицу.
Чтобы сократить дробь, найдите общий множитель числителя и знаменателя.
Поделите числитель и знаменатель на общий множитель.
Чтобы разделить многочлен на множители, вынесите общий множитель за скобку.
Второй способ разделить многочлен на множители — применить формулы сокращенного умножения.
Выучите все формулы сокращенного умножения — они помогут легко преобразовывать выражения и экономить время при решении задач.
Можно забыть свое имя, но формулу разности квадратов помнить обязательно — она будет встречаться чаще других.
Всегда проверяйте результат сокращения: алгебра — точна, коварна и не любит давать вторые шансы.
Шпаргалки для родителей по математике
Все формулы по математике под рукой
Анастасия Белова
К предыдущей статье
247.6K
Осевая и центральная симметрия
К следующей статье
Равнобедренный треугольник: свойства, признаки и формулы
Получите план обучения, который поможет понять и полюбить математику
На вводном уроке с методистом
Выявим пробелы в знаниях и дадим советы по обучению
Расскажем, как проходят занятия
Подберём курс
Разложение на простые множители / Обыкновенные дроби / Справочник по математике 5-9 класс
Главная
Справочники
Справочник по математике 5-9 класс
Обыкновенные дроби
Разложение на простые множители
Любое составное число можно представить в виде произведения простых чисел, то есть разложить на простые множители. Заметим, что при любом способе получается одно и то же разложение, если не учитывать порядка записи множителей.
Например, число 150 — это произведение чисел 15 и 10, то есть 150 = 1510. Числа 15 и 10 составные, значит, их тоже можно разложить на множители: 15 = 35, 10 = 25. Значит, получаем, что 150 = 3525, в данном произведении все множители простые, то есть мы разложили число 150 на простые множители.
Разложим 150 на простые множители другим способом:
150 = 530 = 556 = 5532.
Как мы видим, в обоих случаях получились одни и те же простые множители. Обычно множители принято записывать в порядке их возрастания, то есть от меньшего к большему:
150 = 2355.
При разложении на множители удобно использовать признаки делимости на 2, 3 и 5.
Например, разложим на простые множители число 3 528:
Данное число заканчивается на чётную цифру 8, значит, оно делится на 2. Получаем 3 528 : 2 = 1 764. Проведем вертикальную черту и запишем слева от неё делимое 3 528, а справа делитель — 2. Частное запишем под числом 3 528.
Число 1 764 заканчивается тоже на четную цифру 4, а , значит, тоже делится на 2, при делении получаем в частном число 882.
882 тоже делится на 2. При делении в частном получаем число 441.
Число 441 заканчивается нечетной цифрой, значит, оно не делится на 2. Сумма цифр данного число равна 4 + 4 + 1 = 9, 9 делится на 3, значит, 441 тоже делится на 3. При делении в частном получаем число 147.
1 + 4 + 7 = 12, значит, 147 делится на 3. При делении в частном получаем число 49.
4 + 9 = 13. 13 не делится на 3, значит, 49 тоже не делится на 3. Также число 49 не заканчивается цифрами 0 и 5, а, значит, оно не делится на 5. Но 49 делится на простое число 7. При делении получаем в частном 7.
7 — это простое число и оно делится только на простое число 7. В частном получаем 1:
Разложение закончено. При этом справа от черты мы получили все простые множители, на которые можно разложить число 3 528. Получаем:
3 528 = 2223377.
При этом одинаковые множители можно заменить степенью, то есть мы можем записать:
3 528 = 233272.
Советуем посмотреть:
Доли. Обыкновенные дроби
Сравнение дробей
Делители и кратные
Признаки делимости на 10, на 5 и на 2
Четные и нечетные числа
Признаки делимости на 9 и на 3
Простые и составные числа
Наибольший общий делитель
Наименьшее общее кратное
Деление и дроби
Сложение и вычитание дробей с одинаковыми знаменателями
Смешанное число
Сложение и вычитание смешанных чисел
Основное свойство дроби
Решето Эратосфена
Приведение дробей к общему знаменателю
Сравнение, сложение и вычитание дробей с разными знаменателями
Задание 2,
Виленкин, Жохов, Чесноков, Шварцбург, Учебник, часть 2
7 класс
Номер 349,
Мерзляк, Полонский, Якир, Учебник
Номер 434,
Мерзляк, Полонский, Якир, Учебник
Обзор факторинга с примерами — Smartick
Еще раз привет! Вы знаете, что такое факторинг ? Вы знаете, для чего он используется? В этом посте мы ответим на эти вопросы.
Как разложить на множители
Факторизация числа выполняется путем записи числа как произведения всех его простых множителей.
Пример:
12 = 2 x 2 x 3
Вы также можете выразить это с помощью степеней:
12 = 2 2 x 3
Если вы хотите увидеть больше примеров факторинга, нажмите здесь.
Теперь, когда мы знаем, как факторизовать, давайте посмотрим, для чего он полезен и как мы можем его использовать.
Вычисление наименьшего общего кратного (НОК)
НОК набора чисел вычисляется путем факторизации всех чисел. После факторизации выбираются общие кратные с наибольшим показателем степени и кратные, которые не являются общими. Они умножаются, и в результате получается LCM этих чисел. Пример:
LCM (12, 20)
Мы учитываем числа:
12 = 2 2 x 3
20 = 2 2 x 5
Теперь мы выбираем. общие множители (22) и необщие множители (3 и 5) Умножьте множители:
2 2 x 3 x 5 = 60
Следовательно, НОК (12, 20) = 60
Вычисление наибольшего общего делителя (GCF)
GCF рассчитывается путем факторизации всех чисел. После факторизации выбираются общие множители, возведенные в меньшую степень. После этого коэффициенты умножаются. Пример:
GCF (30, 40)
Мы учитываем числа:
30 = 2 x 3 x 5
40 = 2 3 x 5
9
Теперь мы выбрать самые высокие общие множители, возведенные в низшую степень (2 и 5) Умножить множители:
2 x 5 = 10 Следовательно, GCF (30, 40) = 10
Для упрощения дробей
Дроби упрощаются путем деления числителя и знаменателя на одно и то же число до тех пор, пока они не будут иметь общих делителей. Пользоваться факторингом в этом случае очень просто: мы факторизуем числитель и знаменатель, затем сокращаем общие множители и, наконец, умножаем оставшиеся множители. Пример: Сначала разложите числитель и знаменатель. Теперь сократите множители, которые находятся в числителе и знаменателе. Факторы, которые остались, это факторы, которые мы должны умножить. И это упрощенная дробь!
Выполнение умножения
Некоторые умножения могут быть проще, если сначала выполнить разложение на множители, поскольку множители можно удобно сгруппировать. Пример:
25 x 12
Мы учитываем числа:
25 = 5 x 5
12 = 2 x 2 x 3
Следовательно, 25 x 12 = 5 x 5 x 2 х 2 х 3
Возьмем 2 и 5 с одной стороны и остальные множители с другой.
Таким образом, мы можем выполнить умножение гораздо проще.
Как правило, факторинг можно использовать для упрощения числовых расчетов. Можете ли вы придумать какой-либо другой способ использования факторинга? Поделись с нами!
И вы уже знаете: чтобы продолжить изучение факторинга и всех предметов математики, зарегистрируйтесь в Smartick и станьте математическим гением ☺.
Подробнее:
Автор
Последние сообщения
Smartick
Команда создания контента. Мультидисциплинарная и мультикультурная команда, состоящая из математиков, учителей, профессоров и других специалистов в области образования! Они стремятся создать наилучший математический контент.
Последние сообщения от Smartick (посмотреть все)
Как разложить многочлены на множители
Лучший способ разложения многочленов на множители начинается с преобразования дробей в более простые термины. Полиномы представляют собой алгебраические выражения с двумя или более членами, точнее, суммой нескольких членов, которые имеют разные выражения одной и той же переменной. Стратегии, которые помогают упростить многочлены, включают в себя выделение наибольшего общего множителя с последующим группированием уравнения в его наименьшие члены. То же самое справедливо и при решении многочленов с дробями.
Определение многочленов с дробями
У вас есть три способа просмотра многочленов фраз с дробями. Первая интерпретация касается многочленов с дробями в качестве коэффициентов. В алгебре коэффициент определяется как числовая величина или константа, находящаяся перед переменной. Другими словами, коэффициенты для 7_a_, b и (1/3) c равны 7, 1 и (1/3) соответственно. Таким образом, двумя примерами многочленов с дробными коэффициентами будут: 92 + x — 2}
оценивается с помощью разложения на неполные дроби, которое, кстати, включает разложение многочленов на множители, и в простейшей форме будет:
Основы факторинга – распределительное свойство и метод FOIL
Факторы представляют собой два числа, которые при умножении дают третье число. В алгебраических уравнениях факторизация определяет, какие две величины были перемножены, чтобы получить данный многочлен. Дистрибутивное свойство сильно соблюдается при умножении многочленов. Распределительное свойство по существу позволяет умножать сумму, умножая каждое число по отдельности перед добавлением произведений. Обратите внимание, например, как применяется распределительное свойство в примере:
7(10x + 5) \text{ для получения бинома } 70x + 35.
Но, если два бинома умножаются вместе, то используется расширенная версия распределительного свойства с помощью метода FOIL. FOIL представляет собой аббревиатуру для перемножения первых, внешних, внутренних и последних терминов. Следовательно, разложение полиномов на множители влечет за собой выполнение метода FOIL в обратном порядке. Возьмем два вышеупомянутых примера с полиномами, содержащими дробные коэффициенты. Выполнение метода FOIL в обратном порядке для каждого из них приводит к коэффициентам 92 + \frac{3}{4}x + \frac{1}{8} = \bigg(x + \frac{1}{4}\bigg)\bigg(x + \frac{1}{2} \bigg)
Действия при разложении полиномиальных дробей на множители
Как видно из вышеизложенного, полиномиальные дроби включают полином в числителе, деленный на полином в знаменателе.
Простые вещества: молекулы состоят из атомов одного вида (атомов одного элемента).
Пример: h3, O2,Cl2, P4, Na, Cu, Au.
Сложные вещества (или химические соединения): молекулы состоят из атомов разного вида (атомов различных химических элементов).
Пример: h3O, Nh4, OF2, h3SO4, MgCl2, K2SO4.
Аллотропия — способность одного химического элемента образовывать несколько простых веществ, различающихся по строению и свойствам.
Пример:
С — алмаз, графит, карбин, фуллерен.
O — кислород, озон.
S — ромбическая, моноклинная, пластическая.
P — белый, красный, чёрный.
Явление аллотропии вызывается двумя причинами:
Различным числом атомов в молекуле, например кислород O2 и озон O3.
Образованием различных кристаллических форм, например алмаз, графит, карбин и фуллерен (смотри рисунок выше).
Основные классы неорганических веществ
Бинарные соединения
Вещества, состоящие из двух химических элементов называются бинарными (от лат. би – два) или двухэлементными.
Названия бинарных соединений образуют из двух слов – названий входящих в их состав химических элементов.
Первое слово обозначает электроотрицательную часть соединения – неметалл, его латинское название с суффиксом –ид стоит всегда в именительном падеже.
Второе слово обозначает электроположительную часть – металл или менее электроотрицательный элемент, его название стоит в родительном падеже, затем указывается степень окисления (только в том случае, если она переменная):
Запомни!
Bh4 — боран
B2H6 — диборан
Ch5 — метан
Sih5 — силан
Nh4 — аммиак
Ph4 — фосфин
Ash4 — арсин
Оксиды
Оксиды — сложные вещества, состоящие из двух химических элементов, один из которых кислород в степени окисления -2.
Общая формула оксидов: ЭхОу
Основные оксиды
Основные оксиды — оксиды, которым соответствуют основания.
Основные оксиды образованы металлом со степенью окисления +1, +2.
Пример
Соответствие основных оксидов и оснований
Na2O — Na2(+1)O(-2) — NaOH
MgO — Mg(+2)O(-2) — Mg(OH)2
FeO — Fe(+2)O(-2) — Fe(OH)2
MnO — Mn(+2)O(-2) — Mn(OH)2
Амфотерные оксиды
Амфотерные оксиды — оксиды, которые в зависимости от условий проявляют либо основные, либо кислотные свойства.
Амфотерные оксиды образованы металлом со степенью окисления +3, +4, а также некоторыми металлами (Zn, Be) со степенью окисления +2.
Кислотные оксиды — оксиды, которым соответствуют кислоты.
Кислотные оксиды образованы неметаллом, а также металлом со степенью окисления +5, +6, +7.
Пример
Соответствие кислотных оксидов и кислот
SO3 — S(+6)O3(-2) — h3SO4
N2O5 — N2(+5)O5(-2) — HNO3
CrO3 — Cr(+6)O3(-2) — h3CrO4
Mn2O7 — Mn2(+7)O7(-2) — HMnO4
Гидроксиды
Гидроксиды — сложные вещества, состоящие из трех элементов, два из которых водород со степенью окисления +1 и кислород со степенью окисления -2.
Общая формула гидроксидов: ЭхОуНz
Основания
Основания — сложные вещества, состоящие из ионов металла и одной или нескольких гидроксо-групп (ОН-).
В основаниях металл имеет степень окисления +1, +2 или вместо металла стоит ион аммония Nh5+
Пример
NaOH, Nh5OH, Ca(OH)2
Амфотерные гидроксиды
Амфотерные гидроксиды — сложные вещества, которые в зависимости от условий проявляют свойства оснований или кислот.
Амфотерные гидроксиды имеют металл со степенью окисления +3, +4, а также некоторые металлы (Zn, Be) со степенью окисления +2.
Пример
Zn(OH)2, Be(OH)2, Al(OH)3, Cr(OH)3
Кислоты
Кислоты — сложные вещества, состоящие из атомов водорода и кислотных остатков.
В состав кислот входит неметалл или металл со степенью окисления +5, +6, +7.
Пример
h3SO4, HNO3, h3Cr2O7, HMnO4
Соли
Соли- соединения, состоящие из катионов металлов (или Nh5+) и кислотных остатков.
Общая формула солей: MexAcy
Me — металл
Ac — кислотный остаток
Пример
KNO3 — нитрат калия
(Nh5)2SO4 — сульфат аммония
Mg(NO3)2 — нитрат магния
Названия кислот и кислотных остатков
Кислота
Кислотный остаток
Название
Формула
Название
Формула
Соляная (хлороводородная)
HCl
Хлорид
Cl(-)
Плавиковая (фтороводородная)
HF
Фторид
F(-)
Бромоводородная
HBr
Бромид
Br(-)
Иодоводородная
HI
Иодид
I(-)
Азотистая
HNO2
Нитрит
NO2(-)
Азотная
HNO3
Нитрат
NO3(-)
Сероводородная
h3S
Сульфид Гидросульфид
S(2-) HS(-)
Сернистая
h3SO3
Сульфит Гидросульфит
SO3(2-) HSO3(-)
Серная
h3SO4
Сульфат Гидросульфат
SO4(2-) HSO4(-)
Угольная
h3CO3
Карбонат Гидрокарбонат
СО3(2-) НСО3(-)
Кремниевая
h3SiO3
Силикат
SiO3(2-)
Ортофосфорная
h4PO4
Ортофосфат Гидроортофосфат Дигидроортофосфат
РО4(3-) НРО4(2-) Н2РО4(-)
Муравьиная
НСООН
Формиат
НСОО(-)
Уксусная
СН3СООН
Ацетат
СН3СОО(-)
Полезные ссылки
Источник материала
Классификация неорганических веществ (видео)
Классификация неорганических веществ. Сложные вопросы (видео)
Кислотные оксиды (видео)
Основные оксиды (видео)
Основания (видео)
Характеристика солей (видео)
Дополнительные материалы
Классификация и номенклатура неорганических веществ (видео)
Классификация соединений (видео)
Аллотропные формы углерода (видео)
Реакции алюминия
Химические свойства алюминия
Химические свойства алюминия определяются его положением в периодической системе химических элементов.
Ниже представлены основные химические реакции алюминия с другими химическими элементами. Эти реакции определяют основные химические свойства алюминия.
С чем реагирует алюминий
Простые вещества:
галогены (фтор, хлор, бром и иодин)
сера
фосфор
азот
углерод
кислород (горение)
Сложные вещества:
вода
минеральные кислоты (соляная, фосфорная)
серная кислота
азотная кислота
щелочи
окислители
оксиды менее активных металлов (алюмотермия)
С чем не реагирует алюминий
Алюминий не реагирует:
с водородом
в обычных условиях – с концентрированной серной кислотой (из-за пассивации – образования плотной оксидной пленки)
в обычных условиях – с концентрированной азотной кислотой (также из-за пассивации)
См. подробнее по химии алюминия
Алюминий и воздух
Обычно поверхность алюминия всегда покрыта тонким слоем оксида алюминия, который защищает ее от воздействия воздуха, точнее, кислорода. Поэтому считается, что алюминий не вступает в реакцию с воздухом. Если же этот оксидный слой повреждается или удаляется, то свежая поверхность алюминия реагирует с кислородом воздуха. Алюминий может гореть в кислороде ослепительно белым пламенем с образованием оксида алюминия Al2O3.
Реакция алюминия с кислородом:
4Al + 3O2 —> 2Al2O3
Алюминий и вода
Алюминий реагирует с водой по следующим реакциям [2]:
2Al + 6H2O = 2Al(OH)3 + 3H2 (1)
2Al + 4H2O = 2AlO(OH) + 3H2 (2)
2Al + 3H2O = Al2O3 + 3H2 (3)
В результате этих реакций образуются, соответственно, следующие соединения алюминия:
модификация гидроксида алюминия байерит и водород (1)
модификация гидроксида алюминия богемит и водород (2)
оксид алюминия и водород (3)
Эти реакции, кстати, представляют большой интерес при разработке компактных установок для получения водорода для транспортных средств, которые работают на водороде [2].
Все эти реакции являются термодинамически возможными при температуре от комнатной до температуры плавления алюминия 660 ºС. Все они являются также экзотермическими, то есть происходят с выделением тепла [2]:
При температуре от комнатной до 280 ºС наиболее устойчивым продуктом реакции является Al(OH)3.
При температуре от 280 до 480 ºС наиболее устойчивым продуктом реакции является AlO(OH).
При температуре выше 480 ºС наиболее устойчивым продуктом реакции является Al2O3.
Таким образом, оксид алюминия Al2O3 становится термодинамически более устойчивым, чем Al(OH)3 при повышенной температуре. Продуктом реакции алюминия с водой при комнатной температуре будет гидроксид алюминия Al(OH)3.
Реакция (1) показывает, что алюминий должен самопроизвольно реагировать с водой при комнатной температуре. Однако на практике кусок алюминия, опущенный в воду, не реагирует с водой в условиях комнатной температуры и даже в кипящей воде. Дело в том, что алюминий имеет на поверхности тонкий когерентный слой оксида алюминия Al2O3. Эта оксидная пленка прочно удерживается на поверхности алюминия и предотвращает его реакцию с водой. Поэтому, чтобы начать и поддерживать реакцию алюминия с водой при комнатной температуре необходимо постоянно удалять или разрушать этот оксидный слой [2].
Алюминий и галогены
Алюминий бурно реагирует со всем галогенами – это:
фтор F
хлор Cl
бром Br и
иодин (йод) I,
с образованием соответственно:
фторида AlF3
хлорида AlCl3
бромида Al2Br6 и
иодида Al2Br6.
Реакции водорода со фтором, хлором, бромом и иодином:
2Al + 3F2 → 2AlF3
2Al + 3Cl2 → 2AlCl3
2Al + 3Br2 → Al2Br6
2Al + 3l2 → Al2I6
Алюминий и кислоты
Алюминий активно вступает в реакцию с разбавленными кислотами: серной, соляной и азотной, с образованием соответствующих солей: сернокислого алюминия Al2SO4, хлорида алюминия AlCl3 и нитрата алюминия Al(NО3)3.
Реакции алюминия с разбавленными кислотами:
2Al + 3H2SO4 —> Al2(SO4)3 + 3H2
2Al + 6HCl —> 2AlCl3 + 3H2
2Al + 6HNO3 —> 2Al(NO3)3 + 3H2
С концентрированными серной и соляной кислотами при комнатной температуре не взаимодействует, при нагревании реагирует с образованием соли, окислов и воды.
Алюминий и щелочи
Алюминий в водном растворе щелочи – гидроксида натрия – реагирует с образованием алюмината натрия.
Реакция алюминия с гидроксидом натрия имеет вид:
2Al + 2NaOH + 10H2O —> 2Na[Al(H2O)2(OH)4] + 3H2
Все важнейшие реакции с алюминием
Для полноты информации приводим перечень основных реакций с участием алюминия из фундаментальной книги про алюминий [3]:
Важнейшие реакции с участием алюминия [3]
Источники:
1. Chemical Elements. The first 118 elements, ordered alphabetically / ed. Wikipedians – 2018
2. Reaction of Aluminum with Water to Produce Hydrogen /John Petrovic and George Thomas, U.S. Department of Energy, 2008
3. Тринадцатый элемент: Энциклопедия / А. Дроздов – Библиотека РУСАЛа, 2007.
= | Сбалансированное уравнение химической реакции
Поиск
Результаты поиска по химическому уравнению
Реклама
1 результатов найдено Отображение уравнения от 1 до 1 Страница 1 — Пожалуйста, прокрутите до конца, чтобы увидеть больше результатов
Уравнение Результат #1
Нажмите, чтобы увидеть более подробную информацию и рассчитать
вес/моль >>
png» substance-weight=»44.0095″> CO 2
+
Реклама
Дополнительная информация об уравнении CO
2 + NaAl(OH) 4 → Al(OH) 3 + NaHCO 3
Условия реакции CO2 (двуокись углерода) реагирует с NaAl(OH)4 () ?
Не найдено информации для этого химического уравнения
Объяснение: идеальные условия окружающей среды для реакции, такие как температура, давление, катализаторы и растворитель. Катализаторы — это вещества, которые ускоряют темп (скорость) химической реакции, не потребляясь и не становясь частью конечного продукта.
Катализаторы не влияют на равновесные ситуации.
Как могут происходить реакции с образованием Al(OH)3 (гидроксид алюминия) и NaHCO3 (бикарбонат натрия)?
Явление после реакции CO2 (углекислого газа) с NaAl(OH)4 ()
Это уравнение не несет никакой конкретной информации о явлении.
В этом случае вам просто нужно наблюдать, чтобы убедиться, что вещество продукта NaHCO3 (бикарбонат натрия), появляющийся в конце реакции.
Или если какое-либо из следующих реагентов NaAl(OH)4 , исчезает
Какую другую важную информацию вы должны знать о реакции
У нас нет дополнительной информации об этой химической реакции.
Категории уравнений
Нажмите, чтобы увидеть более подробную информацию и рассчитать
вес/моль >>
Другие вопросы, связанные с химическими реакциями CO
2 + NaAl(OH) 4 → Al(OH) 3 + NaHCO 3
Вопросы, связанные с реагентом CO2 (двуокись углерода)
9 0018 Какие химические и физические характеристики CO2 (углекислого газа)? В каких химических реакциях используется CO2 (углекислый газ) в качестве реагента?
Вопросы, связанные с реагентом NaAl(OH)4 ()
Каковы химические и физические характеристики NaAl(OH)4 ()? В каких химических реакциях используется NaAl(OH)4 () в качестве реагента?
Вопросы, связанные с продуктом Al(OH)3 (гидроксид алюминия)
Каковы химические и физические характеристики Al(OH)3 ()? Каковы химические реакции, в которых Al(OH)3 (гидроксид алюминия) используется в качестве продукт?
Вопросы, связанные с продуктом NaHCO3 (бикарбонат натрия)
Каковы химические и физические характеристики NaHCO3 ()? Какие химические реакции имеют NaHCO3 (бикарбонат натрия) в качестве продукта?
1 результатов найдено Отображение уравнения от 1 до 1 Страница 1
Дополнительная информация о веществах, которые используют уравнение
Реакция CO2 (Cacbon dioxit) взаимодействие с NaAl(OH)4 (тетрагидроксиалюминат натрия) с образованием Al(OH)3 (Nhom hiroxit)
Реакция с образованием вещества
CO2 (Cacbon dioxit) (двуокись углерода)
C + O 2 → CO 2 2CO + O 2 → 2CO 2 Fe 2 (CO 3 ) 3 + 3Н 2 O → 3CO 2 + 2Fe(OH) 3
Реакция с образованием вещества
NaAl(OH)4 (тетрагидроксиалюминат натрия) ()
Al 2 O 3 + 3H 2 O + 2NaOH → 2NaAl(OH) 4 Al(OH) 3 + NaOH → NaAl(OH) 4 2H 2 O + NaAlO 2 → NaAl(OH) 4
Реакция с образованием вещества
Al(OH)3 (Nhom hiroxit) (гидроксид алюминия)
Реакция с образованием вещества
NaHCO3 (гидрокакабонат натрия) (бикарбонат натрия)
H 2 O + CO 2 + C 2 H 5 ONa → C 2 H 5 OH + NaHCO 3 Ca(HCO 3 ) 2 + NaOH → CaCO 3 + H 2 O + NaHCO 3 2H 2 O + NaCrO 2 + CO 2 → NaHCO 3 + Cr(OH) 3
Essentt — Подобранные продукты
Подобранные вручную продукты необходимы при работе из дома!
Cho khí CO2 vào một binh kin chứa Al(OH)3.
A. Có phản ứng xảy ra và
Кау Хой:
24.02.2020
15 086
Cho khí CO 2 vào một bình kin chứa Al(OH) 3
A. Có phản ứng xảy ra và tạo ra muối Al 2 (CO 3 ) 3 .
B. Có tạo Al 2 (CO 3 ) 3 lúc đầu, sau đó với CO 2 có dư sẽ thu được Al (HCO 3 ) 3 .
C. Lúc đầu tạo Al 2 (CO 3 ) 3 nhưng không bền nó tự phân hủy t или Al(OH) 3 và CO 2 .
D. Khong có phản ứng xảy ra.
Đáp án chính xác
Xem lời giải
Câu hỏi trong đề: Đề thi thử THPTQG Hóa Học chuẩn cấu trúc bộ giáo dục có lời giải !!
Куанг Као
Chọn đáp án D
NHÀ SÁCH VIETJACK
Xem Thêm Kho Sach »
Сач Тонг Он нгу пхап Тионг ань ко Транг Ань бан мой нхот (Чинь ханг)
№ бан 0
₫250. 000
₫189.000
Ханой
Sách Tiếng anh cho người mất gốc — người mới bắt đầu cô Trang Anh (Chính hãng)
№ бан 0
₫200.000
₫159. 000
Ханой
Sách Bộ đề minh họa môa môn toán n thi THPT quốc gia bản 2023 (Chính hang)
№ бан 0
₫200.000
₫159.000
Ханой
Bộ đề minh họa 2023 môn Tiếng Anh cô Trang Anh (Chính hãng)
№ бан 0
₫200. 000
₫159.000
Ханой
Sách tiếng anh cô Trang Anh: 500 bài đọc hiểu đọc điền thi THPT Quốc Gia 2023 (Chính hãng)
№ бан 0
₫200.000
₫189. 000
Ханой
Sách tổng ôn toán học lớp 12 ôn thi THPT quốc gia và luyện thi đánh giá năng lực bản mới nhất (Chính hang)
№ бан 0
₫200.000
₫149.000
Ханой
Sách — 200 công thức giải nhanh Toán 12 (Chính hang)
Да бан 56
100000
59000
Ханой
Tổng ôn cấp tốc luyện thi Đánh giá năng lực (Theo cấu trúc đề thi của ĐHQGHN)
№ бан 0
₫225. 000
₫185.000
Ханой
Combo 2 tập Bí quyết chinh phục điểm cao kì thi THPT Quốc gia môn Toán (Tập 1, tập 2) — Chính hang
№ бан 0
₫336.000
₫252. 000
Ханой
Tăng tốc luyện thi Đánh giá năng lực ĐHQG TP.HCM
№ бан 0
₫225.000
₫185.000
Ханой
CÂU HỎI HOT CÙNG CHỦ ĐỀ
Câu 1:
Tiến hành 4 thí nghiệm sau:
Thí nghiệm 1: Nhung thanh Fe vào dung dịch FeCl 3
Thí nghim 2: Nhúng thanh Fe vào dung dịch CuSO 4 .
Thí nghiệm 3: Nhúng thanh Cu vào dung dịch FeCl 3
Thí nghiệm 4: Cho thanh Fe tiếp xúc với thanh Cu rồi nhung và или навозный душ HCl.
Số trường hợp xuất hiện ăn mòn điện hóa la:
А. 3.
Б. 4.
С. 3.
Д. 2
Ксем Джап »
24. 02.2020
35 116
Кау 2:
Cho các phát biểu sau:
(a). Mantozơ có Khả năng tham gia phản ứng táng bạc
(b).Glucozơ có Khả năng tham gia phản ứng táng bạc
(c). Saccarozơ được cấu tạo từ hai gốc β-glucozơ và α-fructozơ
(d). Sự chuyển hóa tinh bột trong cơ thể người co sinh ra mantozơ
Số phát biểu đúng là:
А. 4.
Б. 2
К. 3
Д. 1
Ксем Джап »
26.02.2020
8 278
Кау 3:
Khi cho 0,56 л (đktc) khí hidro clorua hấp thụ vào 50ml dung dịch AgNO 3 8% (d= 1,1 г/мл). Nồng độ % của HNO 3 чт được là :
А. 3,01%
Б. 6,3%
С. 1,575%
Д. 3,587%
Ксем Джап »
24.02.2020
7 460
Кау 4:
Dung dịch HCl có thể tac dụng được với bao nhiêu chất trong số cac chất sau: NaHCO 3 , NaHSO 4 90 041, NaClO, AgCl, Fe(NO 3 ) 2 , C 2 H 5 ONa, CaC 2
А. 5.
Б. 6.
К. 3.
Д. 4
Ксем Джап »
24.02.2020
5 359
Кау 5:
Hòa tan hết 1,62 gam Ag bằng axit HNO 3 nồng độ 21% (1,2 г/мл), chỉ thu được khí NO. thể tích dung dịch axit nitric tối thiểu cần phản ung là:
А. 7,5 мл
B. 6 мл.
С. 4 мл.
Д. 5 мл
Ксем Джап »
25. 02.2020
4 943
Кау 6:
X là este của một axit hữu cơ đơn chức và rượu đơn chức. Thủy phân hoàn toàn 6,6 gam chất X đã dung 90 мл dung dịch NaOH 1M, lượng NaOH này dư 20% so với lượng NaOH cần dung cho phản ung. Dung dịch sau phản ứng cô cạn thu được chất rắn nặng 5,7 gam. Công thức cấu tạo X la
Решение задач по теме «площади фигур». 9 класс (подготовка к ОГЭ)
Похожие презентации:
Элементы комбинаторики ( 9-11 классы)
Применение производной в науке и в жизни
Проект по математике «Математика вокруг нас. Узоры и орнаменты на посуде»
Знакомство детей с математическими знаками и монетами
Тренажёр по математике «Собираем урожай». Счет в пределах 10
Методы обработки экспериментальных данных
Лекция 6. Корреляционный и регрессионный анализ
Решение задач обязательной части ОГЭ по геометрии
Дифференциальные уравнения
Подготовка к ЕГЭ по математике. Базовый уровень Сложные задачи
РЕШЕНИЕ ЗАДАЧ ПО ТЕМЕ «ПЛОЩАДИ ФИГУР» 9 КЛАСС (ПОДГОТОВКА К ОГЭ) ЗАДАЧА №1. На клетчатой бумаге с размером клетки 1×1 изображён треугольник. Найдите его площадь. ЗАДАЧА №2. На клетчатой бумаге с размером клетки 1×1 изображена трапеция. Найдите её площадь. ЗАДАЧА №3. На клетчатой бумаге с размером клетки 1×1 изображён параллелограмм. Найдите его площадь. ЗАДАЧА №4. На клетчатой бумаге с размером клетки 1×1 изображён ромб. Найдите площадь этого ромба. ЗАДАЧА №5. Периметр квадрата равен 160. Найдите площадь этого квадрата. ЗАДАЧА №6. Найдите площадь квадрата, описанного около окружности радиуса 40. ЗАДАЧА №7. На клетчатой бумаге с размером клетки 1×1 изображён треугольник. Найдите его площадь. ЗАДАЧА №8. На клетчатой бумаге с размером клетки 1×1 изображена трапеция. Найдите её площадь. ЗАДАЧА №9. На клетчатой бумаге с размером клетки 1×1 изображён параллелограмм. Найдите его площадь. ЗАДАЧА №10. Найдите площадь параллелограмма, изображённого на рисунке. ЗАДАЧА №11. Два катета прямоугольного треугольника равны 6 и 7. Найдите площадь этого треугольника. ЗАДАЧА №12. Найдите площадь ромба, если его диагонали равны 14 и 6. ЗАДАЧА №13. Сторона треугольника равна 24, а высота, проведённая к этой стороне, равна 19. Найдите площадь этого треугольника. ЗАДАЧА №14. Основания трапеции равны 4 и 10, а высота равна 5. Найдите площадь этой трапеции. ЗАДАЧА №15. Найдите площадь параллелограмма, изображённого на рисунке. ЗАДАЧА №16. Сторона квадрата равна 5√3. Найдите площадь этого квадрата. ЗАДАЧА №17. Сторона ромба равна 10, а расстояние от точки пересечения диагоналей ромба до неё равно 3. Найдите площадь этого ромба. ЗАДАЧА №19. Периметр ромба равен 36, а один из углов равен 30°. Найдите площадь этого ромба. ЗАДАЧА №20. Площадь параллелограмма ABCD равна 180. Точка E — середина стороны AB. Найдите площадь трапеции DAEC. ЗАДАЧА №21. В равнобедренной трапеции основания равны 2 и 8, а один из углов между боковой стороной и основанием равен 45°. Найдите площадь этой трапеции. ЗАДАЧА №22. На стороне AC треугольника ABC отмечена точка D так, что AD=5, DC=7. Площадь треугольника ABC равна 60. Найдите площадь треугольника ABD. ЗАДАЧА №23. Площадь параллелограмма равна 40, а две его стороны равны 5 и 10. Найдите его высоты. В ответе укажите большую высоту. ЗАДАЧА №24. НАЙДИТЕ ПЛОЩАДЬ ТРЕУГОЛЬНИКА ЗАДАЧА №25. НАЙДИТЕ ПЛОЩАДЬ ТРЕУГОЛЬНИКА ЗАДАЧА №26. ЗАДАЧА №27. ЗАДАЧА №28. ЗАДАЧА №29. ЗАДАЧА №30. ЗАДАЧА №31. ЗАДАЧА №32. 85 ? 40 ЗАДАЧА №33. ЗАДАЧА № 34 ЗАДАЧА № 35 НА КЛЕТЧАТОЙ БУМАГЕ С РАЗМЕРОМ КЛЕТКИ 1 СМ × 1 СМ ИЗОБРАЖЕНА ФИГУРА. НАЙДИТЕ ЕЁ ПЛОЩАДЬ. ОТВЕТ ДАЙТЕ В КВАДРАТНЫХ САНТИМЕТРАХ. ЗАДАЧА № 35 НА КЛЕТЧАТОЙ БУМАГЕ С РАЗМЕРОМ КЛЕТКИ 1 СМ × 1 СМ ИЗОБРАЖЕНА ФИГУРА. НАЙДИТЕ ЕЁ ПЛОЩАДЬ. ОТВЕТ ДАЙТЕ В КВАДРАТНЫХ САНТИМЕТРАХ. ЗАДАЧА № 35 НА КЛЕТЧАТОЙ БУМАГЕ С РАЗМЕРОМ КЛЕТКИ 1 СМ × 1 СМ ИЗОБРАЖЕНА ФИГУРА. НАЙДИТЕ ЕЁ ПЛОЩАДЬ. ОТВЕТ ДАЙТЕ В КВАДРАТНЫХ САНТИМЕТРАХ. ЗАДАЧА № 35 НА КЛЕТЧАТОЙ БУМАГЕ С РАЗМЕРОМ КЛЕТКИ 1 СМ × 1 СМ ИЗОБРАЖЕНА ФИГУРА. НАЙДИТЕ ЕЁ ПЛОЩАДЬ. ОТВЕТ ДАЙТЕ В КВАДРАТНЫХ САНТИМЕТРАХ.
English
Русский
Правила
Геометрия. Задачи на клетчатой бумаге и площади фигур
Чтобы уверенно решать задачи по геометрии — даже такие простые — необходимо выучить основные понятия и формулы.
Это формулы площадей фигур — треугольника (5 формул), параллелограмма, ромба, прямоугольника, произвольного четырехугольника, а также круга. Формулы для длины окружности, длины дуги и площади сектора. Для средней линии треугольника и средней линии трапеции.
Надо знать, что такое центральный и вписанный угол. Знать основные тригонометрические соотношения. В общем, учите основы планиметрии.
Больше полезных формул — в нашем ЕГЭ-Справочнике.
Смотри также материал: Как быстро выучить формулы
В этой статье — основные типы заданий №1 Базового ЕГЭ по математике. Задачи взяты из Банка заданий ФИПИ.
Вычисление длин отрезков, величин углов и площадей фигур по формулам
1. На клетчатой бумаге с размером клетки изображена трапеция. Найдите длину средней линии этой трапеции.
Средняя линия трапеции равна полусумме её оснований:
Ответ: 3.
2. Найдите величину угла ABC. Ответ дайте в градусах.
Величина вписанного угла равна половине величины центрального угла, опирающегося на ту же дугу. Соединим точки А и С с центром окружности и проведем диаметры через точки А и С. Видим, что величина центрального угла АОС равна Тогда
Ответ: 45.
3. Найдите синус угла AOB. В ответе укажите значение синуса, умноженное на
Решение:
Проведем из точки В перпендикуляр к прямой ОА. Из прямоугольного треугольника ОВС по теореме Пифагора:
Осталось умножить найденное значение синуса на
Ответ: 1.
4. Найдите площадь ромба, изображенного на клетчатой бумаге с размером клетки Ответ дайте в квадратных сантиметрах.
Самый простой способ — воспользоваться формулой площади ромба, выраженной через его диагонали:
, где и — диагонали.
Получим:
Ответ: 12.
5. Найдите площадь трапеции, изображенной на клетчатой бумаге с размером клетки Ответ дайте в квадратных сантиметрах.
Площадь трапеции равна произведению полусуммы оснований на высоту:
Основания нашей трапеции равны 4 и 8, а высота равна боковой стороне (поскольку трапеция прямоугольная), то есть 3 см. Площадь трапеции
Ответ: 18.
Нахождение площадей многоугольников сложной формы
А что делать, если надо найти не площадь трапеции или треугольника, а площадь какой-либо сложной фигуры? Есть универсальные способы! Покажем их на примерах из банка заданий ФИПИ и на авторских задачах.
6. Как найти площадь нестандартной фигуры? Например, произвольного четырёхугольника? Простой приём — разобьём эту фигуру на такие, о которых мы всё знаем, и найдем её площадь — как сумму площадей этих фигур.
Разделим этот четырёхугольник горизонтальной линией на два треугольника с общим основанием, равным . Высоты этих треугольников равны и . Тогда площадь четырёхугольника равна сумме площадей двух треугольников: .
Ответ: .
7. В некоторых случаях площадь фигуры можно представить как разность каких-либо площадей.
Не так-то просто посчитать, чему равны основание и высота в этом треугольнике! Зато мы можем сказать, что его площадь равна разности площадей квадрата со стороной и трёх прямоугольных треугольников. Видите их на рисунке? Получаем: .
Ответ: .
Многие репетиторы рекомендуют в таких задачах пользоваться формулой Пика. В ней нет необходимости, однако эта формула довольно интересна.
Согласно формуле Пика, площадь многоугольника равна В+Г/2-1
где В — количество узлов внутри многоугольника, а Г — количество узлов на границе многоугольника.
Узлами здесь названы точки, в которых пересекаются линии нашей клетчатой бумаги.
Посмотрим, как решается задача 7 с помощью формулы Пика:
Синим на рисунке отмечены узлы внутри треугольника. Зеленым — узлы на границе.
Аккуратно посчитав те и другие, получим, что В = 9, Г = 5, и площадь фигуры равна S = 9 + 5/2 — 1 = 10,5.
Выбирайте — какой способ вам больше нравится.
8. Найдите площадь четырехугольника, изображенного на клетчатой бумаге с размером клетки
Такой четырехугольник получится, если от квадрата размером отрезать 2 прямоугольника и 4 треугольника. Найдите их на рисунке.
Площадь каждого из больших треугольников равна
Площадь каждого из маленьких треугольников равна
Тогда площадь четырехугольника
9. Авторская задача. Найдите площадь закрашенной фигуры, изображенной на клетчатой бумаге с размером клетки
Решение:
На рисунке изображен ромб с вырезанным из него квадратом.
Площадь ромба равна половине произведения его диагоналей.
Площадь вырезанного квадрата равна 4.
Площадь фигуры равна 36 — 4 = 32.
Ответ: 32.
Площадь круга, длина окружности, площадь части круга
Длина дуги во столько раз меньше длины окружности, во сколько раз ее градусная мера меньше, чем полный круг, то есть 360 градусов.
Площадь сектора во столько раз меньше площади всего круга, во сколько раз его градусная мера меньше, чем полный круг, то есть 360 градусов.
10. Иногда в задании надо найти площадь не всей фигуры, а её части. Обычно речь здесь идет о площади сектора — части круга.Найдите площадь сектора круга радиуса , длина дуги которого равна .
На этом рисунке мы видим часть круга. Площадь всего круга равна , так как . Остается узнать, какая часть круга изображена. Поскольку длина всей окружности равна (так как ), а длина дуги данного сектора равна , следовательно, длина дуги в раз меньше, чем длина всей окружности. Угол, на который опирается эта дуга, также в раз меньше, чем полный круг (то есть градусов). Значит, и площадь сектора будет в раз меньше, чем площадь всего круга.
Ответ: .
11. На клетчатой бумаге нарисован круг площадью 2,8. Найдите площадь закрашенного сектора.
На рисунке изображен сектор, то есть часть круга. Но какая же это часть? Это четверть круга и еще круга, то есть круга.
Значит, нам надо умножить площадь круга на . Получим:
Ответ: 1,05.
12. На клетчатой бумаге изображены два круга. Площадь внутреннего круга равна 9. Найдите площадь закрашенной фигуры.
Площадь фигуры равна разности площадей двух кругов, один из которых расположен внутри другого. По условию, площадь внутреннего круга равна 9. Радиус внешнего круга относится к радиусу внутреннего как 4 к 3. Площадь круга равна , то есть пропорциональна квадрату радиуса. Значит, площадь внешнего круга в раза больше площади внутреннего и равна 16. Тогда площадь фигуры равна 16 — 9 = 7.
Ответ: 7.
Задачи на координатной плоскости
13. Найдите площадь четырехугольника, вершины которого имеют координаты (4;2), (8;4), (6;8), (2;6).
Заметим, что этот четырехугольник — квадрат. Сторона квадрата a является гипотенузой прямоугольного треугольника с катетами, равными 2 и 4. Тогда
Ответ: 20
14. Найдите площадь четырехугольника, вершины которого имеют координаты
На рисунке изображен параллелограмм (четырехугольник, имеющий две пары параллельных сторон). Площадь параллелограмма равна произведению основания на высоту. Основание равно 2, высота 8, площадь равна 16.
Ответ: 16.
Благодарим за то, что пользуйтесь нашими статьями.
Информация на странице «Геометрия. Применение формул. Задача 1 Базового ЕГЭ по математике» подготовлена нашими редакторами специально, чтобы помочь вам в освоении предмета и подготовке к ЕГЭ и ОГЭ.
Чтобы успешно сдать необходимые и поступить в ВУЗ или техникум нужно использовать все инструменты: учеба, контрольные, олимпиады, онлайн-лекции, видеоуроки, сборники заданий. Также вы можете воспользоваться другими статьями из данного раздела.
Публикация обновлена:
08.04.2023
Найдите с помощью палитры равную площади. Метод поиска областей с помощью палитры
.
Цель:
Задачи урок:
учебный :
познакомить с методом измерения площади криволинейных форм; с приспособлением для измерения площадей — палитрой; закреплять умение находить площади прямолинейных фигур.
развивающая :
развивать внимание, наблюдательность, способность рассуждать, обобщать и делать выводы.
учебная :
воспитывать познавательный интерес к предмету;
поощрять инициативу и самостоятельность в познавательной деятельности.
Во время занятий
1. Организационный момент.
Что самое ценное на земле?
Этот вопрос волнует человечество уже не одну тысячу лет. Вот ответ известного ученого Аль-Бируни: «Знание — самое превосходное из владений. Все к этому стремятся, а оно не приходит.
У нас урок математики — урок открытия новых знаний.
2. Актуализация знаний и фиксация затруднений в пробном учебном действии.
— На столе каждой карточки с названными цифрами, которые необходимо расположить в порядке возрастания ( единиц площади ).
Самопроверка:
Если вы выполнили задание правильно, то, перевернув карты, вы получите предложение: Есть желание, есть способ!
Пусть эти слова станут девизом нашего урока.
Что вы расположили в порядке возрастания? (единицы площади).
О чем будет разговор на уроке? (о площади фигур)
Какие знания вы уже имеете по этой теме?
(Слышны ответы нескольких учеников.)
Расскажите нам о районе.
Что еще вы хотели бы узнать о площади?
Нахождение площади фигур (работа в парах)
Проверка выполненных заданий.
Какова площадь первой фигуры? (18 см 2).
Кто вычислил площадь второй фигуры?
Почему нельзя?
3. Выявление причин затруднений.
В чем проблема? (мы не знаем, как найти площадь фигуры неопределенной формы)
Определить тему урока (Нахождение площади криволинейной фигуры ).
Какова цель нашего урока? (Узнать, как найти площадь криволинейных фигур).
4. Выход из затруднения.
Как мы решим эту проблему? Как вы нашли площадь прямоугольника, если еще не знали формулы его площади? (Площадь прямоугольника мы измерили меркой)
А для криволинейной фигуры можно попробовать этот способ? (Да.)
Как узнать площадь криволинейной фигуры с помощью одноклеточного измерения? (Разделить по меркам, продолжая ряд измерительных ячеек)
Что вы сделаете, разбив фигуру на мерки, чтобы узнать площадь фигуры? (Рассчитаем количество мерок на рисунке)
Работа в группах.
Представители групп записывают свои ответы на доске. Ответы разные.
Почему ответы были разными? (Группа с меньшим количеством измерений поясняет: «Нецелочисленные измерения мы не считали»)
Что делать с неполными измерениями, ребята? Как их посчитать? (Сложите в два такта.)
Еще раз посчитайте количество полных тактов. Неполные измерения.
Учащиеся работают в группах.
Сколько квадратных мер составляет размер фигуры? (Представители групп называют ответы. Все сверяются со своими ответами.)
Подумайте, удобно ли каждый раз отмечать цифру? (Нет)
Как вы думаете, какой выход из этой проблемы нашли математики? (Есть специальная заготовка, разделенная на квадраты) — Да, для ускорения работы люди придумали прибор для определения площади фигур. (Воспитатель раздает детям диапозитивы, разложенные на квадратные сантиметры, и карточки с цифрами.)
Перед вами такое устройство.
Откройте учебники на странице 45 и прочитайте, как это называется.
Палитра — прозрачная пленка, разделенная на равные квадраты: это могут быть квадратные дециметры, квадратные сантиметры, квадратные миллиметры.
5. Вывод алгоритма измерение площади фигур палитрой.
«Алгоритм измерения площади фигур с помощью палитры».
6. Первичная консолидация.
Работа по учебнику (с комментированием)
№ 208 (зеленая цифра)
7. Самостоятельная работа с самопроверкой.
Самостоятельная практическая работа в парах.
8. Подведение итогов, самооценка.
9. Конспект урока.
В школе вы изучаете предметную область. Как вы считаете, нужны ли эти знания в жизни?
Донецкая общеобразовательная школа I-III ступеней №97
Министерство образования и науки
Донецкая Народная Республика
УРОК МАТЕМАТИКА
Тема: Измерение площади фигур палитрой 900 04 .
подготовлено и проведено
учитель начальных классов
Кисенко Н.В.
Метод нахождения площадей с помощью палитры. На начальном курсе математики учащиеся измеряют площади фигур с помощью палитры таким образом: считают количество квадратов, лежащих внутри фигуры, и количество квадратов, через которые проходит контур фигуры. Затем второе число делится пополам и прибавляется к первому. Полученную сумму считают площадью фигуры. Палитра позволяет с определенной точностью измерить площадь фигуры. Для получения более точного результата нужно взять палитру с меньшими квадратами.
Слайд 9 из презентации «Квадраты в геометрии» . Размер архива с презентацией 1561 КБ.
Геометрия 9 класс
конспект других презентаций
«Геометрия вокруг нас» — Границы. Предполагаемый результат изучения элективного курса. Реализация задачи внутрипредметных и межпредметных связей. Практическая часть. Математик. Построить бордюр типа «параллельный перенос». Зеркальное отражение и параллельный перенос. Различные способы построения границ. Геометрия вокруг нас. Способы построения границ. Теоретическая часть программы. Алмаз.
Золотое сечение в жизни — Золотое сечение в архитектуре и искусстве. Золотая спираль в природе. Золотая спираль. Архитектор М.Ф. казаки. Путешествие в историю математики. Холст. Золотое сечение заключается в пропорциях человеческого тела. Что такое золотое сечение. Золотое сечение в природе. Разделение сегмента. Научный аппарат. Золотая спираль в искусстве. Концепция золотого сечения. Золотое сечение. Валуйки. Живопись и золотое сечение.
«Метод золотого сечения» — Золотое сечение в математике. Проект. Золотое сечение в архитектуре. Принцип гармонии. Гармоничны ли люди? Пейзажная фотография. Наши задачи. Рамка выглядит более выигрышно, если вы поместите композицию или предмет не в центр кадра. Золотой прямоугольник. Широкие плечи почти равны высоте тела. Деление прямой по золотому сечению. «Золотая пропорция» в человеке. Золотое сечение в искусстве. Золотая спираль в природе.
«Понятие вектора в геометрии» — Откладывание вектора от заданной точки. Ненулевые векторы. Назовите все изображенные векторы. Длина вектора. Что такое коллинеарные векторы? Определение Физминутка. Шарада. Проверь себя. Векторы называются равными, если они сонаправлены и их длины равны. Коллинеарные векторы. Сегодня на уроке. Понятие вектора. Что называется вектором. Любая точка на плоскости является нулевым вектором. Прямоугольная коробка.
««Координатный метод» 9 класс» — Задача. Равнобедренный прямоугольный треугольник. Уравнение первой степени. Уравнение окружности. Формула. Координаты точки. Докажем формулу. Два противоположных луча. Воспользуемся формулой, чтобы найти расстояние. В скобках указаны координаты точки М. Абсцисса. Отрезок AB параллелен оси OY. Координатный метод. Воспользуемся равенствами. Координаты середины отрезка. Точка M1 (x1; y1) не принадлежит окружности.
«Определение многоугольника» — Ход урока. Полигоны. Вещь. Ломаная линия называется простой, если она не имеет точек самопересечения. Сумма любых n несмежных углов описанного четырехугольника. Какой из них называется закрытым. Вокруг четырехугольника можно описать окружность. Произведение диагоналей произвольного четырехугольника. Определение сломанного. Определение многоугольника. Какой из них называется простым. Какова общая формула суммы углов многоугольника?
План урока открытого урока
Предмет: математика.
Тема: «Измерение площади фигуры. Палитра.»
Цели:
· Познакомить детей со способом нахождения площади фигур различной формы с помощью палитры.
· Научитесь анализировать геометрические фигуры.
· Развивать логическое мышление учащихся, умение точно и аргументировано рассуждать, выделять те стороны наблюдаемых явлений, которые необходимы для изучения и понимания проблемы.
· Улучшить способность решать проблемы.
· Воспитывать интерес к предмету, любознательность, доброжелательное отношение к одноклассникам в сотрудничестве.
Цели урока: Создать условия для самостоятельного поиска знаний.
Оборудование: «Измерение площади фигуры с помощью палитры», презентация
Учебно-методические материалы к занятию:
Учебный курс
1.Организационный момент :
Давай проверим, мой друг, Ты готов начать урок? Все на месте Все в порядке Ручка, книга и блокнот? Все правильно сидят? Все внимательно смотрят?
Все хотят получить Только оценку «5»?
2. Тема урока пост
Учитель: Ребята, мы снова погружаемся в мир скучной математики. Сегодня мы познакомимся с геометрическими фигурами, площадь которых находим по-новому. А как, узнаем на уроке. Мы можем сделать это.
3. Оформление работ в тетрадях.
Математика — королева всех наук. Он нужен в любой науке, в любой профессии, например, археологи. Вы знаете, кто такие археологи? Давайте посмотрим презентацию «Кто такие археологи».
7. Работа по индивидуальным карточкам.
Нахождение площади фигур прямоугольной и квадратной формы.
Давайте поиграем в эту профессию. Вы пришли на место раскопок. Вам нужно определить, какова площадь прямоугольной или квадратной земли, на которой вы будете что-то искать. (У каждого карточка с рисунком, данными. Дети по формуле находят площадь, делают рисунок и записывают в тетрадь.) Один ученик у доски.
S = 5 * 9 = 45 м2
ЧЕК. Воспитатель раздает карточки с указанием правильного ответа, дети зачитывают свои ответы — Сложите ответы и вы увидите археологов, которых вы откопали. Динозавры..jpg «width=»45″ height=»59 src=»>.jpg» width=»49″ height=»65 src=»>
8 . Физминута
Вы, наверное, устали? -Да! «И так все встали». Дружелюбные вытянули шеи как динозавры зашипели: зарычали. Шипели, молчали как динозавры, скакали. Поскакал, поскакал И скрылся за кустом.
9. Знакомство с новой темой.
Я нашел такого динозавра. (На доске вывешен плакат.)
— Можно ли найти его площадь по формуле? Почему?
Существует способ нахождения площади неправильных форм с помощью палитры — прозрачной пленки с нанесенными на нее квадратными сантиметрами.
10. Ознакомление с презентацией «Палитра» 11. Работа по теме урока
Нахождение области динозавра по алгоритму . использование большой палитры. Учитель комментирует.
Алгоритм площади поддона
1. Поставить палитру.
2. Сосчитайте количество
полных квадратов на рисунке.
3. Подсчитайте количество неполных квадратов и разделите это число на 2:
4. Сложите количество полных квадратов и количество неполных квадратов, деленное на 2.
Дети делают запись в тетради.
12. Физминутка. (Парный танец)
Давай, Дино, прыгай, прыгай. мы прыгаем.
И ногами прыгаем, прыгаем, прыгаем .
13. Самостоятельная работа. Найдите площадь яйца с помощью палитры.
Знаете ли вы, как рождаются маленькие динозавры? Из яиц. Продолжаем наши археологические раскопки. Тот, кто найдет яйцо динозавра в пределах квадратного метра, нарисованного на полу, имеет право посмотреть на него.
(Дети находят «яйцо» от киндера, внутри каждого – палитра и надпись «палитра»)
Воспитатель раздает рисунки яйца, просит найти его площадь. Дети самостоятельно находят площадь яйца с помощью палитры.
14. Тестирование.
Мы проверим наши знания. В тесте обведите правильный ответ.
ТЕСТ по теме «Палитра»
1. Прямоугольник, у которого все стороны равны, называется
Треугольник
2. Чтобы найти площадь квадрата или прямоугольника, нужно
Умножить длина по ширине
Найдите сумму всех сторон.
3. С помощью палитры площадь находится следующим образом:
Сложите количество полных квадратов и количество неполных квадратов, деленное на 2. 915 . РЕЗЕРВ .
Работа в группах.
1 группа записывает наименования предметов, площадь которых удобно найти по формуле
2. группа записывает наименования предметов, площадь которых удобно находить с помощью палитры.
16. Конспект урока. Комментирование оценок.
Узнали ли вы что-то новое на уроке?
Что понравилось?
17. Домашнее задание.
Используйте палитру для нахождения площади мелких предметов, рисунков.
Для определения площадей малых участков с криволинейными контурами на плане применяют палитры, чаще прямолинейные. К прямым поддонам относятся известные и наиболее распространенные квадратные и параллельные поддоны.
Квадратная палитра представляет собой сеть взаимно перпендикулярных линий, проведенных через 1-2 мм на прозрачном целлулоиде, оргстекле, фотопленке, стекле или кальке.
Площадь фигуры определяется простым подсчетом ячеек палитры, наложенных на фигуру. Доли клеток, расчлененных контуром на части, учитывают на глаз (рис. 13). Квадратную палитру не рекомендуется определять на плане площадей больше 2 см 2 . Недостаток его применения (помимо того, что площади рассекаемых контуром долей клеток приходится оценивать на глаз) состоит в том, что подсчет числа целых клеток часто сопровождается грубыми ошибками.
Таких недостатков не наблюдается при определении участков параллельной палитрой, представляющей собой лист прозрачного целлулоида, оргстекла или кальки, на котором проведены параллельные линии преимущественно на расстоянии 2 мм друг от друга. Область контура определяется по этой палитре следующим образом. Разложите его по контуру так, чтобы крайние точки a и b расположились посередине между параллельными линиями палитры. Тогда весь контур оказывается разделенным параллельными прямыми на фигуры, близкие к трапециям, с одинаковыми высотами, а отрезки параллельных прямых внутри контура являются средними линиями трапеции (рис. 14). Штриховыми линиями показаны основания этих трапеций.
Сумма площадей трапеций, т.е. площадь контура равна
Следовательно, чтобы получить площадь контура, надо взять сумму средних прямых, т.е. сумму отрезков параллельных линии внутри контура, и умножить на расстояние между ними.
Для упрощения определения площади в раствор компаса последовательно вычерчивают сумму средних линий, которую определяют по масштабной линейке и полученную длину умножают на ч , м (рис. 15). Чтобы не производить таких расчетов, для каждой шкалы строят специальную шкалу, по которой отсчитывают площадь контура, зная сумму средних линий.
Расчет масштаба: М 1:10000, h = 2 мм, при длине шкалы 1 см площадь равна (0,2 см 100 м) (1 см 100 м) = 2000 м 2 = 0,2 га. Параллельная палитра не должна определять на плане участки площадью более 10 см 2 .
4.5. Точность расчета площадей графически и с помощью палитры
При делении сечения на простые фигуры точность расчета для разных вариантов будет неодинаковой. Площадь треугольника графически вычисляется точнее, чем площадь других фигур. Поэтому площадь при делении площади на треугольники вычисляется точнее, чем при делении на другие фигуры (трапеции, прямоугольники). При делении площади на треугольники из всех вариантов лучшим будет тот, в котором треугольники равносторонние или высота. h примерно равно основанию a .
Ошибка уменьшается, если площадь треугольника вычисляется не как
, и по формуле Герона
где
. Это дает уточнение до 13% даже для равностороннего треугольника. Основание треугольника может быть во много раз меньше высоты, если измерять его на местности, а не на плане.
При делении площади на треугольники ошибка площади участка
,
где М — знаменатель числового масштаба плана. Если вычислить дважды, то
.
Количество треугольников, на которые разбит участок, не влияет на ошибку площади. Поэтому при делении участка на треугольники не нужно стремиться к тому, чтобы их было меньше. Точность однократного определения площади квадратных и параллельных поддонов, а также ротометра характеризуется эмпирической формулой
.
«Расчет площади плоских изделий с помощью интеграла синга». Вычислите площадь фигуры Отримана по формуле
На эту тему есть три урока, каждый урок разный.
Цели занятия:
Закрепление и разрушение знаний о певческом интеграле и його дополнить знания области фигур;
Формирование умін сходо застосування знаний и способов изменения этих новых исходных ситуаций; — развитие информационно-коммуникативной культуры студентов;
Проведение познавательной деятельности, вминня деятельности в коллективе, зазятти что достигают мети.
Задание на уроке:
Повторить таблицу и правила определения начальных, понимание криволинейной трапеции, алгоритм определения площади криволинейной трапеции; — необходимо помнить о том, что важно знать, что площадь плоской фигуры.
Формы организации роботов и учащихся: работа в группах.
Принадлежит играемым программам: интерактивная доска Smart Board, «Математика жива».
Выигрышные функции программного обеспечения интерактивной доски:
Функция — шторка:
Функция — клонирование объекта:
Функция — повторная затяжка объекта;
Функция — интеллектуальное перо.
Zavantage:
Вид спереди:
Занятие на тему: «Вычисление площадей фигур для дополнительного интегрирования»
В 11 классе.
Урок Скрыть:
Организационный момент ((проверяется готовность перед занятием, озвучивается тема и цель занятия, записывается номер).906:20
Урок проходить под нос: Скажи мне, и я забуду, Покажи мне, и я запомню, Дай мне дышать самостоятельно, я научусь.
Конфуций.
Этап актуализации предыдущих знаний(Мета этого этапа: повторение таблицы и правил распознавания примитивов, понимание криволинейной трапеции, алгоритм распознавания площади криволинейной трапеции).
Учитель: На предыдущих уроках мы узнали о понятиях первых, из таблицы и правилах их значения.
Питание 1 : Что называется первичным для функции y = f(x) на некотором интервале? Еда 2: Как установить все первичные функции y = f(x), чтобы F(x) была одной из них? Питание 3: Отмените правила познания первого. Как только открывается 2-я горка, открывается занавес, после чего прикрепляется еда для учеников. Голова 1: Знайте одну из первых функций для назначения. (изучение функции — перетаскивание, чтобы установить первую функцию на первую). Менеджер 2: Для назначенной функции узнайте одну из первых строк графика, чтобы пройти через эту точку. (Учащиеся на заданиях самостоятельно выступают, один из учниверя выдается, отвешивая экран).
А) Функции: 2х5 — 3х2; 3 cos x — 4 sin x; 3е х + 5 х — 2; е 2х — cos3x; 1/х + 1/sin 2 х — х.
Первичный: пер | х | -ctgx-х 2/2; 1/2e 2x — 1/3 sin 3x; х 6/3 — х 3; 3 sin x + 4 cos x; 3x + 5x/1n5.
B) Для функции f(x) = 2x + 3, чтобы узнать первую, график должен пройти через точку M(1; 2).
Питание 4: Какую фигуру называют криволинейной трапецией? Задание 3: Запишите мнение назначенной женщины, зафиксированное на слайде. Задание 4: Напишите формулу Ньютона Лейбница.
Задача 5: Вычислить интеграл. (Учащиеся рассчитывают самостоятельно, с повторной проверкой наступления). А) х 2 — 2х) дх; б)
Задача 6: Вычислить площадь фигуры, окруженной линиями у = 0, х = е, у = 1/х. (Учитесь самостоятельно считать задание с дальнейшей перепроверкой, открывая экраны на дощах).
Этап лепки и повторения, убавления и начала при выполнении разных заданий по теме «Расчет площади фигур для дополнительного интегрирования»
906:20
1. Учить определять мощность площади
и указывать торец фигуры, площадь которой можно вычислить по формуле S = Вычислить площадь фигуры, окруженной строки у = 0, у = х 2
— 4. (Один урок по дополнительным функциям — смарт-ручкой писать решения на интерактивном планшете).
2.
Учащиеся обсуждают план вычисления площади фигуры, окруженной линиями у = х 2
— 6х +11 я у = х +1. Кожаная сцена сопровождается занавесом.
Групповая работа. Класс заздалэгід подразделяется на группы. Три ученика практикуются у доски, и решают ученики по трем вариантам (группам разбиений вариантов) на задания: Вычислить площадь фигуры, окруженной линиями: Вариант 1 — у = (х — 3) 2
, у = 0, х = 1, х = 4. Вариант 2 — у = х — 2, у = х 2
— 4х+2. 3 вариант — у = х, у = 5 — х, х = 1, х = 2.
Групповая работа. Для кожных поражений 8 предметных стекол необходимо рассчитать площадь рисунка. У учеников в группах есть набор этих малышек. Научитесь выбирать формулу, по которой можно узнать площадь. Показан слайд, правое кресло показывает формулы, на него наложена функция клонирования. После обсуждения в группах по очереди изучите тип группы и перенесите выбранную формулу или напишите свою, так как на дощах такого нет. Дали следующую дискуссию: — Почему была выбрана именно эта формула? — Чьи є ще способы знаходження площади цієї фигури? — Яка с формулами найзручніша застосуванни 906:20
Домашнее задание.
Подсумок урока. Учний ответит на вопрос: — Что было собрано под часовым уроком? — Какая новая вонь обнаружилась на уроке? — Как он работал с этой группой?
Практическая работа на тему: «Вычисление площади плоских предметов с помощью интеграла синга»
Метароботы: освоить задачу вычисления площади криволинейной плоской фигуры с помощью интеграла синг.
Право собственности: учебная карта, таблица интегралов, лекционный материал на тему: «Одинарный интеграл. Геометрический смысл интеграла.
Методические положения:
1) Ознакомиться с лекционными материалами: «Поющий интеграл. Геометрический смысл интеграла.
Краткие теоретические положения
Единый интеграл функции на відрізку — це границу, до
, к которому pragne интегральная сумма при pragnnn_ ноль является суммой наибольшей части счета.
Нижняя граница интегрирования – это верхняя граница интегрирования.
Для вычисления единого интеграла подайте Формулу Ньютона-
Лейбница:
Геометрический смысл линейного интеграла . Как интегрируется на
Поскольку функция неотрицательна, то численно более ценная площадь криволинейной трапеции равна:
Криволинейная трапеция — фигура, окруженная графиком функции
Ось абсцисс и прямые, .
Возможность различных типов плоских форм, образующих координатную плоскость:
Подобно криволинейной трапеции с основанием, окруженным кривой внизу , то из зеркального отражения симметрии видно, что площадь фигуры больше или меньше.
Как будто фигура окружена кривой, как бы заполнена положительными и отрицательными значениями . Таким образом, чтобы вираховать квадрат фигуры шукано, необходимо разбить ее на части, затем
Как плоскую фигуру в окружении двух кривых , то її площадь можно узнать дополнительной площади двух криволинейных трапеций:
стыка. Вычислить площадь фигуры, окруженной линиями:
Раствор. 1) Сделаем параболу прямой в координатной плоскости (Малюнок до встречи).
2) Мы видим (заштриховали) фигуру, обведенную этими линиями.
Малышка до завдання
3) Мы знаем точку абсцисс перекладины параболита и прямой. Для кого мы видим
систему в способе поринга:
Площадь фигуры известна как разность площадей криволинейных трапеций,
окружен параболой и прямой линией.
5) Видповид.
Алгоритм решения задач на вычисление площади фигуры, окруженной заданными линиями:
Оставаться в одной координатной плоскости заданной линии.
Заштрихуйте фигуру, окруженную этими линиями.
Определить интеринтеграцию (узнать точку абсцисс линии кривых).
Рассчитайте площадь фигуры, составив необходимую формулу.
Написать записку
2) Виконаите взять задание по одному из вариантов:
Менеджер. Вычислить площади фигур, окруженных линиями (корректировать алгоритм решения задач на вычисление площадей фигур):
Усна робот 1. Проверить с помощью интеграла площади фигур, изображенных на рисунках:
2. Вычислить интегралы:
Найдите площадь фигуры:
5) 1/3; log2 ;√2
Троки истории
«Интеграл» придумав Яков Бернулли (1690р.)
«изобретать» от латинского интегро
«цилий» как латинское целое число
«Примитивная функция»,
9 0002 Латинский
примитивный — Коб,
Жозеф Луи Лагранж
Интеграл давно
Первый известный способ розрахунку итеграів є Евдоксов наместный метод ( примерно 370 к св. е.), пытавшийся узнать площадь той обсяги, розрываючи їх на несшитых обезличенных частях , мы покрываем площадь чи обсяг вже відомы.
Цей метод був пидопления та розвинены Архимеда , и выкористовувався за параболическую площадь розраговки и площадку кола, примыкающую к розгонке.
Евдокс Книдский
Исаак Ньютон (1643-1727)
Самый последний вывод дифференциального и интегрального расчета в
Переменные величины — свободно
Изменение скорости свободно — поток (дополнительно)
Лейбниц Готфрид Вильгельм (1646-1716)
9 0028
первый використы Лейбниц например
Символ скрытия от букв
S — короткое слово
сумма (Сума)
Формулы расчета площади фигур, заштрихованных на младших
Алгоритм расчета площади плоской фигуры :
За головой ума добавьте схематическое кресло.
Покажите нужную вам функцию, например сумму или разность криволинейных площадей трапеции, выберите правильную формулу.
Знать между интеграцией (и тем, что б) помыть завдання или стул, потому что вони не дано.
Рассчитать площадь кожной криволинейной трапеции и площадь фигуры шукано.
ЗАДАНИЕ
Перед школьной кабиной соорудили клумбу. Эль, форма клумбы может быть круглой, квадратной или прямоугольной. Она виновна в мести своим прямым и кривым линиям. Пусть она будет плоской фигурой, обведенной линиями
Y = 4/X + 2; Х=4; Y = 6.
Рассчитаем площадь снятой фигуры по формуле:
de f(x)=6 , a г(х)=4/х +2
Итак, так как за квадратный метр кожи платят 50 рублей, то доход составит:
6,4 * 50 \u003 д 320 (руб. ).
Домашние задания:
1125 Вычисление площадей плоских фигур с помощью интеграла Рыбина, Н.В. Федотова 0 Минобрнауки РФ СПО Укладычи С.Л. Рыбина, Н.В. Федотова Воронеж 2015 1 Вычисление площади плоских фигур для дополнительного интеграла: Методические введения к самостоятельной работе по математике для студентов 1 курса СПО/Воронежка ДАГУ; сост. : С.Л. Рыбина, Н.В. Федотов. – Воронеж, 2015. – с. Даны теоретические сведения о том, как вычислить площади плоских фигур для дополнительного интеграла, приведены приложения задач, даны задания для самостоятельной работы. Возможен выигрыш за подготовку индивидуальных проектов. Предназначен для студентов 1 курса факультета среднего профессионального образования. Ил. 18. Библиография: 5 наименований. УДК 51:373(07) ББК 22.1й721 Стремиться к решениям на исходно-методической основе ради Воронежского ДАСУ Рецензент – Глазкова Мария Юрьевна, к.т.н. физ.-мат. наук, доцент, Вкладч кафедры высшей математики Воронежского ДАГУ В пункте 1 выполнены теоретические расчеты для вычисления площади плоских фигур для дополнительного интеграла, в пункте 2 дано приложение задач, а в пункте 3 предлагаются задания для самостоятельной работы. Основной позицией самостоятельной работы студентов является такая же роботизация, так как они одерживают победу над главой произведения, без какого-либо опосредованного участия (але под керівнитством) в специальных представлениях на этот час. Цели и задачи самостоятельной работы: систематизация и закрепление знаний и практических знаний и умений студентов; разрушение этого расширения теоретических и практических знаний; формирование ума к подборке специальной, справочной литературы, Интернета; развитие учебных достижений и активности обучающихся, творческой инициативы, самостоятельности, жизнеспособности и организованности; формирование самостоятельности мышления, стремления к саморазвитию, саморазвитию и самореализации; развитие новейших знаний. базовые знания для профессиональной подготовки аспиранта соответствуют требованиям ФГОС; формирование и развитие глобальных компетенций, закрепленных в ФГОС среднего профессионального образования; подготовка к формированию и развитию профессиональных компетенций, что способствует систематизации основных видов профессиональной деятельности, закреплению, разрушению и расширению теоретических знаний и практических знаний обучающихся; развитие учебных достижений и активности студентов: творческой инициативы, самостоятельности, жизнеспособности и организованности; формирование самостоятельности мышления: построение на саморазвитие, саморазвитие и самореализацию; володиння практический новичок застосування информационно-коммуникационных технологий в профессии ияльности; развитие последних смертей. Критериями оценки результатов самостоятельной работы студента на уроке являются усвоение студентом основного материала; 3 умных студента, чтобы выиграть теоретические знания в конце дня; обґрунтованість и читкість відповіді відповіді; оформление материала понятно требованиям ФГОС. 4 1. Расчет площади плоских изделий с помощью интегрального 1. Доводочный материал. 1.1. Криволинейной трапецией называется фигура, окруженная графиком непрерывной и неотрицательной функции y=f(x), снизу от оси Ox, а по бокам прямыми линиями x=a, x=b ( Рис. 1) 1 Площадь криволинейной трапеции можно вычислить с помощью дополнительного интеграла: b S f x dx F x b a F b (1) F a a 1.2. Пусть функция y=f(x) непрерывна с другой стороны и принимает положительные значения с другой стороны (рис. 2). Затем необходимо разбить валки на части, затем вычислить по формуле (1) общую площадь до этих частей, а площадь сложить. S = S1 + S2 c S b f x d x f x d x a (2) c 2 1.3. Для этого, если функция f (x) непрерывна g(x) на всем интервале (a; b). Для какого направления площади цифры рассчитываются по формуле y b S = (f (x) g (x)) dx y = f (x) (4) a 1 -1 O -1 b 1 y = g (х) х Рис. 4 1.5. Задание на расчет площади плоских изделий можно выполнить по такому плану: 1) для задания на ум — эскизное кресло; 2) представить фигуру шукана в виде суммы или разности площадей криволинейных трапеций. Завдання, что кресло обозначают взаимосвязью для обшивки склада криволинейной трапеции; 3) записать скин-функцию как f x ; 4) рассчитать площадь кожной криволинейной трапеции и фигуры шукано. 6 2. Применить разбивку задач 1. Вычислить площадь криволинейной трапеции, окруженной линиями у = х + 3, у = 0, х = 1 и х = 3. . САБСД = В_дпов_д: 10. 2. Фигура, окруженная линиями у = -2х + 8, х = -1, у = 0, разделена линией у = х2 — 4х + 5 на две части. Знать площадь участка кожи. Решение: Давайте посмотрим на функцию y = x2 — 4x +5. у = х2 — 4х + 5 = (х2 — 4х + 4) — 4 + 5 = (х — 2) 2 + 1, т.е. график функции представляет собой параболу с вершиной K(2; 1). САБК = . 7 САБКМЭ = С1 = САБКМЭ + СЭМК, С1 = С2 = САБК — С1, С2 = Видповид: i = . . 3. Задания для самостоятельной работы Обычный тест 1. Какую фигуру называют криволинейной трапецией? 2. Как узнать площадь криволинейной трапеции? 4. Найдите площадь заштрихованной фигуры: 8 5. Назовите формулу вычисления площади изображения фигур: Письменный тест 1. На какой фигурке изображена фигура, а не криволинейная трапеция? 2. Для дополнительных формул Ньютона-Лейбница вычислите: A. Первичную функцию; Б. Площадь криволинейной трапеции; Б. Интеграл; Г. Отпусти. 3. Знайте площадь заштрихованной фигуры: 9А. 0; БИ 2; В 1; Д. 2. 4. Найдите площадь фигуры с бахромчатым небом О и параболой у = 9 — х2 А. 18; Б. 36; ст. 72; Д. Не подсчитывается. 5. Найдите площадь фигуры, окруженной графиком функции у = sin х, прямыми х = 0, х = 2 и всеми абсциссами. А. 0; Пчела. 2; В 4; Д. Не подсчитывается. Вариант 1 Вычислите площадь фигуры, окруженной линиями: а) у х2, б) у х2 в) у cos х, г) у 1, х3 у 0, х у 0; х, у 0, 0, 4; х х 1, х 0, х 6; 2. 10 Вариант 2 Вычислите площадь фигуры, окруженной линиями: б) у 1 2 х, у 2 х2 2 х, в) у sin х, г) у 1, х2 а) у 0, х у 0 ; 0, х 0, х 3; 3 2; х 1. Вариант 3 Вычислите площадь фигуры, окруженной линиями: а) у = 2 — х3, у = 1, х = -1, х = 1; б) у = 5 — х2, у = 2х2 + 1, х = 0, х = 1; в) у = 2sin х, х = 0, х = р, у = 0; г) у = 2х — 2, у = 0, х = 3, х = 4. Вариант 4 Вычислите площадь фигуры, окруженной линиями: а) у = х2 + 1, у = 0 , х = — 1, х = 2; б) у = 4 — х2 i у = х + 2; в) у = х2 + 2, у = 0, х = — 1, х = 2; г) у = 4 — х2 i у = 2 — х. Вариант 5 Вычислите площадь фигуры, окруженной линиями: а) 7 х, х=3, х=5, у=0; б) у в) у г) у 8, х = — 8, х = — 4, у = 0; х 0,5 х 2 4 х 10, у х 2; х 2, у х 6, х = -6 и оси координат. 11 Вариант 6 Вычислите площадь фигуры, обведенной линиями а) у 4 х 2, у=0; б) у cos х, х, х в) у х 2 8 х 18, у г) у х, у 2, у = 0; 2х 18; 1 х = 4. х Вариант 7 Вычислить площадь фигуры, окруженной линиями а) у х 2 6 х, х = -1, х = 3, у = 0; б) у=-3х, х=1, х=2, у=0; в) у х 2 10 х 16, у = х + 2; г) у 3 х, у = -х + 4 и оси координат. Вариант 8 Вычислите площадь фигуры, окруженной линиями а) y sin x, x 3, x, y = 0; б) у х 2 4 , х=-1, х=2, у=0; в) у х 2 2 х 3, у 3 х 1; г) у х 2, у х 4 2, у = 0, Вариант 1 1. Вычислить площадь фигуры, окруженной линиями: а) у = х2, х = 1, х = 3, у = 0; б) у = 2cos х, у = 0, х = — Вт ш, х =; 2 2 в) у = 2х2, у = 2х. 2. (дополнительно) Найти площадь фигуры, окруженной графиком функции у = х2 — 2х + 3, так, чтобы график был проведен в його точке с абсциссой 2 и прямой х = -1. 12 Вариант 2 1. Вычислите площадь фигуры, окруженной линиями: а) у = х3, х = 1, х = 3, у = 0; б) у = 2cos х, у = 0, х = 0, х = W; 2 в) у = 0,5 х2, у = х. 2. (дополнительно) Найти площадь фигуры, окруженной графиком функции у = 3 + 2х — х2, так, чтобы график располагался в точке у-о с абсциссой 3 и прямой х = 0. Вариант 3 1. Вычислить площадь фигуры, обведенной линиями: а) у = х, х=1, х=2, у=0; б) y = 2cos x, y = 0, x = W 3W , x = ; 2 2 в) у = х2, у = -х2 + 2. 2. (дополнительно) Найти площадь фигуры, окруженной графиком функции у = 2х — х2, так что график построен в точке y-o-t от оси абсцисс 2 и оси y. Вариант 4 1. Вычислить площадь фигуры, окруженной линиями: а) у = 0,5 х, х = 1, х = 2, у = 0; б) y = 2cos x, y = 0, x = W W , x = ; 4 2 в) у = 9- х2, у = 2х + 6. 2. (дополнительно) Найти площадь фигуры, окруженной графиком функции у = х2 + 2х, так, чтобы график был проведен в у-о -точка от оси абсцисс -2 и оси у. Задания для работы в парах: 1. Вычислить площадь заштрихованной фигуры 2. Вычислить площадь заштрихованной фигуры 13 3. Вычислить площадь заштрихованной фигуры 14 5. Вычислить площадь заштрихованной фигуры 14 заштрихованная фигура 6. Представьте площадь в виде трапеций кругов. 7. Покажите площадь заштрихованной фигуры как сумму разности хи площадей криволинейных трапеций, окруженных графиками, которые показывают вам линии. 15 Библиографический список 1. Шарыгин И. Ф. Математика: алгебра и основы математического анализа, геометрия. Геометрия. Базовый уровень. 10 — 11 класс: тьютор / И.Ф. Шарыгин. — 2-й вид., стерт. – Москва: Дрофа, 2015. – 238 с. 2. Муравин Г.К. Математика: алгебра и основы математического анализа, геометрия. Базовый уровень. 11 класс: ассистент / Г. К. Муравин, О. В. Муравина — 2-й вид, стер. – Москва: Дрофа, 2015. – 189 с.п. 3. Муравин Г.К. Математика: алгебра и основы математического анализа, геометрия. Базовый уровень. 10 класс: ассистент / Г. К. Муравин, Муравина О. В. — 2-й вид., стерт. — Москва: Дрофа, 2013. — 285 с. 4. Преподавание геометрии в 10-11 классах: Метод. рекомендации к навч.: Кн. для учителя / С. М. Саакян, В. Ф. Бутузов. — 2-й вид. — М.: Просвитництво, 2014. — 222 с.: ил. 5. Изучение алгебры и начало анализа в 10-11 классах: Учеб. для учителя / Н.Я. Федорова, М. В. Ткачева. — 2-й вид. — М.: Просвитництво, 2014. — 205 с.: ил. 6. Алгебра и анализ в початке. 10-11 кл.: На две части. Часть 1: Учебник для глобального образования. установка / Мордкович А.Г. — 5 род. – М.: Мнемозина, 2014. – 375 с.: ил. Интернет-ресурсы: 1. http://www.exponenta.ru/educat/links/l_educ.asp#0 – корисн_ сообщение на сайте математической и светотехнической направленности: Исходные материалы, тесты 2. http://www.fxyz. ru/ — Интерактивный справочник по формулам и сведениям из алгебры, тригонометрии, геометрии, физики. 3. http://maths.yfa1.ru — Справочник по извлечению материала из математики (арифметика, алгебра, геометрия, тригонометрия). 4. allmatematika.ru — Основные формулы алгебры и геометрии: те же преобразования, прогрессии, изменения, стереометрии и другие. 5. http://mathsun.ru/ — История математики. Биографии великих математиков. 16 бесплатных интро. …………………………………………. . ………………………………………… .. …………………………… 3 Вычисление площадей плоских фигур с помощью интеграла…. …………………………………………….. .. 5 1. Заключительный материал …………………………………. ………………………………………………….. …………. 5 2. Применить развязку задач …………………. …………….. .. …………………………. …………….. .. .. .. 7 3.
Безлимит интернета для смартфона. Тарифы и пакеты А1
Пакет безлимит на мессенджеры включен в абонентскую плату тарифов «Комфорт S», «Комфорт М», «Комфорт L», «Комфорт XL», «lemon Z», «lemon Y», «lemon X» на социальные- в абонентскую плату тарифов «Комфорт М», «Комфорт L», «Комфорт XL», «lemon Y», «lemon X».
В августе 2020 сервис Telegram ввел новые инструменты антицензуры для пользователей в Беларуси, которые затрудняют распознавание некоторой части интернет-трафика операторами связи. Для абонентов A1 это означает, что при использовании услуги «Безлимит на мессенджеры» в настоящее время возможно расходование основного интернет-трафика, включенного в тарифный план или дополнительный пакет. Ожидается, что при снятии ограничений со стороны сервиса Telegram корректное распознавание трафика будет восстановлено. Во избежание дополнительных расходов на интернет-трафик при помегабайтной тарификации после расходования основного трафика компания A1 рекомендует подключить пакет «Безлимитный интернет на скорости до 512 Кбит/с» (либо аналогичные пакеты на скорости 1 или 2 Мбит/с).
Трафик пакетов «Безлимит на музыку», «Безлимит на YouTube», «Безлимит на соц. сети», «Безлимит на мессенджеры» не расходуется:
при загрузке или обновлении приложений с Google Play, App Store или Windows Store
при переходе на сторонние ресурсы
при просмотре видео и прослушивании аудио со сторонних ресурсов
при использовании сайтов через прокси-браузеры: Opera Mini, Internet Explorer Mobile, UCWeb Browser и др., а также с настройками прокси в телефоне или браузере
при использовании VPN
при использовании сайтов на смартфонах Blackberry
при прямом обмене данными между абонентами компании, использовании функционала звонков в ВКонтакте
некоторые ресурсы приложений, подгружаемые со сторонних серверов (например, live video, карты в Instagram)
видеозвонки через месенджеры/стриминг и социальные сети не распознаются
В случае неполадок, влияющих на распознавание интернет-трафика, доступ на ресурсы, может осуществляться с расходованием трафика, включенного в абонентскую плату тарифного плана, а также трафика пакетов «500 МБ», «1 ГБ», «3 ГБ», «Турбокнопка», «Безлимитный интернет до 512 Кбит/с», «Безлимитный интернет до 1 Мбит/с», «Безлимитный интернет до 2 Мбит/с» при их наличии.
Приобрести пакет «Турбокнопка» можно, как в процессе использования включенного трафика тарифного плана, так и после его израсходования. Период предоставления услуги – с момента подключения до 23:59:59 этого же дня. Услуга не отключается:
При смене тарифного плана в рамках линейки тарифных планов «Комфорт», «Комфорт+ для бизнеса», тарифных планов «Комфорт+», «Комфорт S», «Комфорт M», «Комфорт L», «Комфорт XL», Light для бизнеса, «Удобный», «Лето зовет»;
При одновременном использовании пакета «Турбокнопка» и услуги «Пауза». При наличии подключенной услуги и статуса абонента, отличный от «активный», после восстановления обслуживания, тарификация мобильного интернета будет осуществляться по стоимости тарифного плана. Возможность пользования пакетом «Турбокнопка» будет предоставлена на следующие сутки.
Включенный трафик не расходуется в роуминге. Оплата в роуминге интернет-трафика — по роуминговым тарифам.
Пакет «Безлимит интернета»:
Вместе с тарифным планом «Без Лимита» всем абонентам по умолчанию подключен пакет «Безлимит интернета», а также предоставляется возможность управления данным пакетом (подключение/отключение).
Абоненты тарифного плана «Без Лимита» могут самостоятельно подключить/отключить пакет «Безлимит интернета» в каналах самообслуживания, за исключением USSD и ИССА.
Стоимость пакета списывается ежедневно равными долями пропорционально количеству дней в месяце.
После окончания рекламной акции стоимость пакета составит 10,50 руб/мес с НДС.
Период предоставления пакета – календарный месяц с автоматическим продлением. Трафик предоставляется в полном объеме в течение 24 часов с момента подключения, продления пакета.
7.2 — Арифметические последовательности
7.2 — Арифметические последовательности
Арифметическая последовательность – это последовательность, в которой разница между последовательными
условия постоянны.
Общая разница
Поскольку это различие является общим для всех последовательных пар терминов, оно называется
общая разница. Обозначается д. Если разница в последовательных
термов непостоянна, то последовательность не является арифметической. Общая разница
можно найти, вычитая два последовательных члена последовательности.
Формула обыкновенной разности арифметической прогрессии: d = a n+1 —
а н
Общие условия
Арифметическая последовательность является линейной функцией. Вместо y=mx+b мы пишем a n =dn+c
где d — общая разность, а c — константа (не первый член
последовательность, однако).
Рекурсивное определение, поскольку каждый термин находится путем добавления общего различия
к предыдущему члену равно a k+1 =a к +д.
Для любого члена последовательности мы добавили разницу на один раз меньше, чем
номер срока. Например, для первого члена мы не добавили
разница вообще (0 раз). Для второго члена мы добавили разницу
один раз. Для третьего члена мы добавили разницу два раза.
Формула для общего члена арифметической прогрессии: a n =
а 1 + (n-1) d
Частичная сумма арифметической последовательности
Серия – это сумма последовательности. Мы хотим найти n th парциальных
сумма или сумма первых n членов последовательности. Обозначим n th парциальных
сумма как S n .
Рассмотрим арифметический ряд S 5 = 2 + 5 + 8 + 11 + 14. Здесь
это простой способ вычислить сумму арифметического ряда.
S 5 = 2 + 5 + 8 + 11 + 14
Ключ в том, чтобы изменить порядок терминов. Сложение коммутативно, поэтому изменение
порядок не меняет сумму.
S 5 = 14 + 11 + 8 + 5 + 2
Теперь сложите эти два уравнения вместе.
2*S 5 = (2+14) + (5+11) + (8+8) + (11+5) + (14+2)
Обратите внимание, что каждая из этих сумм в правой части равна 16. Вместо записи
16 (сумма первого и последнего членов) пять раз, мы можем записать это как 5 *
16 или 5*(2+14)
2*S 5 = 5*(2 + 14)
Наконец, разделите все это на 2, чтобы получить сумму, а не удвоенную сумму
С 5 = 5/2 * (2 + 14)
Я намеренно не упростил 2+14, чтобы вы могли видеть, где числа
родом из. Эта сумма будет равна 5/2 *(16) = 5(8) = 40,
.
Теперь, если мы попытаемся понять, откуда взялись разные части этой формулы
Отсюда мы можем сделать предположение о формуле для n th частичной суммы.
5 потому, что было пять терминов, n. 16 это сумма первого и
последние термины, a 1 + a n . 2 потому что мы добавили сумму
дважды и останется 2. Следовательно, сумма первых n членов арифметического
последовательность S n =n/2*(a 1 +a n )
Существует еще одна формула, которая иногда используется для n th частичного
сумма арифметической прогрессии. Он получается подстановкой формулы
общий термин в приведенную выше формулу и упрощение. Предпочтительный метод
это пойти дальше и найти термин n th , а затем просто подставить это число
в формулу.
S n = n/2 * ( 2a 1 + (n-1) d )
Пример
Найдите сумму от k=3 до 17 из (3k-2).
Первый член находится путем подстановки k=3 в 3k-2, чтобы получить 7. Последний
срок равен 3(17)-2 = 49. Всего 17 — 3 + 1 = 15 терминов. Итак, сумма равна 15 /
2 * (7 + 49) = 15 / 2 * 56 = 420.
Обратите внимание, что здесь 15 терминов. Когда нижний предел суммирования
равно 1, то несложно выяснить, каково количество терминов. Однако,
когда нижним пределом является любое другое число, это, кажется, вызывает у людей затруднения.
Никто не станет спорить с тем, что если перейти от 1 к 10, получится 10 чисел. Однако,
разница между 10 и 1 всего 9. Итак, когда вы находите число
терминов, это верхний предел минус нижний предел плюс один.
Какая последовательность получится, если начальные условия \(a_0 = 1\text{,}\) \(a_1 = 2\text{?}\) Приведите замкнутую формулу для этой последовательности.
Какую последовательность вы получите, если начальные условия будут \(a_0 = 1\text{,}\) \(a_1 = 3\text{?}\) Приведите замкнутую формулу.
Что если \(a_0 = 2\) и \(a_1 = 5\text{?}\) Найдите замкнутую формулу.
Мы видели, что часто легче найти рекурсивные определения, чем замкнутые формулы. К счастью для нас, есть несколько методов преобразования рекурсивных определений в закрытые формулы. Это называется решением рекуррентного соотношения . Напомним, что рекуррентное отношение — это рекурсивное определение без начальных условий. Например, рекуррентное соотношение для последовательности Фибоначчи имеет вид \(F_n = F_{n-1} + F_{n-2}\text{.}\) (это вместе с начальными условиями \(F_0 = 0\) и \(F_1 = 1\) дать полное рекурсивное определение для последовательности. )
Пример 2.4.1
Найти рекуррентное соотношение и начальные условия для \(1, 5, 17, 53, 161, 485\ldots\text{.}\)
Решение
Нахождение рекуррентного отношения было бы проще, если бы у нас был некоторый контекст для проблемы (например, Ханойская башня). Увы, у нас есть только последовательность. Помните, что рекуррентное отношение говорит вам, как перейти от предыдущих терминов к будущим терминам. Что здесь происходит? Мы могли бы посмотреть на различия между терминами: \(4, 12, 36, 108, \ldots\text{.}\) Обратите внимание, что они увеличиваются в 3 раза. Является ли исходная последовательность такой же? \(1\cdot 3 = 3\text{,}\) \(5 \cdot 3 = 15\text{,}\) \(17 \cdot 3 = 51\) и так далее. Похоже, что мы всегда получаем на 2 меньше, чем следующий член. Ага!
Таким образом, \(a_n = 3a_{n-1} + 2\) — это наше рекуррентное соотношение, а начальное условие — \(a_0 = 1\text{.}\)
Мы попробуем решить эти рекуррентные отношения. Под этим мы подразумеваем что-то очень похожее на решение дифференциальных уравнений: мы хотим найти функцию \(n\) (замкнутая формула), которая удовлетворяет рекуррентному соотношению, а также начальному условию. 2 Рекуррентные соотношения иногда называют разностными уравнениями, поскольку они могут описывать разницу между членами, и это еще больше подчеркивает связь с дифференциальными уравнениями. Как и в случае с дифференциальными уравнениями, найти решение может быть сложно, но проверить правильность решения легко. 9п +1\\
\амп = а_н.
\конец{выравнивание*}
Вот что говорит наше рекуррентное соотношение! У нас есть решение.
Иногда мы можем быть умнее и решить рекуррентное соотношение путем проверки. Мы генерируем последовательность, используя рекуррентное отношение, и отслеживаем, что мы делаем, чтобы увидеть, как перейти к нахождению только термина \(a_n\). Вот два примера того, как вы можете это сделать.
Телескопирование относится к явлению, когда многие члены в большой сумме сокращаются, поэтому сумма «телескопирует». Например:
, потому что каждый третий член выглядит так: \(2 + -2 = 0\text{,}\), а затем \(3 + -3 = 0\) и так далее.
Мы можем использовать это поведение для решения рекуррентных соотношений. Вот пример.
Пример 2.4.3
Решить рекуррентное соотношение \(a_n = a_{n-1} + n\) с начальным членом \(a_0 = 4\text{.}\)
Решение
Чтобы почувствовать рекуррентное соотношение, запишите первые несколько членов последовательности: \(4, 5, 7, 10, 14, 19, \ldots\text{.}\) Посмотрите на разницу между терминами. \(a_1 — a_0 = 1\) и \(a_2 — a_1 = 2\) и так далее. Ключевым моментом здесь является то, что разница между термами равна \(n\text{.}\) Мы можем записать это явно: \(a_n — a_{n-1} = n\text{.}\) Конечно, мы мог бы прийти к этому выводу непосредственно из рекуррентного соотношения, вычитая \(a_{n-1}\) с обеих сторон.
Теперь используйте это уравнение снова и снова, каждый раз меняя \(n\):
Сложите все эти уравнения вместе. В правой части мы получаем сумму \(1 + 2 + 3 + \cdots + n\text{.}\) Мы уже знаем, что это можно упростить до \(\frac{n(n+1)} {2}\text{.}\) Что происходит с левой стороны? Получаем
Это сумма телескопов. У нас остались только \(-a_0\) из первого уравнения и \(a_n\) из последнего уравнения. Собрав все вместе, мы получим \(-a_0 + a_n = \frac{n(n+1)}{2}\) или \(a_n = \frac{n(n+1)}{2} + a_0\text {.}\) Но мы знаем, что \(a_0 = 4\text{.}\) Таким образом, решение рекуррентного соотношения с учетом начального условия равно 9n f(k)\text{,}\), поэтому нам нужно знать замкнутую формулу для этой суммы.
Однако телескопирование не поможет нам с рекурсией, такой как \(a_n = 3a_{n-1} + 2\), поскольку левая часть не будет телескопироваться. У вас будет \(-3a_{n-1}\) , но только один \(a_{n-1}\text{.}\) Однако мы все еще можем быть умными, если используем итерацию .
Мы уже видели пример итерации, когда нашли замкнутую формулу для арифметической и геометрической последовательностей. Идея в том, что мы повторим процесс нахождения следующего члена, начиная с известного начального условия, пока не получим \(a_n\text{.}\) Затем упростим. В примере с арифметической последовательностью мы упростили, умножив \(d\) на количество раз, которое мы добавляем к \(a\), когда мы получаем \(a_n\text{,}\), чтобы получить из \(a_n = a + d + d + d + \cdots + d\) до \(a_n = a + dn\text{.}\)
Чтобы увидеть, как это работает, давайте рассмотрим тот же пример, который мы использовали для телескопирования, но на этот раз используем итерацию.
Пример 2.
4.4
Использовать итерацию для решения рекуррентного соотношения \(a_n = a_{n-1} + n\) с \(a_0 = 4\text{.}\)
Решение
Снова начните с записи рекуррентного соотношения, когда \(n = 1\text{.}\) На этот раз не вычитайте члены \(a_{n-1}\) с другой стороны:
Перегруппировав термины, мы замечаем, что \(a_n\) это просто \(a_0\) плюс сумма целых чисел от \(1\) до \(n\text{.}\) Итак, поскольку \(a_0 = 4\текст{,}\)
Конечно, в этом случае нам еще нужно было знать формулу суммы \(1,\ldots,n\text{.}\) Попробуем выполнить итерацию с последовательностью, для которой телескопирование не работает.
Итерация может быть запутанной, но когда рекуррентное отношение ссылается только на один предыдущий термин (и, возможно, на некоторую функцию от \(n\)) оно может работать хорошо. Однако попытка повторения рекуррентного отношения, такого как \(a_n = 2 a_{n-1} + 3 a_{n-2}\), будет слишком сложной. Нам потребуется отслеживать два набора предыдущих терминов, каждый из которых выражается двумя предыдущими терминами, и так далее. Длина формулы будет расти экспоненциально (фактически удваиваться каждый раз). К счастью, существует метод решения рекуррентных соотношений, который очень хорошо работает с такими отношениями.
ПодразделТехника характерного корня
¶
Предположим, мы хотим решить рекуррентное отношение, выраженное в виде комбинации двух предыдущих терминов, например \(a_n = a_{n-1} + 6a_{n-2}\text{.}\) Другими словами, мы хотим найти функцию \(n\), которая удовлетворяет \(a_n — a_{n-1} — 6a_{n-2} = 0\text{. 2 + \ альфа х + \ бета
\end{уравнение*}
9н.
\end{уравнение*}
Возможно, наиболее известным рекуррентным соотношением является \(F_n = F_{n-1} + F_{n-2}\text{,}\), которое вместе с начальными условиями \(F_0 = 0\) и \(F_1 = 1\) определяет последовательность Фибоначчи. Но заметьте, что это именно тот тип рекуррентного отношения, для которого мы можем использовать метод характеристического корня. Когда вы это сделаете, единственное, что изменится, это то, что характеристическое уравнение не учитывается, поэтому вам нужно использовать квадратичную формулу, чтобы найти характеристические корни. Фактически, это дает третье самое известное иррациональное число, \(\varphi\text{,}\) золотое сечение .
Прежде чем оставить метод характеристического корня, следует подумать о том, что может произойти при решении характеристического уравнения. У нас есть пример выше, в котором характеристический многочлен имеет два различных корня. Эти корни могут быть целыми или, возможно, иррациональными числами (для их нахождения требуется квадратичная формула). n\text{.}\) Нам повезло: 9п\), если было 3 различных корня). Также возможно решить рекуррентные соотношения вида \(a_n = \alpha a_{n-1} + \beta a_{n-2} + C\) для некоторой константы \(C\text{.}\) также возможно (и приемлемо), чтобы характеристические корни были комплексными числами.
ПодразделУпражнения
¶
1
Найдите следующие два термина в \((a_n)_{n\ge 0}\) начиная с \(3, 5, 11, 21, 43, 85\ldots.\text{.}\) Затем дайте рекурсивное определение для последовательности. Наконец, используйте метод характеристического корня, чтобы найти замкнутую формулу для последовательности. 9п\текст{.}\)
5
Найдите решение рекуррентного соотношения \(a_n = 3a_{n-1} + 4a_{n-2}\) с начальными условиями \(a_0 = 5\) и \(a_1 = 8\text{.}\)
Каково решение, если начальные условия равны \(a_0 = 1\) и \(a_1 = 2\text{?}\)
Какими должны быть начальные условия для \(a_9 = 30\text{?}\)
Для которых \(x\) существуют начальные члены, которые делают \(a_9n\) также является решением рекуррентного соотношения для любых констант \(c, d\text{. }\)
9
Вспомните волшебный автомат по производству конфет в ближайшем продуктовом магазине. Предположим, что в первый раз, когда в машину кладут четвертак, выходит 1 кегля. Во второй раз 4 кегли, в третий раз 16 кеглей, в четвертый раз 64 кегли и т.д.
Найдите как рекурсивную, так и замкнутую формулу для определения того, сколько Skittles получит n -й клиент.
Проверьте свое решение на замкнутую формулу, решая рекуррентное соотношение с помощью метода характеристического корня.
10
У вас есть доступ к \(1\times 1\) плиткам 2 разных цветов и \(1\times 2\) плиткам 3 разных цветов. Мы хотим выяснить, сколько различных \(1 \times n\) схем пути мы можем составить из этих плиток.
Найти рекурсивное определение последовательности \(a_n\) путей длины \(n\text{.}\)
Решите рекуррентное соотношение, используя метод характеристического корня.
11
Пусть \(a_n\) будет количеством \(1 \times n\) плиток, которые вы можете сделать, используя \(1 \times 1\) квадратов, доступных в 4 цветах, и \(1 \times 2\) домино, доступных в 5 цветов.